Sei sulla pagina 1di 212

TEACHING

ROUNDS
A VISUAL AID TO TEACHING
INTERNAL MEDICINE PEARLS
ON THE WARDS

Notice
Medicine is an ever-changing science. As new research and clinical experience
broaden our knowledge, changes in treatment and drug therapy are required.
T e authors and the publisher o this work have checked with sources believed
to be reliable in their e orts to provide in ormation that is complete and generally in accord with the standards accepted at the time o publication. However,
in view o the possibility o human error or changes in medical sciences, neither the authors nor the publisher nor any other party who has been involved
in the preparation or publication o this work warrants that the in ormation
contained herein is in every respect accurate or complete, and they disclaim all
responsibility or any errors or omissions or or the results obtained rom use o
the in ormation contained in this work. Readers are encouraged to con rm the
in ormation contained herein with other sources. For example and in particular, readers are advised to check the product in ormation sheet included in the
package o each drug they plan to administer to be certain that the in ormation
contained in this work is accurate and that changes have not been made in the
recommended dose or in the contraindications or administration. T is recommendation is o particular importance in connection with new or in requently
used drugs.

TEACHING
ROUNDS
A VISUAL AID TO TEACHING
INTERNAL MEDICINE PEARLS
ON THE WARDS
Navin Kumar, MD
Clinical Fellow, Medicine
Division o Gastroenterology, Hepatology, and Endoscopy
Brigham and Womens Hospital
Research Fellow in Medicine
Harvard Medical School
Boston, Massachusetts

Anica Law, MD
Clinical Fellow, Medicine
Division o Pulmonary and Critical Care Medicine
Beth Israel Deaconess and Massachusetts General Hospital
Research Fellow in Medicine
Harvard Medical School
Boston, Massachusetts

Edited By
Niteesh K. Choudhry, MD, PhD
Associate Pro essor
Department o Medicine
Brigham and Womens Hospital
Harvard Medical School
Boston, Massachusetts

New York Chicago San Francisco Athens London Madrid Mexico City
Milan New Delhi Singapore Sydney oronto

Teaching Rounds: A Visual Aid to Teaching Internal Medicine Pearls on the Wards

Copyright 2016 by McGraw-Hill Education. All rights reserved. Printed in China.


Except as permitted under the United States Copyright Act o 1976, no part o this
publication may be reproduced or distributed in any orm or by any means, or
stored in a data base or retrieval system, without the prior written permission o
the publisher.

1 2 3 4 5 6 7 8 9 0 DSS/DSS 20 19 18 17 16

ISBN 978-0-07-182162-9
MHID 0-07-182162-7
Book ISBN 978-1-259-64340-8
Book MHID 1-259-64340-9
Binder ISBN 978-1-259-64339-2
Binder MHID 1-259-64339-5

T is book was set in Minion Pro by Aptara, Inc.


T e editors were Amanda Fielding and Cindy Yoo.
T e production supervisor was Richard Ruzycka.
Project management was provided by Amit Kashyap, Aptara, Inc.
T e cover designer was T omas DePierro.
RR Donnelley Shenzhen was printer and binder.

T is book is printed on acid- ree paper.

McGraw-Hill Education books are available at special quantity discounts to use as


premiums and sales promotions or or use in corporate training programs. o contact a representative, please visit the Contact Us pages at www.mhpro essional.com.

We would like to dedicate this book to our avorite teachers (one o whom is
our Editor), who always made the process o teaching seem spontaneous and
e ortless. T is book is also or house o cers everywherewho, with a little
organization and a visual aid, can be made teachers too.
We would like to thank our consultants who contributed their time, thoughts, and
substantive edits to this project, in order by chapter: Amil Shah, MD; Ole-Petter
Hamnvik, MBBCh, BAO, MM; Molly Perencevich, MD; Christopher Gibson,
MD; Holly Rawizza, MD; Francisco Marty, MD; Jeremy Richards, MD, MA; Sagar
Nigwekar, MBBS; Eyal Kimchi, MD, PhD; Eli Miloslavsky, MD; and Sara edeschi,
MD. We learned so much about your specialties rom each o you along the way.
And f nally, we also would like to thank our respective parents and sisters, who were
our f rst and most patient teachers. We could not have arrived where we are without
your love, dedication, and support.

Contents
Preface

xiii

Cardiology
Card No.

eaching opic

eaching Category

Aortic Stenosis

Physical Exam

Cardiac Biomarkers

Diagnostic Approach

Congestive Heart Failure


Hemodynamics

Narrow Complex achycardias

Perioperative Cardiovascular Risk


Strati cation

Pulsus Paradoxus

Physical Exam

Splitting Patterns o the Second


Heart Sound

Physical Exam

S -Segment Elevations on EKG

Classi cation

Stress esting

Diagnostic Approach

Valsalva Square Wave

Physical Exam

10

Endocrinology
Card No. eaching opic

reatment Approach
Classi cation
reatment Approach

eaching Category

11

Adrenal Insu ciency

Diagnostic Approach

12

Amiodarone T yrotoxicosis

Diagnostic Approach

13

Anterior Hypopituitarism

Classi cation

14

Diabetes Mellitus ype II


Inpatient Management

15

Glucocorticoid-Induced Osteoporosis

Pathophysiology

16

Hypercalcemia Part I

Diagnostic Approach

17

Hypercalcemia Part II

reatment Approach

18

Pheochromocytoma

Diagnostic Approach

19

Polyuria

Diagnostic Approach

20

T yroid Function ests

Pathophysiology

Gastroenterology
Card No. eaching opic
21

Acute Pancreatitis

reatment Approach

eaching Category
Evidence-based Medicine

Contents (cont.)
Gastroenterology (cont.)
Card No.

eaching opic

eaching Category

22

Alcoholic Hepatitis

23

Chronic Diarrhea

Diagnostic Approach

24

Clostridium di cile

Diagnostic Approach

25

Helicobacter Pylori esting

Diagnostic Approach

26

Hepatic Encephalopathy

Pathophysiology

27

Hepatorenal Syndrome

Pathophysiology

28

Lower GI Bleed

Diagnostic Approach

29

Spontaneous Bacterial Peritonitis

Diagnostic Approach

30

Upper GI Bleeding

Evidence-based Medicine

Hematology-Oncology
Card No.
eaching opic

reatment Approach

eaching Category

31

Acute Chest Syndrome in Sickle Cell


Disease

32

B12 De ciency

Pathophysiology

33

Hemolytic Anemia

Classi cation

34

Heparin-Induced T rombocytopenia
(HI )

Diagnostic Approach

35

Monoclonal Gammopathy o
Unknown Signi cance (MGUS)
versus Multiple Myeloma

Classi cation

36

Neutropenic Fever

37

Splenomegaly

reatment Approach

reatment Approach
Physical Exam

38

rans usion reactions

Classi cation

39

umor Lysis Syndrome

Pathophysiology

40

War arin-Induced Coagulopathy

Infectious Disease
Card No.
eaching opic
41

Acute HIV

42

Asymptomatic Bacteriuria

43

Cellulitis and Erysipelas

reatment Approach
eaching Category
Diagnostic Approach
reatment Approach
Classi cation

Contents (cont.)
Infectious Disease (cont.)
Card No.

eaching opic

eaching Category

44

Community-Acquired Pneumonia

Classi cation

45

Fever o Unknown Origin

Diagnostic Approach

46

Fungal Markers

Classi cation

47

Hepatitis C, Acute vs. Chronic

Classi cation

48

Meningitis

Physical Exam

49

Osteomyelitis and Diabetic Ulcers

Physical Exam

50

uberculosis (Latent) In ection

Intensive Care
Card No. eaching opic

Diagnostic Approach
eaching Category

51

Acute Respiratory Distress Syndromes

reatment Approach

52

Acute Respiratory Distress Syndrome:


Re ractory Hypoxemia

reatment Approach

53

Albumin in Fluid Resuscitation

54

Mechanical Ventilation

55

Mechanical Ventilation: Peak


Inspiratory Pressures

56

Oxygen Supplementation

57

SEPSIS

58

Steroid Use in Sepsis

59
60

rans usion T resholds in the ICU


Vasopressors

Nephrology
Card No. eaching opic

Evidence-Based Medicine
reatment Approach
Diagnostic Approach
reatment Approach
Evidence-Based Medicine
reatment Approach
Evidence-Based Medicine
reatment Approach
eaching Category

61

AcidBase Disorders

Diagnostic Approach

62

Acute Intrinsic Kidney Injury

Classi cation

63

Erythropoietin in Chronic Kidney


Disease

Evidence-Based Medicine

64

Hypernatremia

Diagnostic Approach

65

Hyponatremia

Diagnostic Approach

Contents (cont.)
Nephrology (cont.)
Card No.

eaching opic

66

Nephrolithiasis

Classi cation

67

Non-Gap Metabolic Acidosis

Classi cation

68

Renal Artery Stenosis

Evidence-Based Medicine

69

Renal ubular Acidosis

Diagnostic Approach

70

Renal Ultrasound in Acute Kidney Injury

Evidence-Based Medicine

Neurology
Card No. eaching opic

eaching Category

eaching Category

71

Benign Paroxysmal Positional Vertigo


(BPPV)

Diagnostic Approach

72

Coma

Physical Exam

73

Delirium

reatment Approach

74

Intravenous PA in Patients with


Ischemic Stroke

reatment Approach

75

Migraine

reatment Approach

76

Neuroimaging Be ore Lumbar Puncture

Evidence-Based Medicine

77

Nystagmus

Physical Exam

78

Seizure Versus Syncope

Diagnostic Approach

79

Status Epilepticus

80

ransient Ischemic Attack

Pulmonary
Card No. eaching opic

reatment Approach
Evidence-Based Medicine
eaching Category

81

Abnormal Breathing Patterns

82

Chronic Asthma and COPD

83

COPD and Oxygenation

84

COPD Exacerbation

85

Interstitial Lung Disease

Diagnostic Approach

86

Obstructive Sleep Apnea

Pathophysiology

87

Pleural E usions

Diagnostic Approach

Physical Exam
reatment Approach
Pathophysiology
reatment Approach

Contents (cont.)
Pulmonary (cont.)
Card No.

eaching opic

eaching Category

88

Pulmonary Embolism

89

Pulmonary Function ests

Diagnostic Approach

90

Pulmonary Hypertension

Classi cation

Rheumatology
Card No. eaching opic

reatment Approach

eaching Category

91

Acute Monoarticular Arthritis

Classi cation

92

Antinuclear Antibodies

Classi cation

93

Antiphospholipid Syndrome

Diagnostic Approach

94

Chronic Gout

95

CRP versus ESR

Diagnostic Approach

96

Sarcoidosis

Diagnostic Approach

97

Scleroderma Renal Crisis

Pathophysiology

98

Systemic Lupus Erythematosus (SLE)


Flare

99
100

emporal Arteritis
Vasculitis

reatment Approach

reatment Approach
Diagnostic Approach
Diagnostic Approach

Preface
odays medical resident has access to in ormation in more orms than ever be ore.
o learn about a given disease entity, there are many options to turn topocket
manuals, textbooks, the peer-reviewed medical literature, residency-speci c housesta primers, websites, and other online resources. While this in ormation quarry
is richer than ever be ore and the ability to mine it is increasingly easy, it has also
become increasingly challenging to identi y the true pearls that one can take orward into practice.
However, one o the most important jobs o medical residents is to do exactly
that, and then to pass these pearls onto medical students, interns, and junior residents in a succinct and organized ashionnot to mention semi-spontaneously on
morning rounds when a patient with a relevant teaching point presents overnight.
As house o cers, we ound the challenge o doing all o this to be requently so
daunting that teaching too o en was sacri ced or e ciency. Despite all the available resources to help us learn, we ound none that could reliably help us teach.
T e ollowing, then, is our answer, born over our kitchen table at home. For
each o 10 medical specialties, we have identi ed 10 topics that we eel are important
to discuss on rounds, with each topic given its own card. We urther categorized the
teaching topics into speci c categories (Diagnostic Approach, reatment Approach,
Disease Classi cation, Evidence-Based Medicine, Pathophysiology, or Physical Examination), so that discussions can also be chosen by teaching content.
On the rst side o every card is a visual aid, central to the topic at hand, to
be presented to the learner(s). T e back side has a question-and-answer ormat or
the teacher to guide the discussion, hitting on the key pearls we believe to be most
important. A section o the question-and-answer side is highlighted; this section
re ers most directly to the image on the ront, and in a time-crunch, the lesson can
be limited to this section alone. Our aim is that each highlighted portion takes 2 to 3
minutes to teach, while the entire card can be taught in less than 10. Although each
card contains all o the in ormation to present a given topic, we recommend that the
teacher review the card prior to rounds, both to become amiliar with the image and
f ow o the discussion, as well as to personalize the teaching experience.

xiii

PHYSICAL EXAM

CARDIOLOGY

AorticStenosis

Hyp e rtro p h ic
Ca rd io m yo p a th y

Ao rtic Ste n o s is
S1

S2

S1

S tanding
(de c re as e d ve no us
re turn to he art)

De cre a s e d blood volume


e je cte d through a ortic
va lve de c re as e d murmur
inte ns ity

Incre a s e d re la tive
obs truction of outflow
tra ct inc re as e d
murmur inte ns ity

S quatting
(inc re as e d ve no us
re turn to he art)

Incre a s e d blood volume


e je cte d through a ortic
va lve incre a s e d murmur
inte ns ity

De cre a s e d re la tive
obs truction of outflow
tra ct de cre a s e d
murmur inte ns ity

S2

Figure 1-1. E ect o standing vs squatting on the murmurs o aortic


stenosis vs hypertrophic cardiomyopathy.

PHYSICAL EXAM

CARDIOLOGY

AorticStenosis
Case: A 72-year-old man with a history o hyperlipidemia presents with a
syncopal episode and is noted to have a systolic murmur on exam.
What valvular abnormality is concerning in the presentation o syncope?

Aortic stenosis (AS)


What are the typical characteristics o the murmur o AS?
iming: Mid-systolic; Shape: crescendo-decrescendo; Location of maximal
intensity: Right 2nd intercostal space (base o heart); Radiation: Upward to
the carotid arteries; Pitch: Low; Quality: Rough
What maneuvers can be used to di erentiate the systolic murmur o AS

rom the systolic murmur o hypertrophic cardiomyopathy (HCM)?

Maneuver

Ef ect on
Preload
(venous return
to the heart)

Change in AS
murmur intensity

Change in HCM
murmur intensity

Standing

(Less blood volume


ejected through a
stenotic valve)

(More relative
obstruction o
out ow tract)

Squatting

(More blood volume


ejected through a
stenotic valve)

(Less relative
obstruction o
out ow tract)

How does the Valsalva maneuver a ect preload?

Straining phase decreased preload (similar e ect as standing)


Release phase increased preload (similar e ect as squatting)
What other eatures on exam make aortic stenosis more or less likely?1
More likely: slow rate o rise o carotid pulse (i.e., parvus et tardus), mid-late
peaking murmur, and decreased intensity o the second heart sound (i.e., S2)
Less likely: lack o murmur radiation to the right carotid artery (negative
likelihood ratio o 0.05:0.10)

REFERENCE
1. Etchells E, Bell C, Robb K. Does this patient have an abnormal systolic murmur? JAMA. 1997;277(7):564571.

DIAGNOSTIC APPROACH

CARDIOLOGY

Myoglobin

MB CK

Tota l CK

Troponins

7
6
5
4
3
2
1

CardiacBiomarkers

0
0

8
12
16
20
24
28
32
36
40
Time a fte r ons e t of a cute myoca rdia l infa rction (h)

44

(Hass EE, Yang EH, Gersh BJ, ORourke RA. Chapter 60. ST-Segment Elevation Myocardial In arction. In:
Fuster V, Walsh RA, Harrington RA. eds. Hursts The Heart, 13e. New York, NY: McGraw-Hill; 2011.)

Figure 1-2. Temporal prof le o cardiac biomarkers.

48

CARDIOLOGY

DIAGNOSTIC APPROACH

CardiacBiomarkers
Case: A 66-year-old man with a history o hypertension and hyperlipidemia
presents with 1 hour o substernal chest pain, with 2-mm ST-segment
depressions in leads V3V6 on EKG.
What laboratory tests are indicative o myocardial damage?

Cardiac biomarkers
Myoglobin (MB), creatinine kinase (CK), creatinine kinase MB isoenzyme
(CKMB), and/or cardiac troponins (troponin I or )
Which o these cardiac biomarkers are most speci c to the myocardium?1
CK-MB (small amount also ound in skeletal muscle, small intestine,
diaphragm)
Cardiac troponins (more speci c to myocardium than CKMB; troponin I
more cardiac speci c than troponin )
Which biomarker peaks earliest? Which biomarker persists the longest?1

First detected in bloodstream: myoglobin (within 8 hours)


Last to clear bloodstream: troponins
I the same patient waited 3 days a er the episode o chest pain to seek medical

attention, and was ound to only have a troponin elevation on initial labs,
what would be the likely timing o the ischemic event?
T is is most likely a presentation o a late myocardial in arction, as the other
biomarkers (CK, CKMB) have already cleared the bloodstream
In addition to plaque rupture leading to an acute coronary syndrome, what
are other causes o cardiac troponin elevation?2
Cardiac damage due to demand ischemia (type 2 myocardial in arction)
achyarrhythmia, anemia, hypo- or hyper-tension, heart ailure, le
ventricular hypertrophy, severe aortic stenosis, pulmonary embolism,
coronary vasculitis, coronary vasospasm, sepsis, acute respiratory ailure
Cardiac damage not due to ischemia
Cardiac contusion, recent cardiac surgery or ablation therapy, myocarditis, akotsubo cardiomyopathy, cardiotoxic agents, requent de brillator
shocks
Extra-cardiac causes
Stroke

REFERENCES
1. Hass EE, Yang EH. Chapter 60. S -segment elevation myocardial in arction. In: Fuster V, Walsh RA, Harrington RA, eds. Hursts T e
Heart. 13th ed. New York, NY: McGraw-Hill; 2011.
2. Harrington R. Chapter 59. Unstable angina and nonS -segment elevation myocardial in arction. In: Fuster V, Walsh RA,
Harrington RA, eds. Hursts T e Heart. 13th ed. New York, NY: McGraw-Hill; 2011.

TREATMENT APPROACH

CARDIOLOGY

Congestive Heart Failure Hemodynamics

Co ng e s tio n
Dry

We t

Wa rm

We llcompe ns a te d

Diure s e

Cool

Inotropic
S upport

Diure s e a nd
Inotropic
S upport

Pe rfus io n

S ig ns /S ympto ms o f
Lo w Pe rfus io n
Na rrow puls e pre s s ure
Cool e xtre mitie s
Alte re d me nta l s ta tus
De cre a s e d urine output

S ig ns /S ympto ms o f
Co ng e s tio n
Orthopne a /pa roxys ma l
nocturna l dys pne a
Ede ma /a s cite s
Ele va te d J VP
Audible S 3
Cra ckle s on lung a us culta tion
He pa tojugula r re flux
Va ls a lva s qua re wa ve

Figure 1-3. The hemodynamic prof les and tailored therapies o


congestive heart ailure.

CARDIOLOGY

TREATMENT APPROACH

Congestive Heart Failure Hemodynamics


Case: A 62-year-old man with a history o severe dilated cardiomyopathy
(EF 20%) presents with shortness o breath and worsening edema.
What are the our di erent hemodynamic pro les o heart ailure?1

Warm/dry, warm/wet, cool/dry, and cool/wet


What are the signs/symptoms o low per usion (i.e., cool hemodynamics)?
Decreased pulse pressure (pulse pressure = systolic BPdiastolic BP), cool
extremities, altered mental status, decreased urine output
What are the signs/symptoms o congestion (i.e., wet hemodynamics)?
Orthopnea, paroxysmal nocturnal dyspnea, edema, ascites, elevated JVP,
audible S3, crackles on lung auscultation, hepatojugular ref ux, Valsalva
square wave
What are the treatment strategies or each hemodynamic pro les?1

Warm/dry: well compensated, no change in management


Warm/wet: diurese to reduce congestion
Cool/dry: add inotropic support to improve per usion
Cool/wet: diurese and add inotropic support
What inotropes are available or low-per usion CHF?

Beta-agonists (dopamine, dobutamine) + phosphodiesterase inhibitors


(milrinone)
Why are inodilators (e.g., dobutamine, milrinone, and low-dose dopamine)
help ul in the short-term treatment o low-per usion CHF?
In low-per usion CHF, decreased cardiac output leads to a compensatory
rise in systemic vascular resistance (SVR) to maintain blood pressure
T e increase in SVR worsens a erload, urther limiting cardiac output
Inodilators work on both actors in low-per usion CHF
Inotropy: increase cardiac contractility
Peripheral vasodilation: decrease a erload to improve cardiac output
What are the major adverse e ects o inodilator therapy?2
Arrhythmias (due to increased chronotropy)
Myocardial ischemia (due to increased myocardial oxygen consumption)
Hypotension (due to peripheral vasodilation)

REFERENCES
1. Nohria A, Lewis E, Stevenson LW. Medical management o advanced heart ailure. JAMA. 2002;287(5):628640.
2. Abraham W , Hasan A. Chapter 28. Diagnosis and management o heart ailure. In: Fuster V, Walsh RA, Harrington RA, eds.
Hursts T e Heart. 13th ed. New York, NY: McGraw-Hill; 2011.

CLASSIFICATION

CARDIOLOGY

NarrowComplexTachycardias

Na rrow Comple x
Ta chyca rdia

Re gula r or
Irre gula r?

Re gula r

Irre gula r

S inus ta chyca rdia

Atria l fibrilla tion

Atria l flutte r with fixe d block

Atria l flutte r with va ria ble block

Atria l ta chyca rdia

Atria l ta chyca rdia with va ria ble block

AVRT/AVNRT
P

Multifoca l a tria l ta chyca rdia


P

Figure 1-4. EKG eatures o narrow complex tachycardias.

CARDIOLOGY

CLASSIFICATION

NarrowComplexTachycardias
Case: A 72-year-old woman is hospitalized with pneumonia and is noted
to have a rapid heart rate on routine vital sign check. EKG is obtained and
shows a narrow-complex tachycardia.
What is the rst step in evaluating a patient with a supraventricular

tachycardia (SVT)?
Assess hemodynamic stability: i the patient is hypotensive or exhibits signs
o hypoper usion (e.g., altered mental status, cool extremities), then proceed
directly to electrical cardioversion
What are the types o SVT with a regular rhythm and their patterns on

EKG?
Sinus tachycardia (normal p wave be ore each QRS complex, and QRS
complex a er each normal p wave)
Atrial f utter with xed block (sawtooth pattern o the p waves)
Atrial tachycardia (p waves all identical but unique; that is, not rom the
SA node)
AVNR /AVR (may see retrograde p waves that arise a er the QRS
complex)
What are the types o SVT with an irregular rhythm and their patterns on
EKG?
Atrial brillation (no discernible p waves)
Atrial f utter with variable block (sawtooth pattern o p waves with
variable conduction to the ventricles)
Atrial tachycardia with variable block (unique p waves o same morphology
with variable conduction to the ventricles)
Multi ocal atrial tachycardia (p waves have at least three di erent
morphologies)
What maneuvers may be per ormed to slow or attempt to cardiovert an SVT?

Vagal maneuvers (e.g., Valsalva, carotid sinus massage, ace in cold water)
What are the medication classes or rate control in atrial brillation/f utter?
Stable (normotensive): beta-blockers or calcium channel blockers
Relative hypotension (without hypoper usion): digoxin or amiodarone
I an accessory pathway is known or suspected to exist in a patient presenting
with an SVT, what must be avoided and why?1
No AV-nodal blocking agents, which would promote conduction o the
atrial beats down the accessory pathway that has no intrinsic delay
Ventricular response to SV would signi cantly increase the ast
ventricular rhythm may degenerate into ventricular brillation

REFERENCE
1. Marchlinski F. Chapter 233. T e tachyarrhythmias. In: Longo DL, Fauci AS, Kasper DL, Hauser SL, Jameson JL, Loscalzo J, eds.
Harrisons Principles of Internal Medicine. 18th ed. New York: McGraw-Hill; 2012.

TREATMENT APPROACH

CARDIOLOGY

Perioperative Cardiovascular RiskStratifcation

Perioperative risk actor

Points

High-risk type o surgery

Ischemic heart disease

Congestive heart ailure

Cerebrovascular disease

Insulin therapy or diabetes mellitus

Preoperative serum creatinine > 2.0 mg/dL

Total RCRI points

Risk o perioperative cardiac complication (%)

0.4

0.9

11

Figure 1-5. The Revised Cardiac Risk Index (RCRI) Score or estimating
perioperative risk.1

CARDIOLOGY

TREATMENT APPROACH

Perioperative Cardiovascular RiskStratifcation


Case: A 73-year-old emale with a history o hypertension presents a ter
a all with a ractured le t hip. You are consulted or perioperative risk
assessment prior to surgery.
For patients undergoing a nonemergent surgery with no evidence o an

acute coronary syndrome, what is the rst step in perioperative risk


management?2
Estimate the perioperative risk o a major adverse cardiac event (MACE)
with a clinical prediction tool
How does the Revised Cardiac Risk Index (RCRI) estimate the perioperative

risk o MACE?1
Lee et al. identi ed six independent predictors o cardiac risk or major
noncardiac surgery and incorporated the actors into a single score
(RCRI)
High-risk surgery (intraperitoneal, intrathoracic, or suprainguinal
vascular), history o ischemic heart disease, history o congestive heart
ailure, history o cerebrovascular disease, insulin therapy or diabetes
mellitus, and preoperative creatinine level > 2.0 mg/dL
Low risk i patient has < 2 risk actors
How does the perioperative risk o MACE inf uence the next steps in

management?2
Low risk: proceed to surgery (no additional need or testing)
High risk: determine patients unctional capacity
< 4 ME s pharmacologic stress testing (i results change management)
4 ME s proceed to surgery
What activities are associated with 4 METs?2
Climbing a f ight o stairs, walking up a hill, walking on level ground at
4 mph, or per orming heavy work around the house
What is the only class I indication or perioperative beta-blocker therapy?2
Per the ACC/AHA, beta-blockers should be continued in all patients who
are already receiving beta-blockers or a clinically appropriate indication
When should statins be administered perioperatively?2
Per the ACC/AHA, statins should be continued in all patients currently
taking statins and scheduled or noncardiac surgery (class I recommendation),
and it is reasonable to start statins in patients undergoing vascular surgery
(class IIa recommendation)

REFERENCES
1. Lee H, Marcantonio ER, Mangione CM, et al. Derivation and prospective validation o a simple index or prediction o cardiac risk
o major noncardiac surgery. Circulation. 1999;100(10):10431049.
2. Fleisher LA, Fleischmann KE, Auerbach AD, et al. 2014 ACC/AHA guideline on perioperative cardiovascular evaluation and management o patients undergoing noncardiac surgery: a report o the American College o Cardiology/American Heart Association
ask Force on Practice Guidelines. Circulation. 2014;130(24):e278e333.

PHYSICAL EXAM

CARDIOLOGY

Pulsus Paradoxus

P a ra doxica l
P uls e >
10 mm Hg

Norma l
Re s pira tory
Va ria tion
(010 mm Hg)

Ke y
Norma l
P uls us
P a ra doxus

Ins pira tion

Expira tion
Re s p ira tory Cyc le

Effus ion

Effus ion

RA

LA

RA

LA

RV

LV

RV

LV

Ins pira tion

Expira tion

Figure 1-6. Blood pressure tracing (above) and pathophysiology


(below) o pulsus paradoxus.

CARDIOLOGY

PHYSICAL EXAM

Pulsus Paradoxus
Case: A 32-year-old woman with a history o breast cancer presents with
shortness o breath and hypotension, and is noted to have distant heart
sounds on exam and an elevated JVP.
What other exam ndings would raise your concern or cardiac tamponade?1

Pulsus paradoxus: a all in the systolic blood pressure (SBP) o > 10 mm Hg


with inspiration
When detected in presence o a pericardial e usion, a pulsus signi cantly
increases the likelihood o cardiac tamponade (likelihood ratio = 3.3)
How do you measure a pulsus paradoxus on exam?
1. Ask the patient to breathe normally and inf ate cu 10 mm Hg above the
point at which no Korotko sounds are heard
2. Slowly def ate cu until the rst Korotko sounds are heard (the sounds will
be intermittent, only occurring during expiration)
3. Continue to def ate cu until all Korotko sounds are heard (the sounds will
be constantnow at the SBP where all inspiratory beats are audible as well)
4. T e pulsus is the di erence in SBP between step 2 and step 3, and i
> 10 mm Hg, then there is a pulsus paradoxus
Why does the SBP drop with inspiration in normal physiology?
Inspiration decreases intrathoracic pressure, which increases preload to the
right side o the heart and thereby increases right-sided cardiac output
As a result, the pulmonary vascular bed distends, which delays lling to
the le side o the heart, and thus le -sided cardiac output alls
Why does a pulsus paradoxus occur?2

Increased pericardial pressure (e.g., rom pericardial f uid)


interdependence between the R and L sides o the heart (equalization
o diastolic pressures)
With inspiration, greater preload leads to increased right ventricular size
Due to inability to expand outward (pericardial space is restricted due
to increased pressures), the interventricular septum shi s to the le
T e le ventricle is similarly unable to expand into the pericardial space,
and thus le ventricular lling is signi cantly decreased
Decreased preload decreased cardiac output decreased SBP
In addition to cardiac tamponade, what are other causes o pulsus paradoxus?

Asthma/COPD exacerbations; tension pneumothorax; rarely in obesity/


pregnancy

REFERENCES
1. Roy CL, Minor MA, Brookhart MA, Choudhry NK. Does this patient with a pericardial e usion have cardiac tamponade? JAMA.
2007;297(16):18101818.
2. Walsh RA, Shaver JA. Chapter 14. T e history, physical examination, and cardiac auscultation. In: Fuster V, Walsh RA, Harrington
RA, eds. Hursts T e Heart. 13th ed. New York, NY: McGraw-Hill; 2011.

PHYSICAL EXAM

CARDIOLOGY

Splitting Patterns o the Second Heart Sound

A2 P 2

Expira tion
Ins pira tion

Expira tion
Ins pira tion

Expira tion
Ins pira tion

Expira tion

xi

S1

Ins pira tion


P2

Figure 1-7. Splitting patterns o the second heart sound.

A2

PHYSICAL EXAM

CARDIOLOGY

Splitting Patterns o the Second Heart Sound


Case: A 78-year-old emale is ound to have splitting o S2 on expiration.
Why is splitting o S2 important diagnostically?

In contrast to S1, where the intensity o the heart sound ref ects strength o
ventricular contraction, the split o S2 can provide greater valvular, septal,
and electrical in ormation
What is the physiology behind physiologic splitting?
Inspiration causes intrathoracic pressure to become more negative, which
causes the ollowing:
Decreased a erload inside pulmonary vessels, delaying closure o the
pulmonic valve (which is closed when a threshold o back pressure in
pulmonary vessels is reached)
Increased RV stroke volume, which increases ejection time and delays P2
Describe the physiology and list di erential diagnoses or each o the

patterns1
Pattern

Physiology

Dif erential
diagnosis

Widened
split

Permanently delayed pulmonic valve


closure. Split still varies with inspiration,
but P2 delay is more exaggerated in both
inspiration and expiration.

RBBB, pulmonic
stenosis, LV
paced

Fixed split

Chronic overload o pulmonary vascular


system causes high-capacitance, lowresistance state o the pulmonary vasculature. Inspiration does not increase the
already elevated vascular capacitance,
and closure time o the pulmonic valve is
thus already maximally delayed.

Atrial septal
de ect, RV ailure

Paradoxical
split

Abnormal delay in the closure o the


aortic valve such that it occurs a ter
P2. Inspiration delays P2, causing it to
become closer to a delayed A2.

LBBB, advanced
aortic stenosis,
RV paced

What is the best way to examine or split S2?

Listen in second or third intercostal space at le sternal border


Splitting on expiration is present in all abnormally split S2 conditions. Listen
or splitting on expiration that persists when patient sits up (physiologic
splitting during expiration occurs when patient is supine but should
extinguish when sitting up)

REFERENCE
1. Leonard L. Pathophysiology of Heart Disease. 4th ed. Baltimore, MD: Lippincott Williams &Wilkins; 2007.

ve

ke

ve

ka

yp

va

ve

ks

yn

ys

ve

CARDIOLOGY
CLASSIFICATION

ST-Segment Elevations on EKG

CARDIOLOGY

CLASSIFICATION

ST-Segment Elevations on EKG


Case: A 66-year-old man with a history o hypertension presents with
chest pain and is ound to have a STEMI. EKGs are taken serially during his
hospitalization.
What is the EKG criteria or a STEMI?1

New S -segment elevation o > 2 mm in leads V2V3 or > 1 mm in other


leads
Must be present in at least two consecutive leads
Alternatively, a new le bundle branch block in the setting o chest pain
Aside rom ischemia, what are the other causes o ST-elevation?1
Acute pericarditis, severe hyperkalemia (e.g., K > 7.0 mEq/L), hypercalcemia, Brugada syndrome, hypothermia (Osborn waves), le ventricular
hypertrophy, le ventricular aneurysm, normal variant
What is the natural progression o a STEMI on EKG?1

Increase in amplitude o -wave (minutes)


S -segment elevation (minutes): convex up or tombstone pattern
Q-wave development (hours)
-wave inversion (12 days)
S -segment normalization (several days)
-wave normalization (weeks)
Q-wave persists (weeks)
Describe the distinguishing eatures o the three examples o ST-elevation
rom nonischemic causes.
Acute pericarditis: S -segment is concave up compared to the convex
up pattern o an S EMI; PR-segment depression may also be present
Severe hyperkalemia: accompanying peaked -waves o hyperkalemia
Brugada syndrome: RSR pattern o the QRS complex (seen in lead V1);
downward sloping pattern o the S -elevation, which leads to a coved
appearance
I a STEMI is noted in the in erior leads, what type o EKG should be

obtained?2
Right-sided EKG (i.e., precordial leads placed over right chest)
S -elevation in the right-sided lead 4 (V4R) is concerning or a right
ventricular in arct, which is a preload-dependent state (need IV f uid support
and avoid venodilators such as nitroglycerin that may reduce preload)

REFERENCES
1. Bays-de-Luna A, Goldwasser D, Fiol M, Bays-Genis A. Chapter 15. Sur ace electrocardiography. In: Fuster V, Walsh RA, Harrington RA, eds. Hursts T e Heart. 13th ed. New York, NY: McGraw-Hill; 2011.
2. Hollander JE, Diercks DB. Chapter 53. Acute coronary syndromes: acute myocardial in arction and unstable angina. In: intinalli
JE, Stapczynski J, Ma OJ, Cline DM, Cydulka RK, Meckler GD, eds. intinallis Emergency Medicine: A Comprehensive Study Guide.
7th ed. New York, NY: McGraw-Hill; 2011.

DIAGNOSTIC APPROACH

CARDIOLOGY

StressTesting
Ca n the pa tie nt exe rcis e ?
(Achieve >5 METs )

No

EXERCIS E
STRES S

P HARMACOLOGIC
STRES S

Ye s

Is the EKG norma l


a t ba s eline ?

Ye s

No

No

DOBUTAMINE
STRES S

Ye s

Fre e of:
Bronchos pa s tic
a irway dis e a s e
Hypote ns ion
S ick s inus
AV block
VAS ODILATOR
STRES S

IMAGING

EKG

Are the re re giona l wa ll motion


a bnorma litie s a t ba s e line ?

No

Ye s

Is loca liza tion


of the le s ion
ne e de d?

Fre e of:
Ba s e line HTN
Ve ntricula r
a rrhythmia s
Re ce nt MI

ECHO

S P ECT

P ET-CT

Figure 1-9. Protocol or determining appropriate stress test modality.

CARDIOLOGY

DIAGNOSTIC APPROACH

StressTesting
Case: A 57-year-old diabetic male is admitted or chest pain and re erred
or stress testing.
Which patients should receive stress testing?

Patients with an intermediate coronary artery disease (CAD) probability to


either rule-in or rule-out CAD
Patients with known CAD: to risk strati y or evaluate or changes
How does one choose the appropriate type o stress?

I the patient can exercise (achieve at least 5 ME s [mean exercise test]),


exercise stress testing (bicycle or treadmill) is pre erred: physiologic and
gauges unctional capacity
I the patient cannot exercise, choose pharmacologic stress
How does one choose between pharmacologic stressors?1
Dobutamine (inotrope; increases cardiac workload) not or patients with
severe baseline H N, ventricular arrhythmias, or recent MI or unstable
angina.
Vasodilators (adenosine, regadenoson, and dipyridamole) induce selective
coronary vasodilation o less diseased vessels, causing hypoper usion o
areas at ischemic risk; not or patients with bronchospastic airway disease
or hypotension. Adenosine not or patients with sick sinus syndrome or
high-degree AV block.
When is EKG appropriate to capture results o stress?

Only in patients receiving exercise stress with a normal EKG at baseline


(no LBBB, LVH, paced rhythm, WPW, < 1 mm S -segment depression)
All other patients should receive imaging (which is needed to localize lesion)
ECHO i the patient does not have wall motion abnormalities at rest
Otherwise choose SPEC (e.g., sestamibi) or PE /C
How does one decide between PET/CT and SPECT?2
PE /C : shorter exam, uses lower doses o radiation, better images in
larger patients. Not widely available, more expensive than other imaging
modalities.
What test results are considered to signi y high risk?
On exercise: drop in BP, Duke score 11, completion o < 4 ME s
On EKG: S depression > 2 mm, S depressions persisting > 5 min a er
exercise, ventricular tachycardia, S elevations
Radionuclide imaging: 1 large or 2 mod reversible de ects, LV dilation
All the above patients should be sent to coronary angiography

REFERENCES
1. Leonard L. Pathophysiology of Heart Disease. 4th edi. Baltimore, MD: Lippincott Williams &Wilkins; 2007.
2. Alguire PC, et al. Medical Knowledge Self-Assessment Program 16: Cardiovascular Medicine. Philadelphia, PA: American College o
Physicians; 2012.

10

ko

S ys tolic Blood P re s s ure (S ta rting p oint 10 mm Hg a b ove norma l S BP )

va

va

ve

va

CARDIOLOGY
PHYSICAL EXAM

Valsalva Square Wave

PHYSICAL EXAM

CARDIOLOGY

Valsalva Square Wave


Case: A 63-year-old woman with a dilated cardiomyopathy presents with
peripheral edema.
How do you check or a Valsalva square wave as a sign o congestion?

Identi y the SBP, inf ate the cu an additional 10 mm Hg, and hold the cu
at that pressure
While listening or Korotko sounds, ask patient to Valsalva or 10 sec and
release
What are the three di erent responses to the Valsalva maneuver?1

Normal Response
Phase I
(initial
Valsalva)

WellCompensated
CHF Response

Decompensated
CHF Response

Increase in intraabdominal pressure


increase in SBP (hear
Korotko sounds)

Same as normal
response

Same as normal
response

Phase II
Decrease in venous
(sustained return decreased
Valsalva) preload decrease in
SBP (absent Korotko
sounds)

Same as normal
response

Patient is volumeoverloaded
Valsalva is not
signif cant enough
to overcome elevated venous f lling
pressures SBP
remains elevated

Phase III
(release o
Valsalva)

Decrease in SBP due to Same as normal


loss o intra-abdominal response
pressure (absent Korotko sounds)

Same as normal
response

Phase IV
(recovery
rom
Valsalva)

Arterial constriction
and increased venous
return SBP overshoot (hear Korotko
sounds again)

Patient is already vasoconstricted due to


chronic compensation or low cardiac
output no SBP
overshoot (absent
Korotko sounds)

Patient is already
vasoconstricted
due to chronic
compensation
or low cardiac
output no SBP
overshoot (absent
Korotko sounds)

How does the Valsalva square wave maneuver assist in diuresis management?

When a patient with congestive heart ailure (CHF) has been adequately
diuresed, the venous lling pressures will be restored and the SBP will drop with
sustained Valsalva during phase II (well-compensated CHF response pattern)

REFERENCE
1. Shamsham F, Mitchell J. Essentials o the diagnosis o heart ailure. Am Fam Physician. 2000;61(5):13191328.

10

DIAGNOSTIC APPROACH

ENDOCRINOLOGY

Adrenal Insuf ciency


S us pe cte d
a dre na l
ins ufficie ncy

Adre na l
ins ufficie ncy
rule d in

<3 mcg/dL

Che ck
ba s e line
6:008:00 AM
cortis ol
le ve l

>15 mcg/dL

Adre na l
ins ufficie ncy
rule d out

Che ck ACTH
le ve l
315 mcg/dL
>100 pg/mL

Low-norma l

P rima ry a dre na l
ins ufficie ncy

Ce ntra l
a dre na l
ins ufficie ncy
Che ck ACTH
le ve l
ACTH high

ACTH low

P os s ible prima ry
a dre na l
ins ufficie ncy

S us pe ct
centra l adre nal
ins ufficie ncy

CS T1

CS T1
Norma l

Abnorma l

Cons is te nt with
e ithe r norma l
function or re ce nt
ons e t ce ntra l
a dre na l
ins ufficie ncy2

Confirme d
ce ntra l
a dre na l
ins ufficie ncy

Cortis ol <1820 mcg/dL


(500550 nmol/L)

Confirme d
prima ry a dre na l
ins ufficie ncy

1 ACTH

cos yntropin s timula tion te s t (CS T): Adminis te r 250 mcg a s IV bolus a nd che ck cortis ol le ve l in 3060 minute s .
(Norma l re s pons e : cortis ol >18 mcg/dL)
2
Re ce nt s e conda ry a dre na l ins ufficie ncy s hould be s us pe cte d in pa tie nts with re ce nt pituita ry s urge ry.

(Stern SC, Ci u AS, Altkorn D. Chapter 21. I Have a Patient with Hyponatremia. I Have a Patient with
Hypernatremia. How Do I Determine the Cause?. In: Stern SC, Ci u AS, Altkorn D. eds. Symptom to
Diagnosis: An Evidence-Based Guide, 2e. New York, NY: McGraw-Hill; 2010.)

11 Figure 2-1. Diagnostic algorithm or adrenal insu ciency.

ENDOCRINOLOGY

DIAGNOSTIC APPROACH

Adrenal Insuf ciency


Case: A 53-year-old emale with SLE on long-term prednisone presents
with pneumonia and re ractory hypotension. You are considering a
diagnosis o adrenal insu ciency (AI).
What are the types o AI and their causes?1,2

1: loss o adrenal gland (glucocorticoid AND mineralocorticoid) unction,


e.g., autoimmune, in ection ( B, CMV, etc.), hemorrhage, malignancy.
Autoimmune, in ection ( B, CMV, MAI), hemorrhage, malignancy.
Central: loss o unction upstream o adrenal gland (pituitary or hypothalamus) loss o glucocorticoid unction only.
2: loss o pituitary AC H secretion, e.g., Sheehan syndrome, pituitary
tumor.
3: loss o hypothalamic CRH secretion, e.g., due to exogenous steroids.
How should AI be worked up?1,2

Check AM cortisol:
I > 15 mcg/dL, AI unlikely.
I < 3 mcg/dL, AI ruled in. Check AC H. High = 1AI; low = central AI.
I 315 mcg/dL, check AC H and do cosyntropin test:
Administer cosyntropin 250 mcg (AC H analog). Cortisol < 18.0 mcg/
dL 60 minutes a er cosyntropin con rms AI.
What are limitations to measuring random cortisol?

Level uctuates during the day. T e available assay measures O AL cortisol,


not physiologically active ree cortisol. otal cortisol varies by protein state
and binding a nity (e.g., pregnancy, critical illness). Free cortisol may be
normal.
How does empiric steroid administration alter results o cosyntropin test?1,2
All steroids will suppress the adrenal axis. Hydrocortisone/prednisone is
arti actually measured along with native cortisol in the cortisol assay.
Dexamethasone is not; a cosyntropin test can still be interpreted accurately.
How does workup and management o AI di er in critically ill patients?
No agreed upon diagnostic criteria or random cortisol level.
Concept o relative adrenal insu ciency (i.e., protracted hypotension in
critically ill patient with normal cortisol) is controversial. Steroids should be
given to reduce in ammatory response rather than treat adrenal dys unction.

REFERENCES
1. Arlt W. Chapter 342: Disorders o the adrenal cortex. In: Longo DL, Fauci AS, Kasper DL, Hauser SL, Jameson J, Loscalzo J, eds.
Harrisons Principles of Internal Medicine. 18th ed. New York, NY: McGraw-Hill; 2012.
2. Lin Liew EC, Sheehy AS, Wood KE, Coursin DB. Chapter 151: Adrenal insu ciency. In: McKean SC, Ross JJ, Dressler DD, Brotman
DJ, Ginsberg JS, eds. Principles and Practice of Hospital Medicine. New York, NY: McGraw-Hill; 2012.

11

12

ENDOCRINOLOGY
DIAGNOSTIC APPROACH

Amiodarone Thyrotoxicosis

ENDOCRINOLOGY

DIAGNOSTIC APPROACH

Amiodarone Thyrotoxicosis
Case: A 39-year-old emale with atrial f brillation presents to the ICU with
hyperthermia, HTN, and psychosis. She has been on amiodarone. Her TSH
is ound to be less than assay.
What are the possible e ects o amiodarone on thyroid unctioning?1

Amiodarone is an iodine source (39% iodine by weight), and the patients


response to iodine is determined by pre-existing state o thyroid
HYPERthyroid ef ect/thyrotoxicosis:
ype I amiodarone-induced thyrotoxicosis: JodBasedow ef ect
Amiodarone load induces thyroid synthesis in abnormal thyroid (e.g.,
Graves or goiter) or in iodine-de cient patient HYPERthyroid state
ype II amiodarone-induced thyrotoxicosis: Drug-induced lysosomal
destructive thyroiditis
Release o pre ormed hormone in patients without intrinsic thyroid
abnormalities
HYPOthyroidism (most common in United States): Wolf Chaikof ef ect
Iodine load inhibits thyroid peroxidase decreased thyroid release
HYPOthyroid
13% patients in the United States may have persistent HYPOthyroid state
More common in emales or autoimmune thyroiditis, and I-replete areas
How can you di erentiate Type I rom Type II amiodarone-induced

thyrotoxicosis?
Doppler can reveal high vascularity in ype I, low vascularity in ype II
(but this is limited by operator-dependency and has low sensitivity)
Iodine uptake scans are variable in ype 1 but low in ype II
In practice, can be hard to distinguish ypes I and II
How should amiodarone thyroiditis be managed?

Patients with hypothyroidism: treat with levothyroxine, continue amiodarone


ype I: Stop amiodarone i possible, administer thioamides (e.g., P U)
ype II: Stop amiodarone i possible, steroids may have modest bene t
Given di culty distinguishing between ype I and ype II, o en treat or
both empirically. T yroidectomy is an option in resistant cases.
How long a er discontinuation o amiodarone will e ects linger?
Amiodarone is stored in adipose tissue and can linger or > 6 months.

REFERENCE
1. Jameson J, Weetman AP. Chapter 341: Disorders o the thyroid gland. In: Longo DL, Fauci AS, Kasper DL, Hauser SL, Jameson J,
Loscalzo J, eds. Harrisons Principles of Internal Medicine. 18th ed. New York, NY: McGraw-Hill; 2012.

12

13

yt

va

yr

yr

ti

ti

va

yr

&

ENDOCRINOLOGY
CLASSIFICATION

Anterior Hypopituitarism

ENDOCRINOLOGY

CLASSIFICATION

Anterior Hypopituitarism
Case: A 54-year-old man with hemochromatosis presents with decreased
libido, erectile dys unction and cold intolerance.
What hormones are secreted by the anterior pituitary?1

Adrenocorticotropic hormone (AC H), growth hormone (GH), Prolactin


(PRL), thyroid-stimulating hormone ( SH), luteinizing hormone (LH), and
ollicle-stimulating hormone (FSH)
How does each hormone def ciency mani est itsel (in adults)?2

GH de ciency: muscle mass and sense o well-being


Hypogonadism (due to FSH/LH de ciency): amenorrhea in women;
libido + erectile dys unction in men
Hypothyroidism: cold intolerance, constipation, dry skin, energy
AC H de ciency: weakness, weight loss, anorexia, nausea/vomiting
Prolactin de ciency: ailure o postpartum lactation
When anterior pituitary unction is damaged, in what order are the

hormones lost?2
From rst to last: GH, LH/FSH, SH, AC H, PRL (although may vary in
practice)
What are the various causes o anterior hypopituitarism?2
Invasive: space-occupying lesion (e.g., large pituitary adenoma,
craniopharyngioma)
In arction: pituitary ischemic damage (e.g., Sheehans syndrome, pituitary
apoplexy)
In ltrative: in ltrative disease processes (e.g., sarcoidosis, hemochromatosis)
Injury: severe head trauma
Immunologic: lymphocytic hypophysitis
Iatrogenic: complication o pituitary surgery or radiation
In ectious: tuberculosis, syphilis, ungal in ection
Idiopathic
Which hormone def ciencies require treatment and with what medications?2
GH de ciency: role o HGH is controversial; typically do not treat
Hypogonadism: estrogen or premenopausal women; testosterone or men
Hypothyroidism: levothyroxine (titrate to 4 level, not SH)
AC H de ciency: hydrocortisone (no need or mineralocorticoid therapy)
Prolactin de ciency: no treatment (bottle- eeding or in ants)

REFERENCES
1. Fitzgerald PA. Chapter 26: Endocrine disorders. In: Papadakis MA, McPhee SJ, Rabow MW, eds. CURREN Medical Diagnosis &
reatment 2014. New York, NY: McGraw-Hill; 2014.
2. Javorsky BR, Aron DC, Findling JW, yrrell J. Chapter 4: Hypothalamus and pituitary gland. In: Gardner DG, Shoback D, eds.
Greenspans Basic & Clinical Endocrinology. 9th ed. New York, NY: McGraw-Hill; 2011.

13

ENDOCRINOLOGY

TREATMENT APPROACH

Diabetes MellitusType IIInpatient Management

Tota l Da ily Dos e (TDD) of Ins ulin


= 0.40.6 Units /kg

Ba s a l Ins ulin
= 0.5 TDD

P ra ndia l Ins ulin


= 0.5 TDD

1. Once da ily (e .g., gla rgine )


OR
2. S plit 2 /da ily (e .g., NP H)

S plit 3 /da ily


pre -me a ls
(e .g., a s pa rt)

Ad d c orre c tiona l-b a s e d ins ulin with a n ins ulin s lid ing s c a le with me a ls

Figure 2-4. Weight-based insulin regimen.

14

ENDOCRINOLOGY

TREATMENT APPROACH

Diabetes MellitusType IIInpatient Management


Case: A 48-year-old women with poorly controlled type II diabetes mellitus
on met ormin alone is admitted with hypoxia and pleuritic chest pain. A
PE-CT scan is ordered.
Should oral hypoglycemic agents be continued or stopped in the inpatient

setting?1
O en stopped due to the unpredictable nature o hospitalization:
Increased risk o lactic acidosis with met ormin i patient will receive IV
contrast or develop renal insu ciency during hospitalization
Increased risk o hypoglycemia with insulin secretagogues
(e.g., sul onylureas) i patient will be NPO or any time
Exception: short hospital stay, no expected IV contrast or NPO state
What are the concerns o starting an insulin sliding scale (ISS) alone?1
Reactive strategy: only responds to high glucose levels a er they are noted
Results in higher rates o hyperglycemia, more peaks and valleys to
glucose levels as opposed to steady control, risk o insulin-stacking
Exception: well-controlled diabetes on oral agents alone or diet-controlled
When should a weight-based insulin regimen be started as an inpatient?1
Start in any patient with poorly controlled diabetes, high doses o oral
hypoglycemics as outpatients and/or already on insulin
Improved glycemic control by anticipating and preventing hyperglycemia
How does one calculate a weight-based insulin regimen?

Step-wise approach 1
Discontinue all oral hypoglycemic agents
Calculate DD o insulin = 0.40.6 Units/kg
Divide o DD into basal and prandial insulin
Add correctional insulin sliding scale with meals
Daily adjustment: calculate DD rom prior day, change by 1020% up/down2
itrate to pre-meal glucose < 140 mg/dL and random glucose < 180 mg/dL
Adjust dosing to avoid glucose < 100 mg/dL ( hypoglycemia risk)
Is there data that supports the use o weight-based insulin versus ISS?3

T e RABBI 2 trial showed improved glycemic control (but no change in


length o stay or mortality) with weight-based insulin versus ISS strategy in
surgical patients

REFERENCES
1. Schnipper JL. Chapter 149: Inpatient management o diabetes and hyperglycemia. In: McKean SC, Ross JJ, Dressler DD, Brotman
DJ, Ginsberg JS, eds. Principles and Practice of Hospital Medicine. New York, NY: McGraw-Hill; 2012.
2. Moghissi ES, Korytkowski M , DiNardo M, et al; American Association o Clinical Endocrinologists; American Diabetes Association. American Association o Clinical Endocrinologists and American Diabetes Association consensus statement on inpatient
glycemic control. Diabetes Care. 2009;32(6):11191131.
3. Umpierrez GE, Smiley D, Jacobs S, et al. Randomized study o basal-bolus insulin therapy in the inpatient management o patients
with type 2 diabetes undergoing general surgery (RABBI 2 surgery). Diabetes Care. 2011;34(2):256261.

14

ENDOCRINOLOGY

PATHOPHYSIOLOGY

Glucocorticoid-Induced Osteoporosis

De cre a s e d bone
mine ra liza tion

Functiona l
hypogona dis m

De cre a s e d inte s tina l


ca lcium a bs orption
a nd incre a s e d re na l
e xcre tion of ca lcium

S uppre s s ion of
gona dotropin
production

Glucocorticoid
long-te rm the ra py
pre dnis one 7.5 mg/da y

Activa tion of
os te ocla s ts

Inhibition of
os te obla s ts

Incre a s e d bone
re s orption

De cre a s e d bone
forma tion

Figure 2-5. E ects o glucocorticoids on bone metabolism.

15

ENDOCRINOLOGY

PATHOPHYSIOLOGY

Glucocorticoid-Induced Osteoporosis
Case: A 66-year-old woman with newly diagnosed rheumatoid arthritis is
set to start prednisone 10 mg/day. She has not received any bone mineral
density (BMD) testing in the past.
At what dose o glucocorticoid does the risk o osteoporosis become a

concern?1
Glucocorticoid dose o 7.5 mg/day o prednisone or 3 months
Associated with increased risk o both vertebral and hip ractures
What are the mechanisms by which glucocorticoids lead to osteoporosis?1,2

Activation o osteoclastic activity increased bone resorption


Inhibition and induced apoptosis o osteoblasts decreased bone ormation
Decreased intestinal absorption o calcium (likely vitamin D-independent
ef ect) and increased urinary excretion o calcium decreased bone
mineralization
Suppression o gonadotropins unctionally hypogonadal state
When should BMD be obtained on a patient on long-term steroids?3

Prior to the start o long-term steroid therapy


DEXA o hip and spine is pre erred or BMD testing
I BMD is normal at the onset o therapy, scans should be repeated every
69 months while on steroids (shorter interval than or nonsteroid osteoporosis)
What measures should be taken to prevent the development o osteoporosis?3,4
Reduce steroid to minimum dose necessary
Ensure adequate calcium and vitamin D intake
Modi y risk actors (promote physical activity, smoking/EtOH cessation)
T e American College o Rheumatology (ACR) recommends prophylactic
bisphosphonate therapy i starting long-term steroids, especially i postmenopausal or personal history o a ragility racture
I glucocorticoid-induced osteoporosis is conf rmed with a T-score o
< 2.5, how would you approach treatment?5
Initiate bisphosphonate therapy (in addition to above preventative measures)
Increases BMD at both hip + spine, as well as reduces vertebral ractures
May alternatively consider nonbisphosphonate therapies (hormone replacement, teriparatide, denosumab) based on patient pre erence and comorbid
conditions

REFERENCES
1. Shoback D. Chapter 58: Osteoporosis & glucocorticoid-induced osteoporosis. In: Imboden JB, Hellmann DB, Stone JH, eds.
CURREN Rheumatology Diagnosis & reatment. 3rd ed. New York, NY: McGraw-Hill; 2013.
2. Lindsay R, Cosman F. Chapter 354: Osteoporosis. In: Longo DL, Fauci AS, Kasper DL, Hauser SL, Jameson J, Loscalzo J, eds.
Harrisons Principles of Internal Medicine. 18th ed. New York, NY: McGraw-Hill; 2012.
3. Shoback D, Sellmeyer D, Bikle DD. Chapter 8: Metabolic bone disease. In: Gardner DG, Shoback D, eds. Greenspans Basic & Clinical
Endocrinology. 9th ed. New York, NY: McGraw-Hill; 2011.
4. Grossman JM, Gordon R, Ranganath VK, et al. American College o Rheumatology 2010 recommendations or the prevention and
treatment o glucocorticoid-induced osteoporosis. Arthritis Care Res (Hoboken). 2010;62(11):15151526.
5. Fraser LA, Adachi JD. Glucocorticoid-induced osteoporosis: treatment update and review. T er Adv Musculoskelet Dis. 2009;1(2):7185.

15

DIAGNOSTIC APPROACH

ENDOCRINOLOGY

Hypercalcemia Part I
Hype rca lce mia (Confirme d)

Me a s ure P TH

Ele va te d or
ina ppropria te ly norma l P TH

Low P TH

Me a s ure 24-hour urina ry ca lcium

Me a s ure P THrp

Ele va te d
(>200 mg/24 hours )

P rima ry
hype rpa ra thyroidis m
(or s e c ond a ry/te rtia ry
hyp e rp a ra thyroid is m
if his tory of c hronic
kid ne y d is e a s e )

Low
(<200 mg/24 hours )

P THrp
e le va te d

P THrp
norma l

FHH,
lithium toxicity

Ma ligna ncy

Me a s ure
Vit D me ta bolite s

1,25(OH)2 D

e le va te d

25(OH)D
e le va te d

Vit D
me ta bolite s
norma l

Lymphoma ,
s a rcoid

Vit D
toxicity

Multiple
mye loma ,
hype rthyroidis m,
Vit A toxicity

Key
P TH P a ra thyroid hormone
P THrp P a ra thyroid hormone -re la te d prote in
FHH Fa milila l hypoca lciuric hype rca lce mia

16

Figure 2-6. Diagnostic algorithm or hypercalcemia.

ENDOCRINOLOGY

DIAGNOSTIC APPROACH

Hypercalcemia Part I
Case: A 74-year-old man with a history o squamous cell lung cancer presents
with con usion and is ound to have hypercalcemia.
A er conf rming an elevated total calcium with a repeat measurement, what

other lab test must be checked to ensure an accurate calcium level?


Serum albumin: Add 0.8 mg/dL to total calcium or every 1.0 g/dL o albumin
below normal (4.0 g/dL). Can alternatively measure an ionized calcium level.
With what lab measurement do you begin the workup o hypercalcemia?1
Parathyroid hormone (P H): should be suppressed in the setting o
hypercalcemia
I the PTH is elevated or inappropriately normal, what is the next

diagnostic step in the workup?1


Measure 24-hour urinary calcium
I elevated, then most likely diagnosis is hyperparathyroidism
Primary: majority o cases due to a single parathyroid adenoma
Secondary: phosphate and calcium levels in chronic kidney disease stimulate parathyroid glands to secrete P H
ertiary: persistent secondary hyperparathyroidism leads to autonomous secretion o P H by parathyroid glands
I low, then need to consider amilial hypocalciuric hypercalcemia
(FHH) or lithium toxicity (as well as co-existing vitamin D de ciency)
I the PTH is low, what is the next lab to send?1

P H-related protein (P Hrp)


Elevated P Hrp: screen or malignancy (e.g., squamous cell lung cancer)
I the PTHrp is normal, what is the next lab to send?
Vitamin D metabolites (1,25(OH)2D and 25(OH)D)
Elevated 1,25(OH)2D: screen or granulomatous diseases and lymphoma
Elevated 25(OH)D: consider Vitamin D toxicity
I the PTH, PTHrp + Vitamin D metabolites are all normal, what are the
remaining diagnoses to consider?2
Multiple myeloma, lytic skeletal metastases, hyperthyroidism, vitamin A
toxicity, milk-alkali syndrome, medications (e.g., thiazides), immobilization

REFERENCES
1. Khosla S. Chapter 46: Hypercalcemia and hypocalcemia. In: Longo DL, Fauci AS, Kasper DL, Hauser SL, Jameson J, Loscalzo J, eds.
Harrisons Principles of Internal Medicine. 18th ed. New York, NY: McGraw-Hill; 2012.
2. Stern SC, Ci u AS, Altkorn D. Symptom to Diagnosis: An Evidence-Based Guide. 2nd ed. New York, NY: McGraw-Hill; 2010.

16

17

cu

ca

>

co

ENDOCRINOLOGY
TREATMENT APPROACH

Hypercalcemia Part II

ENDOCRINOLOGY

TREATMENT APPROACH

Hypercalcemia Part II
Case: A 45-year-old man with lymphoma presents with con usion and
hypercalcemia.
What e ect does hypercalcemia have on intravascular volume and why?1

Intravascular volume contraction via two mechanisms


Polyuria (hypercalcemia impairs renal response to vasopressin)
Decreased thirst (hypercalcemia-induced altered mental status)
What are the f rst-line treatment and other therapeutic options or

hypercalcemia?
First-line treatment: volume restoration with intravenous normal saline
Aggressive volume repletion with saline bolus ollowed by in usion rate
at 200 cc/h or more (most patients need 34 L within rst 24 hours)
Other treatments: bisphosphonates, calcitonin, steroids, denosumab,
dialysis
Should loop diuretics be administered or hypercalcemia?2

Primarily reserved to prevent uid overload rom volume restoration


(especially i underlying heart ailure or renal insu ciency). No longer
routinely used or treatment o hypercalcemia.
How do bisphosphonates treat hypercalcemia, and what are the available
options?1
Mechanism: inhibit osteoclast-mediated bone resorption
Bisphosphonates approved or hypercalcemia: zoledronic acid (4 mg IV) and
pamidronate (6090 mg IV)
Onset o action 13 days with a peak ef ect 35 days later
What are the indications or calcitonin administration?1
Severe hypercalcemia (serum calcium >14 mg/dL) necessitating a rapid
decline (within hours) in serum calcium
Use to rapidly lower serum calcium while awaiting more sustained ef ect
o bisphosphonate therapy
E cacy limited to the rst 48 hours due to tachyphylaxis
By what mechanisms do glucocorticoids lower serum calcium levels?2
Inhibit ectopic production o 1,25(OH)2D
reat hypercalcemia rom granulomatous diseases or lymphoma
Reduce intestinal calcium absorption and Vitamin D ef ects
reat steroid-responsive malignancies (e.g., multiple myeloma)

REFERENCES
1. Shoback D, Sellmeyer D, Bikle DD. Chapter 8: Metabolic bone disease. In: Gardner DG, Shoback D, eds. Greenspans Basic & Clinical
Endocrinology. 9th ed. New York, NY: McGraw-Hill; 2011.
2. Holt EH, Bilezikian JP. Chapter 247: Calcium disorders. In: McKean SC, Ross JJ, Dressler DD, Brotman DJ, Ginsberg JS, eds.
Principles and Practice of Hospital Medicine. New York, NY: McGraw-Hill; 2012.

17

ENDOCRINOLOGY

DIAGNOSTIC APPROACH

Pheochromocytoma

Sensitivity
(Rule Out)

Specif city
(Rule In)

+++

+++

++++

++

+++

++++

+++

++

+++++

+++

CT

++++

+++

MRI

++++

+++

+++

++++

++

++

Diagnostic Method
24-hour urinary tests
Catecholamines
Fractionated metanephrines
Total metanephrines (highest speci city)
Plasma tests
Catecholamines
Free metanephrines (highest sensitivity)
Imaging

MIBG scintigraphy (highest speci city)


Somatostatin receptor scintigraphya
a

Best at detecting extra-adrenal tumors.

(Adapted rom Neumann HH. Chapter 343: Pheochromocytoma. In: Longo DL, Fauci AS, Kasper DL, Hauser
SL, Jameson J, Loscalzo J, eds. Harrisons Principles of Internal Medicine. 18th ed. New York, NY: McGraw-Hill;
2012: Table 343-2.)

Figure 2-8. Diagnostic methods or pheochromocytoma and test


characteristics.

18

ENDOCRINOLOGY

DIAGNOSTIC APPROACH

Pheochromocytoma
Case: A 34-year-old woman with newly diagnosed hypertension presents
with episodic headaches and palpitations. You are concerned about a
secondary cause o hypertension.
What signs/symptoms raise suspicion or a pheochromocytoma?

Classic triadepisodic headache, palpitations, and diaphoresis1


95% o patients with a pheochromocytoma have one o the symptoms o
the classic triad
When all symptoms o the classic triad are present with hypertension, the
speci city o diagnosis is high (93.8%)2
What types o tests are available to diagnose a pheochromocytoma?3,4
1. Urinary or plasma detection o catecholamine excess (pre erably via biochemical testing or the metabolites o catecholamines, a.k.a. metanephrines)
2. Imaging modalities to localize tumor (i catecholamine excess detected)
C with contrast (nonionic pre erred to risk o catecholamine release)
2-weighted MRI with gadolinium contrast (similar sensitivity to C but
better at detecting extra-adrenal tumors)
Nuclear studies (pursue i C /MRI negative but high clinical suspicion)
umor selectively uptakes radioactive tracer: MIBG (radioisotope that
resembles norepinephrine) in MIBG scintigraphy; or radiolabeled
octreotide in somatostatin receptor imaging
Which diagnostic test is most sensitive in the initial workup o pheochro-

mocytoma? Which test is most specif c?4


Most sensitive: plasma ractionated ree metanephrines
I normal, then pheochromocytoma ef ectively ruled out
Most speci c: urinary total metanephrines
I the plasma ree metanephrine test returns positive, what are the next

steps?
Con rm positive biochemical testing with urinary total metanephrines
I also positive, then localize tumor with C or MRI imaging
I inconclusive, consider nuclear testing with MIBG scintigraphy (high
speci city) or somatostatin receptor imaging (best at detecting extra-adrenal
tumors) to localize tumor

REFERENCES
1. Stern SC, Ci u AS, Altkorn D. Symptom to Diagnosis: An Evidence-Based Guide. 2nd ed. New York, NY: McGraw-Hill; 2010.
2. Westphal SA. Diagnosis o a pheochromocytoma. Am J Med Sci. 2005;329(1):1821.
3. Neumann HH. Chapter 343: Pheochromocytoma. In: Longo DL, Fauci AS, Kasper DL, Hauser SL, Jameson J, Loscalzo J, eds.
Harrisons Principles of Internal Medicine. 18th ed. New York, NY: McGraw-Hill; 2012.
4. Fitzgerald PA. Chapter 26: Endocrine disorders. In: Papadakis MA, McPhee SJ, Rabow MW, eds. CURREN Medical Diagnosis &
reatment. New York, NY: McGraw-Hill; 2014.

18

ENDOCRINOLOGY

DIAGNOSTIC APPROACH

Polyuria

Increase in Urine
Osmolality a ter
Water Deprivation

Increase in Urine
Osmolality a ter
Desmopressin

Normal

No change

Primary polydipsia

No change

Central diabetes Insipidus*

No change

Nephrogenic diabetes
Insipidus*

No change

No change

Mechanism o Polyuria

*May see minimal increase in urine osmolality after water deprivation or desmopressin if partial central or
partial nephrogenic DI.

Figure 2-9. E ect o water deprivation and desmopressin on urine


osmolality.

19

ENDOCRINOLOGY

DIAGNOSTIC APPROACH

Polyuria
Case: A 34-year-old woman on lithium or bipolar disorder presents with
excessive urination.
What is the def nition o polyuria and what must it be distinguished rom?1

Urine output > 3 L/day (vs. urinary requency without actual in urine
output)
What are the two di erent categories o polyuria based on urine osmolality?2
Polyuria with dilute urine (e.g., diabetes insipidus or DI) versus nondilute
urine (e.g., osmotic diuresis rom diabetes mellitus, intrinsic renal disease,
or diuretics)
What are the di erent mechanisms o polyuria with dilute urine?2
Primary ingestion o excess uid (i.e., primary polydipsia)
Abnormally decreased synthesis and secretion o vasopressin (i.e., central DI)
Decreased renal response to vasopressin (i.e., nephrogenic DI)
What test helps di erentiate the various mechanisms o polyuria?2

T e water deprivation test (patient is NPO and urine osmolality is


measured with each void until a plateau in osmolality is reached; then,
desmopressin is given). Needs close monitoring as severe hypovolemia and
hypernatremia may develop
Normal: urine becomes maximally concentrated with water deprivation
and no urther response to desmopressin
Primary polydipsia: urine becomes concentrated (but not maximally
due to washing out o urea rom inner medulla and chronic suppression
o aquaporin synthesis) and no urther response to desmopressin
Central DI (complete): urine concentration does not change with water
deprivation but osmolality signi cantly increases a er desmopressin
Nephrogenic DI (complete): urine concentration does not change with
water deprivation or a er desmopressin
What are the most common causes o central and nephrogenic DI?2

Central DI: head trauma/pituitary surgery, malignancy (craniopharyngioma,


metastatic lesions to or hematologic involvement o pituitary), granulomatous disease, vascular (Sheehans syndrome), in ectious (chronic meningitis)
Nephrogenic DI: medications (lithium, demeclocycline), metabolic (hypercalcemia, hypokalemia), vascular (sickle cell disease, ischemia), renal
obstruction, in ltration (amyloidosis), congenital

REFERENCES
1. Lin J, Denker BM. Chapter 44: Azotemia and urinary abnormalities. In: Longo DL, Fauci AS, Kasper DL, Hauser SL, Jameson J,
Loscalzo J, eds. Harrisons Principles of Internal Medicine. 18th ed. New York, NY: McGraw-Hill; 2012.
2. Robinson AG. Chapter 5: T e posterior pituitary (Neurohypophysis). In: Gardner DG, Shoback D, eds. Greenspans Basic & Clinical
Endocrinology. 9th ed. New York, NY: McGraw-Hill; 2011.

19

PATHOPHYSIOLOGY

ENDOCRINOLOGY

Thyroid FunctionTests

Test

Normal State

Hyperthyroid State

Pregnancy

Free T4

NORMAL

NORMAL

Total T4

NORMAL

THBR

NORMAL

Free T4 index

NORMAL

NORMAL

Norma l

Hype rthyroid S ta te

= TBG
= T4

Figure 2-10. Thyroid unction tests in various states.

20

P re gna ncy

ENDOCRINOLOGY

PATHOPHYSIOLOGY

Thyroid FunctionTests
Case: A 33-year-old emale is admitted to the ICU with thyrotoxicosis. Free
T4 is > assay, Total T4 is high, and thyroid hormone binding ratio (THBR)
is elevated.
How should T4 be interpreted di erently rom ree T4?1

Free 4: a measure o only the physiologically active ree hormone circulating


otal 4: a measure o thyroid hormone that is both ree and bound to thyroidbinding globulin ( BG). Changes in BG or binding a nity make total 4
change, though this is without pathologic consequence
What are the limitations o ree T4 measurements?
Most ree 4 assays are inaccurate in the setting o low serum albumin,
critically ill settings (or times o altered protein metabolism), and when total
4 is at extremes.
What happens when TBG is elevated in pregnancy?

More BG more 4 picked up by BG body responds by producing


more ree 4. Physiologically active 4 is unchanged even though total 4
is higher.
How does the THBR aid in interpretation?

HBR is a way to measure and correct or the presence o excess BG


Radiolabeled 3 is mixed with patients sample, binding BG in serum.
Remaining unbound radiolabeled 3 is absorbed by a resin; this is called
the 3 uptake. HBR is the normalized 3 uptake = patients 3 uptake/
normal 3 uptake
HBR is elevated in a hyperthyroid state, where the patient does not have
available BG to take up radiolabeled 3, which is thus taken up by resin
HBR can be low, or example, in pregnancy, where excess BG is made and
available to take up most o radiolabeled 3
HBR serves as a correcting actor or total 4. HBR x total 4 = 4 Index
Unlike ree 4, the 4 index can be used as measurement o thyroid unction in
critically ill settings or in states o protein ux, and ollowed during treatment
What drug classes are used in thyrotoxicosis management?
T ionamide (e.g., P U, methimazole) to block new hormone synthesis,
iodine solution (e.g., SSKI) to block release o hormone, steroids to reduce
4 to 3 conversion, and beta blocker to control adrenergic ef ects

REFERENCE
1. Gardner D, Shoback D. Greenspans Basic and Clinical Endocrinology. 9th ed. San Francisco: McGraw-Hill; 2011.

20

GASTROENTEROLOGY

EVIDENCE-BASED MEDICINE

Acute Pancreatitis

Rans o ns Crite ria 1

BIS AP S c o re 2

(1 point e a ch)

(1 point e a ch)

On Admis s io n:
WBC >16,000/L
Age >55 ye a rs
Glucos e >200 mg/dL
AS T >250 IU/L
LDH >350 IU/L
Within 48 Ho urs o f Admis s io n:
Hct de cre a s e >10%
BUN incre a s e >5 mg/dL
S e rum ca lcium <8 mg/dL
Arte ria l pO2 <60 mmHg
Ba s e de ficit >4 mEq/L
Fluid ne e ds >6 L
Ra ns on s core < 3 03% morta lity
Ra ns on s core = 35 1115% morta lity
Ra ns on s core 6 40% morta lity

B
I
S
A
P

UN >25 mg/dL

mpa ire d me nta l s ta tus

IRS

ge >60

le ura l e ffus ions

BIS AP s core < 2 00.5% morta lity


BIS AP s core = 2 2% morta lity
BIS AP s core 3 520% morta lity

Figure 3-1. Prognostic scores in acute pancreatitis: Ransons criteria


and BISAP score.

21

GASTROENTEROLOGY

EVIDENCE-BASED MEDICINE

Acute Pancreatitis
Case: A 36-year-old man with a history o signif cant alcohol abuse presents
with epigastric pain and nausea, with an elevated lipase on admission labs.
What are the major causes o acute pancreatitis?3

Gallstones (leading cause: 3060% o cases), alcohol (1530% o cases),


post-ERCP, hypertriglyceridemia, trauma, postoperative, or medications
How do you estimate mortality or a patient with acute pancreatitis?

Ransons criteria: one o the rst prognostic scores or acute pancreatitis,


which consists o 11 parameters rom admission and later in the hospitalization 1
Major limitation: requires data rom the rst 48 hours o hospitalization
(prognostic score not available on admission or triage purposes)
BISAP score: identi es ve independent variables o risk2
All data is derived rom the rst 24 hours o admission, with the most
extreme values recorded (1 point or each criteria met)
BISAP score o 3 predicted severe pancreatitis with a prognostic
accuracy similar to that o Ransons or APACHE-II scores4
T e APACHE II score is o en used as well but was originally designed or
critically ill patients, and not speci cally or acute pancreatitis.
How does predicted severity o acute pancreatitis in uence management?5

Patients with high mortality risk (Ransons score 3, BISAP score 3, or


APACHE II score 8) should be considered or ICU admission or
aggressive uid hydration and monitoring
Are prophylactic antibiotics help ul in the management o sterile necrosis o

severe acute pancreatitis?6


Not help ul: a multicenter RC o patients with severe necrotizing
pancreatitis compared prophylactic IV meropenem vs. placebo
No signi cant di erences in the prevention o pancreatic in ections (18%
vs. 12%, p = 0.401), mortality rate (20% vs. 18%, p = 0.799), or surgical
intervention (26% vs. 20%, p = 0.476)
May increase risk o ungal in ection (i.e., Candida species) as well

REFERENCES
1. Ranson JH, Ri ind KM, Roses DF, Fink SD, Eng K, Spencer FC. Prognostic signs and the role o operative management in acute
pancreatitis. Surg Gynecol Obstet. 1974;139(1):6981.
2. Wu BU, Johannes RS, Sun X, et al. T e early prediction o mortality in acute pancreatitis: a large population-based study. Gut.
2008;57:16981703.
3. Greenberger NJ, Conwell DL, Wu BU, Banks PA. Chapter 313. Acute and chronic pancreatitis. In: Longo DL, Fauci AS, Kasper DL,
Hauser SL, Jameson J, Loscalzo J, eds. Harrisons Principles o Internal Medicine. 18th ed. New York, NY: McGraw-Hill; 2012.
4. Papachristou GI, Muddana V, Yadav D, et al. Comparison o BISAP, Ransons, APACHE-II, and C SI scores in predicting organ
ailure, complications, and mortality in acute pancreatitis. Am J Gastroenterol. 2010;105(2):435441.
5. Forsmark CE, Baillie J; AGA Institute Clinical Practice and Economics Committee; AGA Institute Governing Board. AGA Institute
technical review on acute pancreatitis. Gastroenterology. 2007;132(5):20222044.
6. Dellinger EP, ellado JM, Soto NE, et al. Early antibiotic treatment or severe acute necrotizing pancreatitis: a randomized, doubleblind, placebo-controlled study. Ann Surg. 2007;245(5):674683.

21

GASTROENTEROLOGY

TREATMENT APPROACH

AlcoholicHepatitis

Alcoholic Hepatitis

Alcohol a bs tine nce


P re ve nt a nd tre a t
a lcohol withdra wa l
Nutritiona l s upport

Dis crimina nt function 32

Treatment options

P re fe rre d

P re dnis olone 32 mg
p.o. da ily for 4 we e ks,
the n ta pe r for 4 we e ks

Alte rna tive

Pe ntoxifylline 400 mg
p.o. TID for 4 we e ks

(Longo DL, Fauci AS, Kasper DL, et al. Harrison's Principles o Internal Medicine, 18E. New York, NY:
McGraw Hill; 2012.)

Figure 3-2. Treatment algorithm or alcoholic hepatitis.

22

GASTROENTEROLOGY

TREATMENT APPROACH

AlcoholicHepatitis
Case: A 46-year-old women with a history o alcohol abuse presents with
abdominal pain and jaundice a ter a recent alcohol binge.
What is the typical clinical presentation o alcoholic hepatitis?

Clinical eatures vary widely and may range rom:


Asymptomatic to ever, hepatomegaly, abdominal pain, jaundice
Laboratory abnormalities
Elevated AL , AS (27 above normal range, but rarely > 400 IU)
AS : AL ratio > 2
May also see hyperbilirubinemia, hypoalbuminemia, elevated PMNs
How do you grade severity o alcoholic hepatitis?
Discriminant unction (DF) = 4.6 (prolongation o prothrombin time
above control [seconds]) + serum bilirubin (mg/dL)1
Used in the landmark studies (see below) to identi y patients with severe
alcoholic hepatitis that would bene t rom treatment
How does the discriminant unction guide treatment in alcoholic hepatitis?1

Patients with a DF 32 have survival when treated with corticosteroids


or 28 days
Meta-analysis o 3 RC s o severe alcoholic hepatitis (DF 32) compared
prednisolone 32 mg PO daily 28 days to placebo
Survival rate 84.6% or steroid vs. 65.1% or placebo (p = 0.001)
Need to exclude active in ection prior to starting prednisolone therapy
Calculate day 7 Lille score (i > 0.45, will likely not respond to steroids)2
I glucocorticoids are contraindicated, the alternative treatment is
pentoxi ylline3 (particularly e ective at reducing risk o hepatorenal
syndrome)
In the recent S OPAH trial, pentoxi ylline notably did not improve
28-day survival compared to placebo (OR 1.07, p = 0.69). Prednisolone
had a nearly signi cant improvement in mortality (OR 0.72, p = 0.06)4
Why is prednisolone the avored glucocorticoid in alcoholic hepatitis

treatment?
Prednisolone is the active metabolite o prednisone
Prednisone requires activation in the liver (impaired in alcoholic hepatitis)

REFERENCES
1. Mathurin P, Mendenhall CL, Carithers RL Jr, et al. Corticosteroids improve short-term survival in patients with severe alcoholic
hepatitis (AH): individual data analysis o the last three randomized placebo controlled double blind trials o corticosteroids in
severe AH. J Hepatol. 2002;36(4):480487.
2. Louvet A, Naveau S, Abdelnour M, et al. T e Lille model: a new tool or therapeutic strategy in patients with severe alcoholic
hepatitis treated with steroids. Hepatology. 2007;45(6):13481354.
3. Akriviadis E, Botla R, Briggs W, Han S, Reynolds , Shakil O. Pentoxi ylline improves short-term survival in severe acute alcoholic
hepatitis: a double-blind, placebo-controlled trial. Gastroenterology. 2000;119(6):16371648.
4. T ursz MR, Richardson P, Allison M, et al. Prednisolone or pentoxi ylline or alcoholic hepatitis. NEJM. 2015;23;372(17):1619
1628.

22

DIAGNOSTIC APPROACH

GASTROENTEROLOGY

ChronicDiarrhea
CHRONIC DIARRHEA

CLINICAL CHARACTERIS TICS


Volume
Fre que ncy
Gre a s ine s s
As socia te d pa in
LABORATORY S TOOL EVALUATION
S tool culture
Fe ca l le ukocyte s
C.diff te s ting
Occult blood
Ova a nd pa ra s ite s
S uda n s ta in
S tool os mola lity

INFLAMMATORY

FATTY

WATERY

Fre que nt
S ma ll volume
Te ne s mus
+Fe ca l le ukocyte s
+Occult blood

Gre a s y
Bulky
Difficult to flus h
+S uda n s ta in

La rge volume

COLONOS COP Y
WITH BIOP S Y

ENDOS COP Y
WITH S MALL
BOWEL BIOP S Y
AND P ANCREATIC
EXOCRINE TES TING

FACTITIOUS

Abnorma l
s tool os mola lity
(<290 mOs m/kg
or >600 mOs m/kg)

CALCULATE S TOOL
OS MOLALITY GAP
(OS M ga p = 2902 (Na s tool + Ks tool))

OS M ga p >125 mOs m/kg


Volume with fa s ting

Infla mma tory


bowe l dis e a s e
Ma ligna ncy
Is che mia
Infe ction

Ma la bs orption
- P a ncrea tic
ins ufficie ncy
- Bile a cid
ins ufficie ncy
- Ce lia c dis e a s e
- Ba cte ria l ove rgrowth

OS M ga p <50 mOs m/kg


Volume uncha nge d
with fa s ting

Os motic Dia rrhe a


P oor dige s tion of
ca rbohydra te s
(la ctos e )
P oor a bs orption
of ions (la xa tive s ,
a nta cids )

S e cre tory Dia rrhe a


Ca rcinoid
Ga s trinoma
VIP oma

(Longo DL, Fauci AS, Kasper DL, et al. Harrison's Principles o Internal Medicine, 18E. New York, NY:
McGraw Hill; 2012.)

23 Figure 3-3. Diagnostic work up o chronic diarrhea.

GASTROENTEROLOGY

DIAGNOSTIC APPROACH

ChronicDiarrhea
Case: A 56-year-old male is admitted with dehydration ollowing 1 month
o diarrhea.
What is the de nition o chronic diarrhea and what are the major types?

Diarrhea (three or more loose stools per day) > 4 weeks


In ammatory, atty, watery, and actitious
What must be ruled out be ore working up chronic diarrhea?
Fecal incontinence or ecal impaction with over ow incontinence
Medication e ect: ca eine, met ormin, antibiotics, antacids, magnesium, etc.
What are the common clinical characteristics o the our major types o

diarrhea and what urther lab workup is indicated?1


In ammatory: requent, small volume, bloody. O en associated with
tenesmus, severe abdominal pain, evers. Obtain ecal leukocytes, stool
guaiac, stool culture, C. di testing, ecal ova and parasites.
Fatty: greasy, bulky, dif cult to ush and o en leaving a residue in the
bowl. O en associated with weight loss. Obtain Sudan stain to estimate
at in stool; celiac serologies.
Watery: watery osmotic diarrhea improves with asting, while watery
secretory diarrhea continues unabated while asting. Calculate stool
osmotic gap.
Factitious diarrhea: calculate stool osmolality; < 290 mOsm/kg = hypotonic
solution added; > 600 mOsm/kg = hypertonic solution added to stool
How is a stool osmotic gap calculated?

Osmotic gap = 2902 (Nastool + Kstool)


> 125 mOsm/kg = osmotic diarrhea. Indicates ingestion o osmotically
active agents pulling water into stool (e.g., poor digestion o carbohydrates
like sorbitol or lactose, or ingestion o ions in laxatives or antacids)
< 50 mOsm/kg = secretory diarrhea. Di erential includes peptide-secreting
tumors (e.g., carcinoid, gastrinoma, VIPoma), chronic in ection
What are the next steps to con rming each type o diarrhea?
In ammatory: colonoscopy with biopsy
Fatty: endoscopy with small bowel biopsy, pancreatic exocrine testing,
hydrogen breath test or small intestinal bacterial overgrowth
Watery/secretory: small/large bowel imaging, check or peptide-secreting
tumors
Watery/osmotic: lactose- ree dietary challenge
I work-up unrevealing, consider IBS

REFERENCE
1. Sweetser S. Evaluating the patient with diarrhea: a case-based approach. Mayo Clinic Proc. 2001;87:596602.

23

24

Toxin detection

Organism detection

ce

ce

GASTROENTEROLOGY
DIAGNOSTIC APPROACH

ClostridiumDif cile

GASTROENTEROLOGY

DIAGNOSTIC APPROACH

ClostridiumDif cile
Case: A 35-year-old hospitalized male develops multiple episodes o
diarrhea per day. His stool is sent or C. dif cile testing.
How does C. dif cile lead to diarrhea?

Disruption o normal colonic ora C. dif cile spores germinate and


produce toxins A and B cell apoptosis and increased vascular permeability
at the brush border pseudomembrane ormation and diarrhea.
Not all strains are toxin-producing, and toxin B is more virulent than toxin A.
How do you diagnose C. dif cile?1
T ere are two approaches:
Detecting the presence o the organism (ELISA or culture): does not
distinguish between toxin-producing and nontoxin-producing strains
Detecting the presence o the toxin produced only by toxigenic strains
o C. dif cile (ELISA, cytotoxic assay, PCR). T is approach may be more
likely to represent actual in ection
Due to cost/logistics, culture and cytotoxic assays are not used in practice
Which tests are ordered rst?2,3

Initial screening: labs o en start with the dual approach o screening


or toxins and presence o C. dif cile. T ese are low cost and have a ast
turnaround.
ELISA o toxins A + B (low sensitivity)
ELISA o glutamate dehydrogenase (high sensitivity but unable to
distinguish toxin production)
Con rmatory testing: I the ELISA testing shows discrepancy, con rmatory
testing can be per ormed with PCR testing or toxins (more expensive)
How should C. dif cile be treated?1

Nonsevere in ection (e.g., WBC < 15K, Cr < 1.5 upper limit o normal):
oral metronidazole 500 mg ID 10 days
Severe uncomplicated in ection (e.g., WBC > 15K, Cr > 1.5 upper limit o
normal): oral vancomycin 125 mg QID 10 days
Severe complicated in ection (hypotension, ileus, or megacolon): oral
vancomycin 500 mg QID + IV metronidazole 500 mg q8hrs + C scan and
surgical consult
First recurrence: repeat initial treatment; second recurrence: consider long
course o tapered vancomycin or daxomicin or 10 days
Re ractory: consider ecal microbiota transplant (FM )

REFERENCES
1. Cohen SH, Gerding DN, Johnson S, et al. Clinical practice guidelines or Clostridium dif cile in ection in adults: 2010 update by
the society or healthcare epidemiology o America (SHEA) and the in ectious diseases society o America (IDSA). In ect Control
Hosp Epidemiol. 2010; 31(5):431455.
2. Swindells J, Brenwald N, Reading N, Oppenheim B. Evaluation o diagnostic tests or Clostridium dif cile in ection. J Clin Microbiol. 2010;48(2):606608.
3. icehurst JR, Aird DZ, Dam LM, Borek AP, Hargrove J , Carroll KC, et al. E ective detection o toxigenic Clostridium dif cile by
a two-step algorithm including tests or antigen and cytotoxin. J Clin Microbiol. 2006;44(3):11451149.

24

25

&

&

cr

cr

>

GASTROENTEROLOGY
DIAGNOSTIC APPROACH

Helicobacter Pylori Testing

GASTROENTEROLOGY

DIAGNOSTIC APPROACH

Helicobacter Pylori Testing


Case: A 44-year-old emale immigrant rom Latin America reports epigastric pain worse a ter meals. You are considering a diagnosis o nonulcer
dyspepsia and want to test or H. pylori.
What tests are available or H. pylori and what do they test or?1

Noninvasive tests:
1. Serologic ELISA: tests or serum IgG to H. pylori
2. Urea breath test: tests or urease activity in the stomach rom H. pylori
Patient swallows pill containing urea (with radiolabeled carbon)
I the urease enzyme is present in the stomach, the urea will be converted
to CO2 and NH 3
Detection o radiolabeled CO2 upon exhalation = a positive test
3. Stool antigen test: tests or H. pylori antigen in the stool
Invasive tests (require endoscopic biopsy rom antrum o stomach):
1. Histology: identi es the H. pylori bacilli in gastric tissue
2. Rapid urease test: tests or urease activity in the stomach rom H. pylori
Biopsy placed in medium containing urea
I urease is present rom H. pylori, urea will be converted to CO2 + NH 3,
thereby raising the pH indicator in medium changes color
3. Culture: grows H. pylori on culture medium

Which noninvasive tests di erentiate active rom prior in ection?

Urea breath test or H. pylori stool antigen test


Both tests have high sensitivity/speci city and indicate active in ection
(choose based on availability)
Serologic testing does not distinguish between active and prior in ection
Which test should be used to con rm eradication af er H. pylori treatment?

Urea breath test (pre erred given sensitivity) or H. pylori stool antigen test
Serologic testing not help ul: IgG to H. pylori persists in the bloodstream
despite success ul treatment
When should a culture o H. pylori be attempted?
Due to expense and dif culty in growing H. pylori, culture should be
reserved or resistant cases o H. pylori in ection to determine antibiotic
susceptibilities

REFERENCES
1. McQuaid KR. Chapter 15. Gastrointestinal disorders. In: Papadakis MA, McPhee SJ, Rabow MW, eds. Current Medical Diagnosis &
Treatment. New York, NY: McGraw-Hill; 2014.
2. Blumberg RS, Burako R (eds). Current Diagnosis & Treatment: Gastroenterology, Hepatology, & Endoscopy. 2nd ed. New York, NY:
McGraw-Hill; 2012.

25

GASTROENTEROLOGY

PATHOPHYSIOLOGY

HepaticEncephalopathy

Ure a (from live r)


in GI tra ct

P rote in (from die t)


in GI tra ct

Colonic b a c te ria l c a ta b olis m

Cirrhos is
1. Inc re a s e d p ortos ys te mic
s hunting
2. Imp a ire d live r func tion

Incre a s e d NH3
H

La c tulos e
d e c re a s e s
c olonic p H

Conve rs ion to NH4+

Fe ca l e xcre tion of NH4+


(Lew E. Chapter 15. Peptic Ulcer Disease. In: Greenberger NJ, Blumberg RS, Burako R. eds. CURRENT Diag
nosis&Treatment:Gastroenterology,Hepatology,&Endoscopy,2e.NewYork,NY:McGraw-Hill;2012.)

Figure 3-6.Pathogenesisofhepaticencephalopathyandmechanism
o lactulose treatment.

26

GASTROENTEROLOGY

PATHOPHYSIOLOGY

HepaticEncephalopathy
Case: A 54-year-old man with a history o alcoholic cirrhosis presents with
3 days o con usion and is noted to have asterixis on exam.
What is hepatic encephalopathy (HE) and how does it present?

HE is a reversible syndrome o altered mental status due to the impaired


clearance o neurotoxins by the liver
Clinical presentation
Cognitive de cits: altered sleepwake cycle, impaired consciousness,
inattention, personality change
Impaired neuromuscular unction: asterixis, ataxia, rigidity
Why do patients with cirrhosis develop HE?1
1. Impaired liver unction: damaged hepatocytes are less ef cient at converting
neurotoxins into sa e orms that can be excreted
2. Portosystemic shunting: due to portal hypertension, blood ow is shunted
away rom the liver decreased clearance o neurotoxins
What is the classic neurotoxin in HE and where is it generated?1
Ammonia (NH 3): generated in the GI tract rom the breakdown o protein
and urea by colonic bacteria. Importantly, peripheral venous NH 3 levels do
not correlate well with HE and are not diagnostic.
How does lactulose reduce the absorption o ammonia into the

bloodstream?1
Lactulose is metabolized into atty acids by colonic bacteria, which lowers
the pH
T e more acidic colon converts NH 3 to NH 4+ , which is unabsorbable
NH 4+ is trapped in the colon and excreted in the stool (titrate dose to
23 bowel movements/day), thereby reducing plasma NH 3 levels
Other mechanisms: modi cation o colonic ora ( NH 3-producing
bacteria) and cathartic e ect ( gut transit time limits ability to absorb
NH 3).
Are there other treatments in addition to lactulose or HE?2

Oral minimally absorbed antibiotics (e.g., ri aximin): both or acute and


preventive treatment o HE; usually used in addition to lactulose
RC showed signi cant risk reduction in HE episodes over 6 months with
ri aximin vs. placebo (hazard ratio = 0.42, p < 0.001)

REFERENCES
1. Greenberger NJ. Chapter 44. Portal systemic encephalopathy & hepatic encephalopathy. In: Greenberger NJ, Blumberg RS, Burako
R, eds. Current Diagnosis & Treatment: Gastroenterology, Hepatology, & Endoscopy. 2nd ed. New York, NY: McGraw-Hill; 2012.
2. Bass NM, Mullen KD, Sanyal A, et al. Ri aximin treatment in hepatic encephalopathy. N Engl J Med. 2010;362(12):10711081.

26

GASTROENTEROLOGY

PATHOPHYSIOLOGY

Hepatorenal Syndrome

Cirrho s is

S ys te mic/s pla nchnic


va s odila tion

Compe ns a tory re s pons e s :


Incre a s e d ca rdia c output
Activa tion of RAS */s ympa the tic s ys te ms

Effe ctive circula ting volume re s tore d

Cirrho s is
pro g re s s io n

S e ve re s ys te mic/s pla nchnic va s odila tion

Additio nal trig g e rs :


GI ble e d
S po ntane o us bac te rial
pe rito nitis
Ove r-diure s is
Larg e -vo lume
parac e nte s is

De cre a s e d e ffe ctive circula ting volume

Compe ns a tory re s pons e s ins ufficie nt/ma la da ptive :


Ca rdia c output ca nnot incre a s e furthe r
RAS /s ympa the tic s ys te ms furthe r a ctiva te d

Re na l pe rfus ion s ignifica ntly impa ire d

He pato re nal s yndro me


*RAS : re nina ngiote ns in s ys te m

Figure 3-7.Pathophysiologyofhepatorenalsyndrome.
27

GASTROENTEROLOGY

PATHOPHYSIOLOGY

Hepatorenal Syndrome
Case: A 54-year-old man with alcoholic cirrhosis presents with acute
kidney injury with a creatinine o 4.0 mg/dL (elevated rom a baseline o
0.5 mg/dL).
When should hepatorenal syndrome (HRS) be suspected?1

Acute kidney ailure in patients with end-stage liver disease (cirrhosis +


ascites)
Especially i no proteinuria, hematuria, or abnormal ndings on renal U/S
What is the pathophysiology o HRS?1

1. Cirrhosis leads to systemic and splanchnic arterial vasodilation


2. Pooling o blood in the splanchnic vasculature reduced e ective
circulating volume decreased renal per usion
3. Compensatory response activates vasoconstriction/sodium-retaining
responses (e.g., reninangiotensin system) and augments cardiac output
4. Renal per usion is initially restored, but cirrhosis progression and additional
insults (e.g., GI bleeding, SBP, aggressive diuresis and/or paracentesis) leads
to worsening e ective circulating volume
5. Cardiac output cannot compensate suf ciently and vasoconstriction/
sodium-retaining responses are overactivated renal per usion is signi cantly decreased HRS
What condition must be excluded be ore diagnosing HRS?2

Prerenal ailure: exclude by withdrawing diuretics and volume expanding


with IV albumin or at least 2 days
I no improvement in serum creatinine (to 1.5 mg/dL) despite volume
expansion, then HRS is the probable diagnosis
What are the di erences between type I and type II HRS?1
ype I: rapid and progressive renal impairment (doubling o initial creatinine
to 2.5 mg/dL or a 50% decline in GFR to < 20 mL/min within 2 weeks)
ype II: slower, more stable course (increase o initial creatinine to
1.5 mg/dL over weeks to months)
What are the principles o treatment or HRS?
Reverse systemic/splanchnic vasodilation with vasopressor therapy: midodrine
(systemic vasoconstrictor) and octreotide (inhibitor o splanchnic vasodilators)

REFERENCES
1. Wong F. Chapter 10. Hepatorenal syndrome. In: Lerma EV, Berns JS, Nissenson AR, eds. CURRENT Diagnosis & Treatment:
Nephrology & Hypertension. New York, NY: McGraw-Hill; 2009.
2. Greenberger NJ. Chapter 46. Hepatorenal syndrome. In: Greenberger NJ, Blumberg RS, Burako R, eds. Current Diagnosis &
Treatment: Gastroenterology, Hepatology, & Endoscopy. 2nd ed. New York, NY: McGraw-Hill; 2012.

27

/m

/m

ve

yo

<

yn

28

yn

GASTROENTEROLOGY
DIAGNOSTIC APPROACH

Lower GIBleed

GASTROENTEROLOGY

DIAGNOSTIC APPROACH

Lower GIBleed
Case: A 72-year-old man with a history o diverticulosis presents a ter a
large, bloody stool.
What are the most common sources o a lower GI bleed?1

Diverticulosis (1744%), AVM (230%), ischemic colitis (921%), malignancy (414%), hemorrhoids (411%), and post-polypectomy (6%)
What are the di erent modalities or diagnosing an acute lower GI bleed?2
Flexible sigmoidoscopy: endoscopic examination o the distal 60 cm o the
colon
Colonoscopy: endoscopic examination o the entire colon
Nuclear/tagged red blood cell (RBC) scan: radiotracer is injected into the
blood and images are captured to detect extravasation into the GI lumen
C angiography: increased resolution o C scan to detect extravasated
contrast in GI lumen (although less sensitive than a tagged RBC scan)
IR angiography: contrast dye is injected directly into the mesenteric arteries
and extravasation is noted on uoroscopy
Capsule endoscopy: pill capsule with a wireless camera is swallowed and
images o entire small bowel are conveyed to an external receiver
What is the diagnostic modality o choice in a hemodynamically stable

lower GI bleed?2
Colonoscopy (consider exible sigmoidoscopy i < 40 years old and no
history o iron de ciency or amily history o colon cancer)
I hemodynamically unstable, proceed directly to IR angiography or
upper endoscopy (i concern or upper GI bleeding source)
What are the pros/cons o obtaining a nuclear/tagged RBC scan prior to

angiography?
I positive, the scan selects bleeds that are more likely to be seen on angiography and allows or targeted angiography to site o bleeding ( contrast load)
However, in the time needed to obtain a localizing scan, the bleeding may
stop and result in a negative angiography (consider direct to angiography
i unstable)
Does early colonoscopy improve outcomes in patients with lower GI bleeds?3
RC comparing urgent ( 12 hours) to elective (35 days a er presentation)
colonoscopy or signi cant lower GI bleed
No di erence in rebleeding, length o hospital stay, or need or trans usion
(although increased detection o source o bleeding)

REFERENCES
1. Lee LS. Chapter 160. Acute lower gastrointestinal bleeding. In: McKean SC, Ross JJ, Dressler DD, Brotman DJ, Ginsberg JS, eds.
Principles and Practice o Hospital Medicine. New York, NY: McGraw-Hill; 2012.
2. Laine L. Chapter 41. Gastrointestinal bleeding. In: Longo DL, Fauci AS, Kasper DL, Hauser SL, Jameson J, Loscalzo J, eds. Harrisons
Principles o Internal Medicine. 18th ed. New York, NY: McGraw-Hill; 2012.
3. Laine L, Shah A. Randomized trial o urgent vs. elective colonoscopy in patients hospitalized with lower GI bleeding. Am J Gastroenterol. 2010;105(12):26362641.

28

GASTROENTEROLOGY

DIAGNOSTIC APPROACH

Spontaneous Bacterial Peritonitis

Ascitic Fluid Test

Rule -In SBP


(likelihood ratio, 95% CI)

Rule -Out SBP


(likelihood ratio, 95%CI)

Neutrophil (PMN)
count

> 250 cells/L


(6.4,4.68.8)

250 cells/L
(0.2,0.110.37)

Leukocytecount

> 1,000 cells/L


(9.1,5.515.1)

N/A

pH

< 7.35
(9.0,2.040.6)

N/A

Blood ascitic f uid


pH gradient

0.10
(11.3,4.329.9)

< 0.10
(0.12,0.020.77)

(DatafromWongCL,Holroyd-LeducJ,ThorpeKE,StrausSE.Doesthispatienthavebacterialperitonitis
orportalhypertension?HowdoIperformaparacentesisandanalyzetheresults?JAMA. 2008;299(10):
11661178.)

Figure 3-9.Ascitic uidanalysisandtestcharacteristics.

29

GASTROENTEROLOGY

DIAGNOSTIC APPROACH

Spontaneous Bacterial Peritonitis


Case: A 56-year-old woman with cirrhosis and ascites presents with ever
and abdominal pain. You are concerned about spontaneous bacterial
peritonitis (SBP).
Why is SBP more commonly seen in patients with cirrhosis and ascites?1

Cirrhosis impaired hepatic reticuloendothelial system (RES)


decreased clearance o microorganisms by hepatic RES
Ascites altered intestinal wall permeability increased translocation o
gut microorganisms into ascitic uid
How do you diagnose SBP on ascitic uid?2
Neutrophil (PMN) count o > 250 cells/L (classic de nition)
Leukocyte (WBC) count o > 1,000 cells/L
pH < 7.35 or blood-ascitic uid pH gradient 0.10
What ascitic uid test result is most help ul to rule-in SBP, and what test

result is most help ul to rule-out SBP?2


Rule-in: blood-ascitic uid pH gradient 0.10 (likelihood ratio o 11.3)
Rule-out: blood-ascitic uid pH gradient < 0.10 (likelihood ratio o 0.12)
What is secondary bacterial peritonitis and how does ascitic uid analysis

di erentiate it rom SBP?3


De nition: in ection o ascitic uid due to an intra-abdominal in ection
(compare to no identi able intra-abdominal in ection in SBP)
Distinguishing eatures on ascitic uid analysis
PMN count: typically higher levels than seen in SBP, and especially
suspicious i > 10,000 cells/mL
Elevated total protein (> 1 g/dL), elevated LDH (> serum level),
decreased glucose (< 50 mg/dL)
What is the empiric treatment or SBP?4
Antibiotics: third generation cephalosporin (e.g., ce otaxime)
IV albumin: 1.5 g/kg at diagnosis and 1.0 g/kg at day 3
RC comparing ce otaxime + albumin vs. ce otaxime alone or SBP
showed 3-month mortality bene t (22% vs. 41%, p = 0.03) and decreased
renal impairment (10% vs. 33%, p = 0.002) in the albumin + ce otaxime
group4

REFERENCES
1. Greenberger NJ. Chapter 45. Ascites & spontaneous bacterial peritonitis. In: Greenberger NJ, Blumberg RS, Burako R, eds. Current
Diagnosis & Treatment: Gastroenterology, Hepatology, & Endoscopy. 2nd eds. New York: McGraw-Hill; 2012.
2. Wong CL, Holroyd-Leduc J, T orpe KE, Straus SE. Does this patient have bacterial peritonitis or portal hypertension? How do I
per orm a paracentesis and analyze the results? JAMA. 2008;299(10):11661178.
3. McQuaid KR. Chapter 15. Gastrointestinal disorders. In: Papadakis MA, McPhee SJ, Rabow MW, eds. Current Medical Diagnosis &
Treatment. New York, NY: McGraw-Hill; 2014.
4. Sort P, Navasa M, Arroyo V, Aldeguer X, Planas R, Ruiz-del-Arbol L, et al. E ect o intravenous albumin on renal impairment and
mortality in patients with cirrhosis and spontaneous bacterial peritonitis. N Engl J Med. 1999;341(6):403439.

29

GASTROENTEROLOGY

EVIDENCE-BASED MEDICINE

Upper GIBleeding

Restrictive strategy
(trans use or
hemoglobin
< 7 mg/dL) (%)

Liberal strategy
(trans use or
hemoglobin
< 9 mg/dL) (%)

P-value

Probabilityof
survivalat
6 weeks

95

91

0.02

Rebleeding rate

10

16

0.01

Need or rescue
therapy(withbal
loon tamponade
or TIPS procedure)

0.04

Complication rate

40

48

0.02

Outcomes

(DatafromVillanuevaC,ColomoA,BoschA,etal.Transfusionstrategiesforacuteuppergastrointestinal
bleeding. N Engl J Med.2013;368(1):1121.)

Figure 3-10.Outcomesofrestrictiveversusliberaltransfusionstrategies
in acute upper GI bleeding.

30

GASTROENTEROLOGY

EVIDENCE-BASED MEDICINE

Upper GIBleeding
Case: A 54-year-old woman with a history o peptic ulcer disease seeks
medical attention a ter passing black, tarry stool or the past 2 days.
What are the possible sources o an upper GI bleed?

Esophagus: esophagitis, esophageal varices, malignancy


Gastroesophageal junction: MalloryWeiss tears
Stomach: ulcer, gastritis/gastropathy, Dieula oys lesion, gastric varices,
malignancy
Duodenum: ulcer, duodenitis, malignancy
Note: vascular lesions such as AV-mal ormations may occur at any site
What are the cornerstones o treatment or an upper GI bleed?1
Resuscitation with IV uids (and blood trans usions i appropriatesee
below)
Proton pump inhibitors (IV administration as standard o care)
Octreotide (50 g IV 1; then 2550 g/hr) i concern or variceal bleed
Upper GI endoscopy (within 24 hr is pre erred)
Other measures: NPO status, large-bore IVs or resuscitation, serial
hemoglobin checks, and consider holding vs. reversing anti-coagulation
depending on bene trisk ratio
Is there any evidence to help guide trans usion thresholds in upper GI

bleeds?2
RC o patients presenting with upper GI bleed compared a restrictive
strategy (trans use or hemoglobin < 7 mg/dL) vs. liberal strategy
(trans use or hemoglobin < 9 mg/dL)
Signi cant improvement in outcomes or restrictive strategy (improved
mortality, rebleeding rates, need or rescue therapy, and complications)
O note, the ollowing patients were excluded rom the study i :
Massive exsanguinating bleeding, recent acute coronary syndrome or
IA/CVA, or low risk o rebleeding
Why might a liberal strategy o trans usions lead to higher rates o rebleeding?2

Blood trans usion (and restoration o volume status) may reverse the
compensatory vasoconstriction o the splanchnic vascular bed
Increased splanchnic blood ow increased bleeding
Restoration o blood volume increases portal pressure
Increased bleeding rom portal hypertensive-related sites
Blood trans usion may impair coagulation and worsen hemostasis

REFERENCES
1. Overton D . Chapter 78. Upper gastrointestinal bleeding. In: intinalli JE, Stapczynski J, Ma O, Cline DM, Cydulka RK, Meckler
GD, eds. Tintinallis Emergency Medicine: A Comprehensive Study Guide. 7th eds. New York, NY: McGraw-Hill; 2011.
2. Villanueva C, Colomo A, Bosch A, et al. rans usion strategies or acute upper gastrointestinal bleeding. N Engl J Med. 2013;
368(1):1121.

30

31

>

HEMATOLOGY-ONCOLOGY
TREATMENT APPROACH

Acute Chest Syndrome in Sickle Cell Disease

HEMATOLOGY-ONCOLOGY

TREATMENT APPROACH

Acute Chest Syndrome in Sickle Cell Disease


Case: A 34 year old man with sickle cell disease is admitted with chest pain.
CXR shows RLL consolidation. You suspect acute chest syndrome (ACS).
What is ACS?

ACS is a nonspeci c clinical syndrome with many possible underlying causes.


De ned by (1) a new consolidation involving 1 complete lung segment and
(2) any one o chest pain, temperature 38.5C, tachypnea, wheezing, accessory
muscle use, drop in O2 sat o > 2% on room air, PaO2< 60 mmHg, or cough.1
ACS is clinically very di cult to distinguish rom pneumonia and may also
appear like pulmonary embolism.
What are possible causes o ACS?
Causes include at emboli, in ection, and in arction due to sickling
Fat emboli make up a large proportion o cases (as high as 4477% o ACS)2
Appears related to in arction o bone marrow in sickle cell crisis with
subsequent sloughing o at droplets rom marrow into venous circulation
In ections in decreasing order o requency: Chlamydia, Mycoplasma,
respiratory syncytial virus, Staphylococcus aureus, Streptococcus
pneumoniae, and parvovirus1,3
What testing should be ordered in the diagnostic evaluation o ACS?
EKG, CBC with di erential, reticulocyte count (to be evaluated or aplastic
crisis), CXR, blood and sputum cultures. C chest and cardiac biomarkers
may be use ul as well.
Bronchoscopy with bronchoalveolar lavage (BAL) may be per ormed in
atypical cases ( at droplets on BAL are diagnostic o at emboli).
How should patients with suspected ACS be managed?3

Supportive care with pain control and oxygen i hypoxemic


Fluids: 1.5 daily requirements; watch or development o pulmonary edema
Antibiotics: In ection can rarely be ruled out; third generation cephalosporin (to cover S. pneumoniae) plus macrolide or quinolone (to cover
atypicals) are rst line
Blood trans usion: only or moderate/severe episodes and when hemoglobin < 5 g/dL; should not raise hemoglobin to > 10 g/dL. I hemoglobin
> 11 g/dL, consider exchange trans usion instead4
Limited data at this point: steroids, nitric oxide, bronchodilators

REFERENCES
1. Vichinsky EP, Neumayr LD, Earles AN, et al. Causes and outcomes o the acute chest syndrome in sickle cell disease. National Acute
Chest Syndrome Study Group. N Engl J Med. 2000;342(25):18551865.
2. Zaidi Y, Sivakumaran M, Graham C, Hutchinson RM. Fatal bone marrow embolism in a patient with sickle cell beta + thalassaemia.
J Clin Pathol. 1996;49(9):774775.
3. Kato GJ, Gladwin M . Chapter 108. Sickle cell disease. In: Hall JB, Schmidt GA, Wood LH, eds. Principles of Critical Care. 3rd ed.
New York, NY: McGraw-Hill; 2005.
4. Chou S . rans usion therapy or sickle cell disease: a balancing act. Hematology Am Soc Hematol Educ Program. 2013;2013:
439446.

31

PATHOPHYSIOLOGY

HEMATOLOGY-ONCOLOGY

B12 Defciency

1. Ina de qua te
die ta ry inta ke

Food coba la min (Cbl)

2. Abs e nce
of ga s tric a cid

Cbl tra ns fe rre d to intrins ic fa ctor


produce d by ga s tric pa rie ta l ce lls

Ga s tric
a cid

3. Abs e nce of
intrins ic fa ctor

Live r

IF-Cbl comple x binds to re ce ptor


in te rmina l ile um (cubulin)

4. Dys function
of te rmina l ile um

(Bunn H, Heeney M. Megaloblastic Anemias. In: Bunn H, Aster JC. eds. Pathophysiology o Blood Disorders.
New York, NY: McGraw-Hill; 2011.)

Figure 4 2. B12 metabolism (in purple) and mechanisms o B12


def ciency (in red).

32

HEMATOLOGY-ONCOLOGY

PATHOPHYSIOLOGY

B12 Defciency
Case: A 42 year old woman with vitiligo presents with atigue and a
macrocytic anemia.
What is the di erential diagnosis or macrocytosis?1

B12 or olate de ciency, reticulocytosis, liver disease, hypothyroidism,


alcohol abuse, myelodysplastic syndrome, or drug-induced (e.g., azathioprine,
methotrexate)
What are the various mechanisms o B12 de ciency?2

1. Inadequate dietary intake o animal products


Strict vegan diet
2. Absence o gastric acid (needed to cleave B12 rom other proteins)
otal gastrectomy, achlorhydria, or chronic PPI use
3. Absence o intrinsic actors (transport protein that allows or B12 absorption
in the distal ileum)
Pernicious anemia: autoimmune disorder that attacks the gastric parietal
cells (source o intrinsic actor and gastric acid)
4. Dys unction o terminal ileum (site o B12 absorption)
Surgical resection o terminal ileum
Crohns disease involving the terminal ileum
Parasite competition
Fish tapeworm (Diphyllobothrium latum) or bacterial overgrowth
How do you diagnosis B12 de ciency?3

Check serum B12 level


I < 100 pg/mL B12 de ciency con rmed
I 100300 pg/mL check serum MMA and homocysteine levels
I both levels normal B12 de ciency unlikely
I MMA or both levels elevated B12 de ciency likely
I only homocysteine level elevated olate de ciency likely
I > 300 pg/mL B12 de ciency unlikely
What electrolyte abnormality are patients with B12 de ciency at risk or
developing during B12 repletion?2
Hypokalemia: rapid production and maturation o hematopoetic cells
incorporation o potassium into cells decreased serum potassium
levels

REFERENCES
1. Verhovsek M, McFarlane A. Chapter 173. Abnormalities in red blood cells. In: McKean SC, Ross JJ, Dressler DD, Brotman DJ,
Ginsberg JS, eds. Principles and Practice of Hospital Medicine. New York, NY: McGraw-Hill; 2012.
2. Bunn H, Heeney MM. Chapter 6. Megaloblastic anemias. In: Bunn H, Aster JC, eds. Pathophysiology of Blood Disorders. New
York,NY: McGraw-Hill; 2011.
3. Stern SC, Ci u AS, Altkorn D. Chapter 6. I Have a patient with anemia. how do I determine the cause?. In: Stern SC, Ci u AS,
Altkorn D, eds. Symptom to Diagnosis: An Evidence-Based Guide. 2nd ed. New York, NY: McGraw-Hill; 2010.

32

33

kl

y)

yt

yt

yt

va

xt

y)

ke

kl

vi

ve

va

yt

ve

ym

ve

ve

ve

HEMATOLOGY-ONCOLOGY
CLASSIFICATION

HemolyticAnemia

HEMATOLOGY-ONCOLOGY

CLASSIFICATION

HemolyticAnemia
Case: A 32 year old man presents with atigue, jaundice, darkened urine,
and signif cant anemia.
What is the clinical presentation o hemolytic anemia?1

Signs/symptoms: jaundice, darkened urine, and/or enlargement o spleen/


liver
Laboratory results:
Hemolysis: increased indirect bilirubin, LDH, urobilinogen; reduced
haptoglobin
Bone marrow response: increased reticulocytes (and thereby increased
MCV), although may not be present i ine ective bone marrow response
What lab ndings suggest intravascular hemolysis?
Evidence o ree hemoglobin in plasma (hemoglobinemia)
Free hemoglobin is then ltered by the kidney hemoglobinuria or
hemosiderinuria
What are the intravascular vs. extravascular causes o HA?1,2

Intravascular hemolysis
Paroxysmal nocturnal hemoglobinuria, severe in ection (clostridial
sepsis or severe malaria), mechanical destruction (microangiopathic or
prosthetic valves), trans usion reaction (ABO incompatibility)
Extravascular hemolysis
Hemoglobinopathies (sickle cell), enzymopathies (G6PD or pyruvate
kinase de ciencies), membrane-cytoskeletal de ects (hereditary
spherocytosis/elliptocytosis), toxic agents or drugs (copper, lead,
methyldopa, rarely penicillin), in ection (malaria or babesiosis),
autoimmune hemolytic anemia, hypersplenism
What is the di erence between the indirect and direct Coombs test?3

Direct: antibodies against IgG or C3 added to patients RBCs


I RBCs coated with IgG or complement RBCs agglutinate
(positive in autoimmune hemolytic anemia or post-trans usion reaction)
Indirect: patients serum added to sample o RBCs with known RBC antigens
Antibodies against IgG added; i patients serum has antibodies to one o
known RBC antigens RBCs agglutinate (positive i incompatible blood
during cross-match testing)

REFERENCES
1. Luzzatto L. Chapter 106. Hemolytic anemias and anemia due to acute blood loss. In: Longo DL, Fauci AS, Kasper DL, Hauser SL,
Jameson J, Loscalzo J, eds. Harrisons Principles of Internal Medicine. 18th ed. New York, NY: McGraw-Hill; 2012.
2. Chapter 12. Hematopathology. In: Kemp WL, Burns DK, Brown G, eds. Pathology: T e Big Picture. New York, NY: McGraw-Hill;
2008.
3. Bunn H. Chapter 11. Acquired hemolytic anemias. In: Bunn H, Aster JC, eds. Pathophysiology of Blood Disorders. New York, NY:
McGraw-Hill; 2011.

33

34

ce

>

<

<

HEMATOLOGY-ONCOLOGY
DIAGNOSTIC APPROACH

Heparin-InducedThrombocytopenia (HIT)

HEMATOLOGY-ONCOLOGY

DIAGNOSTIC APPROACH

Heparin-InducedThrombocytopenia (HIT)
Case: A 63 year old man with coronary artery disease undergoes a CABG
and is noted to have new onset thrombocytopenia and a swollen right cal
5 days a ter initiating heparin therapy.
What is the de nition o HIT?1

A all in the platelet count to 150,000/L or a all o > 50% while on heparin
What is the di erence between type I and type II HIT?2
ype I: benign and nonimmunologic response to heparin
Mild thrombocytopenia due to platelet clumping ( rst 4872 hours o
heparin)
ype II: clinically relevant and immune-mediated response to heparin
risk o thrombosis due to platelet activation:
Heparin binds PF4 antibodies orm against heparinPF4 complex
immune complex (heparin-PF4-antibody) activates platelets
iming (classic): 510 days a er initiation o heparin therapy
What are the our Ts and how do they assess risk or type II HIT?1

T rombocytopenia: risk i plt all > 50% or plt count 20100 k/L
iming o platelet all: risk i occurred within 510 days o heparin therapy
T rombosis: risk i major vessel thrombus developed (arterial or venous)
T rombocytopenia rom other causes: risk i no other alternative cause
A total score o 68 imparts high risk or HI ; 45 intermediate risk,
03 low risk2
How does the our Ts score help guide management?

Low risk do not per orm diagnostic testing or start empiric treatment (HI
e ectively ruled out due to high negative predictive value o low-risk score)3
Intermediate or high risk send diagnostic testing and treat or presumed
HI :
Discontinue heparin + start a non-heparin anticoagulant (e.g., bivalirudin)
What lab tests are available to diagnose type II HIT?
Antigen assay: tests or IgG antibodies to PF4heparin complex
Initial screening test, but alse positives can occur i cross-reactive
antibodies; interpret with caution i low pretest probability o HI
Activation assay: tests or serotonin release (indication o platelet activation)
a er mixing patients platelets and serum with heparin
Con rmatory test: sensitivity/speci city (but cost + processing time)

REFERENCES
1. Sadler J, Poncz M. Chapter 133. Antibody-mediated thrombotic disorders: thrombotic thrombocytopenic purpura and heparin-induced thrombocytopenia. In: Lichtman MA, Kipps J, Seligsohn U, Kaushansky K, Prchal J , eds. Williams Hematology. 8th ed.
New York, NY: McGraw-Hill; 2010.
2. Ahmed I, Majeed A, Powell R. Heparin induced thrombocytopenia: diagnosis and management update. Postgrad Med J. 2007;
83(983):575582.
3. Cuker A, Gimotty PA, Crowther MA, Warkentin E. Predictive value o the 4 s scoring system or heparin-induced thrombocytopenia: a systematic review and meta-analysis. Blood. 2012;120(20):41604167.

34

CLASSIFICATION

HEMATOLOGY-ONCOLOGY

Monoclonal Gammopathyo Unknown Signifcance (MGUS)


versus Multiple Myeloma

Plasma Cell
Disorder

a
b

M spike

Bone Marrow Presence o


Involvement Symptoms

Management

MGUS

< 3 g/dL < 10% plasma


cells

No

Observe

Smoldering
multiple
myeloma a

3 g/dL 10% plasma


cells

No

Observe

Multiple
myeloma b

3 g/dL 10% plasma


cells

Yes

Treat

Need to meet both criteria or M-spike and/or bone marrow involvement and asymptomatic
Need to meet all three criteria: M-spike, bone marrow involvement and symptomatic

C hype rCa lce mia


R Re na l ins ufficie ncy
A Ane mia
B Bone dis e a s e
Figure 4 5. Criteria or diagnosing MGUS vs. smoldering multiple
myeloma vs. multiple myeloma (above); symptoms rom end-organ
damage in multiple myeloma (below).

35

HEMATOLOGY-ONCOLOGY

CLASSIFICATION

Monoclonal Gammopathyo Unknown Signifcance


(MGUS) versus Multiple Myeloma
Case: A 66 year old man presents with bone pain and has a positive
M spike on serum protein electrophoresis (SPEP) testing.
What does a positive M-spike on SPEP indicate?1

T e presence o a monoclonal protein (immunoglobulin)


Majority o cases are IgG (70%), IgM (1520%), or IgA (10%)
What are the most common plasma cell disorders?2
Monoclonal gammopathy o unknown signi cance (MGUS; occurs in
3% o individuals > 50 years old) and multiple myeloma (smoldering or
symptomatic)
How do you diagnose MGUS vs. smoldering multiple myeloma vs. multiple

myeloma?3
M-spike
3 g/dL in MGUS; 3 g/dL in multiple myeloma (both orms)
Bone marrow involvement
< 10% plasma cells in MGUS; 10% in multiple myeloma (both orms)
Symptoms ( rom end-organ damage)
Absent in MGUS and smoldering multiple myeloma; present in multiple
myeloma (CRAB mnemonic or symptoms o end-organ damage)
C: HyperCalcemia (due to progressive bone destruction)
R: Renal insu ciency (majority o cases due to Bence-Jones protein casts
in distal tubule, but also due to calcium, uric acid, and dehydration)
A: Anemia (due to erythropoietin levels, RBC li espan, impaired
iron utilization)
B: Bone disease (due to osteoclastic activity + plasma cell in ltration
o bone)
How does management vary between MGUS and the two types o MM?2

MGUS: no treatment, but close ollow-up given risk o progression to other


plasma cell disorders (1% annual risk to progression to MM)
Smoldering MM: observe until evidence o progression, then treat
MM: treat (induction chemotherapy ollowed by autologous stem-cell
transplant is the current standard o care)

REFERENCES
1. Lichtman MA. Chapter 108. Essential monoclonal gammopathy. In: Lichtman MA, Kipps J, Seligsohn U, Kaushansky K, Prchal J ,
eds. Williams Hematology. 8th ed. New York, NY: McGraw-Hill; 2010.
2. Avery P, Shah N, Fu W, et al. Chapter 11. Multiple myeloma and other plasma cell dyscrasias. In: Kantarjian HM, Wol RA, Koller
CA, eds. T e MD Anderson Manual of Medical Oncology. 2nd ed. New York, NY: McGraw-Hill; 2011.
3. Kyle RA, Rajkumar SV. Criteria or diagnosis, staging, risk strati cation and response assessment o multiple myeloma. Leukemia.
2009;23(1):39.

35

HEMATOLOGY-ONCOLOGY

TREATMENT APPROACH

NeutropenicFever

Initial
evaluation

Initial
therapy

P hys ica l exa mina tion


Abs olute ne utrophil count 500/mm 3
Blood culture s ; che s t ra diogra ph; othe r a ppropria te
s tudie s ba s e d on his tory a nd phys ica l exa m

Antibiotics : gra m-ne ga tive +/


gra m-pos itive cove ra ge

Follow-up

Obvious infe ctious


s ite found

Subsequent
therapy

Ta ilor a ntibiotics
a s a ppropria te

No obvious
infe ctious s ite

Afe brile

Fe brile

Continue
re gime n

Add
a ntifunga l
a ge nt

Continue tre a tme nt until ne utrope nia re s olve s (a bs olute ne utrophil count >500/L)

(LongoDL,FauciAS,KasperDL,etal.HarrisonsPrinciplesofInternalMedicine,18E.NewYork,NY:
McGraw-Hill; 2012.)

Figure 4 6. Treatment algorithm or neutropenic ever.


36

HEMATOLOGY-ONCOLOGY

TREATMENT APPROACH

NeutropenicFever
Case: A 54 year old man with a history o non Hodgkins lymphoma
presents with ever 10 days a ter receiving chemotherapy and is noted
to be neutropenic.
What is the de nition o neutropenic ever (aka ebrile neutropenia)?1

Absolute neutrophil count (ANC) 500/mm 3 + oral temperature 100.4 F


or over an hour or a single oral temperature > 101 F
What organisms account or the highest morbidity and mortality in
neutropenic ever?2
Aerobic gram-negative organisms (principally Pseudomonas aeruginosa)
A er obtaining a history, how would you begin your evaluation o
neutropenic ever?3
Physical exam: skin, mucous membranes, lungs, vascular access, peri-rectum
Labs: CBC with di erential, basic metabolic panel, liver unction tests
Additional tests: blood cultures 2 (peripheral set + rom each lumen o
central catheter i present), chest x-ray, urine culture i symptomatic or
catheter in place
Further testing (stool cultures i diarrhea; CSF testing i concern or
meningitis) based on clinical suspicion
What type o empiric antibiotics should be started or neutropenic ever?2

Gram-negative agents with excellent anti-pseudomonal coverage


Ce azidime (2 g IV q8h) or ce epime (2 g IV q12h)
Piperacillin/tazobactam (3.375 g IV q6h)
Imipenem (500 mg IV q6h) or meropenem (1 g IV q8h), especially i concern
or extended-spectrum beta-lactamase (ESBL) producing organism
What clinical eatures merit empiric MRSA/gram-positive coverage?4

Suspected catheter-related in ection, suspected skin or so -tissue in ection,


blood cultures positive or a gram-positive organism, known history o
MRSA in ection, and/or hemodynamic instability
When should ungal coverage be added?4
I persistent ever a er 4 days, new ever a er initial de ervescence, and/or
new pulmonary in ltrates develop despite broad-spectrum antibiotics
I no identi able source is ound and the patient de ervesces, when can
antibiotics be discontinued?4
Upon resolution o neutropenia (ANC > 500/mm 3) or at least 2 days

REFERENCES
1. Bow EJ. Chapter 47. Approach to in ection in patients receiving cytotoxic chemotherapy or malignancy. In: Hall JB, Schmidt GA,
Wood LH, eds. Principles of Critical Care. 3rd ed. New York, NY: McGraw-Hill; 2005
2. Ramphal R. Chapter 152. In ections due to pseudomonas species and related organisms. In: Longo DL, Fauci AS, Kasper DL,
Hauser SL, Jameson J, Loscalzo J, eds. Harrisons Principles of Internal Medicine. 18th ed. New York, NY: McGraw-Hill; 2012.
3. Rolston KI. Chapter 43. In ection in the neutropenic patient. In: Kantarjian HM, Wol RA, Koller CA, eds. T e MD Anderson
Manual of Medical Oncology. 2nd ed. New York, NY: McGraw-Hill; 2011.
4. Davidson J, Chia SK. Chapter 182. Oncologic emergencies. In: McKean SC, Ross JJ, Dressler DD, Brotman DJ, Ginsberg JS, eds.
Principles and Practice of Hospital Medicine. New York, NY: McGraw-Hill; 2012.

36

37

ri

xi

xi

HEMATOLOGY-ONCOLOGY
PHYSICAL EXAM

Splenomegaly

HEMATOLOGY-ONCOLOGY

PHYSICAL EXAM

Splenomegaly
Case: A 20 year old college student presents with atigue, sore throat and
abdominal ullness.
What are the most common mechanisms o splenomegaly?1

Hypertrophy due to removal o de ective RBCs (hereditary spherocytosis,


thalassemias), immune response (in ectious mononucleosis), or immunologic diseases (I P, SLE)
Passive congestion due to portal hypertension (cirrhosis, CHF, BuddChiari)
In ltrative diseases (lymphoma, amyloidosis, myeloproli erative disorders)
De ne Castells point and how it is used to detect splenomegaly compared

to palpation1.
Castells point: the most in erior interspace along the le anterior axillary
line
Patient position: supine
echnique: percuss Castells point throughout the respiratory cycle
I dullness is noted (on inspiration) positive or splenomegaly
(82% sensitive, 83% speci c)2
More sensitive/speci c than raubes space percussion 2
Palpation:
Patient position: supine with knees bent (right lateral decubitus position
may improve detection)
echnique: using ngertips o right hand, start in LLQ and move toward
le costal margin (can apply counter-pressure to f ank with le hand)
I spleen tip palpated below costal margin positive or splenomegaly
(58% sensitive, 92% speci c)2
How does pre-test probability inf uence the utility o the exam maneuvers?2

I pre-test probability low (< 10%), both maneuvers are not sensitive/speci c
enough to rule-out or rule-in splenomegaly (need to obtain imaging)
I pretest probability high ( 10%), then start with percussion
I percussion positive proceed with palpation
I palpation positive splenomegaly likely
I palpation negative obtain imaging
I percussion negative no need to palpate obtain imaging
Palpation here is not speci c enough to rule-in splenomegaly

REFERENCES
1. Henry PH, Longo DL. Chapter 59. Enlargement o lymph nodes and spleen. In: Longo DL, Fauci AS, Kasper DL, Hauser SL,
Jameson J, Loscalzo J, eds. Harrisons Principles of Internal Medicine. 18th ed. New York, NY: McGraw-Hill; 2012.
2. Grover SA, Barkun AN, Sackett DL. T e rational clinical examination. Does this patient have splenomegaly? JAMA. 1993;270(18):
22182221.

37

38

ri

<

HEMATOLOGY-ONCOLOGY
CLASSIFICATION

Trans usion Reactions

HEMATOLOGY-ONCOLOGY

CLASSIFICATION

Trans usion Reactions


Case: A 48 year old woman with multiple myeloma with a hemoglobin
level o 6 g/dL receives a trans usion; 30 minutes a ter the trans usion is
started, she develops a ever.
When consenting a patient or blood, what possible acute adverse e ects

must be discussed?
Nonimmune: f uid overload, electrolyte imbalance (e.g., via chelation o calcium)
Immune reactions: ebrile nonhemolytic trans usion reaction (FNH R),
allergic (urticarial) reaction, acute hemolytic trans usion reaction (AH R),
anaphylactic reaction, trans usion-related acute lung injury ( RALI)
How do you di erentiate among the acute immune trans usion reactions?1,2

Based on mechanism, signs/symptoms, timing, and treatment (see chart)


What are delayed immune trans usion reactions?

Delayed hemolytic trans usion reaction: anamnestic antibody response


against reexposure to oreign red cell antigen (e.g., Rh). Antibodies are
undetectable pretrans usion but rise rapidly ollowing trans usion
Presents as hemolysis, slight ever within 330 days a er trans usion
No treatment needed, but need to screen uture trans usions or antigen
rans usion-associated gra -versus-host disease ( -GVHD): rare. Donor
-lymphocytes engra and recognize an immunocompromised hosts HLA
antigens as oreign and mount immune response (does not occur in the immunocompetent host, as the host-immune system eliminates donor lymphocytes)
Presents as pancytopenia, ever, cutaneous eruption, diarrhea, liver unction
abnormalities 810 days a er trans usion. reat with immunosuppressives.
Can immune-mediated reactions be prevented?3,4,5
Yes: using medications and modi ying blood products
Antipyretics and antihistamines: although universally used to prevent
FHN Rs or allergic reactions, RC s showed no di erence in overall risk o
trans usion reactions5
Leukocyte reduction: reduces risk o ebrile nonhemolytic trans usion
reaction
Washed cells: reduces risk o anaphylactic and allergic trans usion reactions
-irradiated cells: reduces risk o trans usion-related gra -versus-host disease

REFERENCES
1. Sanders RP, Geiger L, Heddle N, Pui CH, Howard SC. A revised classi cation scheme or acute trans usion reactions. ransfusion.
2007; 47:621628.
2. Dzieczkowski JS, Anderson KC. Dzieczkowski JS, Anderson KCChapter 113. rans usion biology and therapy. In: Longo DL, Fauci
AS, Kasper DL, Hauser SL, Jameson J, Loscalzo J. Longo DL, Fauci AS, Kasper DL, Hauser SL, Jameson J, Loscalzo J, eds. Harrisons
Principles of Internal Medicine. 18th ed. New York, NY: McGraw-Hill; 2012.
3. Shanwell A, Kristiansson M, Remberger M, Ringdn O. Generation o cytokines in red cell concentrates during storage is prevented
by prestorage white cell reduction. ransfusion. 1997;37:678684.
4. Stack G, Snyder EL. Cytokine generation in stored platelet concentrates. ransfusion. 1994;34:2025.
5. Kennedy LD, Case LD, Hurd DD, Cruz JM, Pomper GJ. A prospective, randomized, double-blind controlled trial o acetaminophen and diphenhydramine pretrans usion medication versus placebo or the prevention o trans usion reactions. ransfusion.
2008;48:22852291.

38

PATHOPHYSIOLOGY

HEMATOLOGY-ONCOLOGY

Tumor Lysis Syndrome

TUMOR
T
TUM
TU
U
UM
MOR
OR
C EL
CE
CEL
CELL
ELL
L

CYTOKINES

HYP ERKALEMIA

NUCLEIC
ACID

HYP ERP HOS P HATEMIA

XANTHINE

BINDS CALCIUM
ALLOP URINOL
HYP OCALCEMIA

URIC ACID
ECTOP IC
CRYS TAL
DEP OS ITION
RAS BURICAS E

TETANY,
S EIZURES
RENAL FAILURE
Due to:
- uric a cid induce d va s ocons triction,
re na l pe rfus ion
- uric a cid crys ta l de pos ition
- Ca -P hos crys ta l
de pos ition

S IRS , MULTIORGAN FAILURE

Figure 4 9. Pathophysiology o tumor lysis syndrome.

39

CARDIAC
ARRHYTHMIAS

HEMATOLOGY-ONCOLOGY

PATHOPHYSIOLOGY

Tumor Lysis Syndrome


Case: A 35 year old male with acute myeloid leukemia is admitted or
induction chemotherapy. You consider the possible complications, including
tumor lysis syndrome (TLS).
How is TLS de ned?

T e CairoBishop de nition o laboratory LS is 2 o the ollowing in the


3 days be ore to 7 days a er initiating chemotherapy:1
Uric acid 8 mg/dL; potassium 6 mEq/L; phosphorus 6.5 mg/dL;
calcium 7 mg/dL; or a change in the same direction o any o these labs
o 25% rom baseline
What types o cancer are prone to TLS?2,3
Most common with hematologic malignancies (e.g., AML, Burkitts lymphoma),
but may also be seen in other cancers with a high tumor cell proli eration
rate, bulky tumor mass, or high chemosensitivity
Some patient characteristics increase the risk: pre-existing nephropathy,
dehydration, exogenous K+ , acidic urine, and pretreatment hyperuricemia
or hyperphosphatemia
How does TLS cause morbidity and mortality?

LS primarily a ects the heart and kidney:


Arrhythmias are caused by hyperkalemia, calcium-phosphate precipitation
in the conduction system, and hypocalcemia
Renal ailure is caused by:
Calcium-phosphate crystals precipitate (exacerbated by uric acid)
Uric acid crystals precipitate (exacerbated by calcium phosphate and
acidosis)
Uric acid also causes renal vasoconstriction
What are the other e ects o TLS to watch or?

Hypocalcemia seizures, tetany; released cytokines SIRS, multiorgan


ailure
What are the keystones to TLS prevention and management?
Prevention: hyperhydration, diuretics, prevention o uric acid ormation
with allopurinol, avoid calcium supplementation unless symptomatic
Management: breakdown o uric acid with rasburicase, phosphate binding
with sevelamer, treat hyperkalemia; hemodialysis or severe kidney injury.

REFERENCES
1. Cairo MS, Bishop M. umour lysis syndrome: new therapeutic strategies and classi cation. Br J Haematol. 2004;127(1):311.
2. Howard SC, Jones DP, Pui CH. T e tumor lysis syndrome. N Engl J Med. 2011;364:18441854.
3. Gucalp R, Dutcher J. Chapter 276. Oncologic emergencies. In: Longo DL, Fauci AS, Kasper DL, Hauser SL, Jameson J, Loscalzo J,
eds. Harrisons Principles of Internal Medicine. 18th ed. New York, NY: McGraw-Hill; 2012.

39

HEMATOLOGY-ONCOLOGY

TREATMENT APPROACH

War arin-Induced Coagulopathy

Eleva te d INR

Life thre a te ning or s e rious


ble e ding pre s e nt

Yes

Hold wa rfa rin inde finite ly


IV vita min K 10 mg, re pe a t in 12 hrs
if INR s till e leva te d
FFP (34 units ) or othe r clotting fa ctor
re pla ce me nt

No

INR >9

Yes

Hold next 12 wa rfa rin dos e s


Ora l vita min K 2.55 mg
Re s ume wa rfa rin whe n INR is the ra pe utic

No

INR 59

Yes

Hold next 12 wa rfa rin dos e s


If high-ris k for ble e ding,* cons ide r
ora l vita min K 12.5 mg
Re s ume wa rfa rin whe n INR is the ra pe utic

No

INR 3.55

Yes

Lowe r wa rfa rin dos e or omit one dos e


for pa tie nts a t high ris k of ble e ding*
Re s ume wa rfa rin whe n INR is the ra pe utic

*High-ris k for ble e ding: a ge >75 ye a rs, concurre nt a ntipla te le t drug us e, live r or re na l
dis e a s e , re ce nt s urge ry, or tra uma .
(SlatteryDE,PollackCV,Jr..Chapter234.Anticoagulants,AntiplateletAgents,andFibrinolytics.In:Tintinalli
JE, Stapczynski J, Ma O, Cline DM, Cydulka RK, Meckler GD, T. eds. Tintinallis Emergency Medicine: A
Comprehensive Study Guide, 7e. New York, NY: McGraw-Hill; 2011.)

Figure 4 10. Treatment algorithm or war arin-induced coagulopathy.

40

HEMATOLOGY-ONCOLOGY

TREATMENT APPROACH

War arin-Induced Coagulopathy


Case: A 75 year old man with atrial f brillation on war arin presents with
an INR o 7 a ter starting antibiotics or a community acquired pneumonia.
He has no evidence o bleeding.
Why do patients on war arin get supratherapeutic INRs?1

Medication interactions (concomitant use o medications that decrease


war arin metabolism, such as amiodarone or levof oxacin)
Incorrect dosing due to misadministration or dispensing errors
New/worsening systemic illness
Hepatic ailure: production o clotting actors
Hypothyroidism: catabolism o clotting actors
CHF: hepatic congestion impaired war arin metabolism
Dietary e ects (abrupt avoidance o green lea y vegetables)
What are the three means o lowering a supratherapeutic INR rom war arin?2
1. Hold war arin therapy
2. Administer vitamin K (potency: IV > PO > SC route)
3. Replace clotting actors
Fresh rozen plasma (FFP): only i INR > 1.5
Prothrombin complex concentrate (PCC) or recombinant actor VIIa or
emergent reversal
What are the indications or IV Vitamin K and clotting actor replacement?
Elevated INR and serious/li e-threatening bleeding, INR> 20, or war arin
overdose
I there is no signi cant bleeding, what should be done or the ollowing

INR ranges:2
INR > 9: hold war arin and administer oral vitamin K (2.55 mg)
INR 59: hold war arin and administer low-dose oral vitamin K (12.5 mg)
i high risk or bleeding (low-dose vitamin K will lower INR within 16 hr)
INR 3.55: lower war arin dose or hold war arin i high risk or bleeding
In addition to over-correction o INR, what are the speci c risks o vitamin K

and the various clotting actor replacements?3


Vitamin K: anaphylaxis (IV administration; no risk or PO route)
FFP: trans usion-related acute lung injury ( RALI) and volume overload
PCC or recombinant actor VIIa: venous and arterial thrombosis

REFERENCES
1. Ja er A, Bragg L. Practical tips or war arin dosing and monitoring. Cleve Clin J Med. 2003;70(4):361371.
2. Slattery DE, Pollack CV Jr. Chapter 234. Anticoagulants, antiplatelet agents, and brinolytics. In: intinalli JE, Stapczynski J, Ma O,
Cline DM, Cydulka RK, Meckler GD, eds. intinallis Emergency Medicine: A Comprehensive Study Guide. 7th ed. New York, NY:
McGraw-Hill; 2011.
3. Baggett M, Hunt DP. Chapter 76. Bleeding and coagulopathy. In: McKean SC, Ross JJ, Dressler DD, Brotman DJ, Ginsberg JS, eds.
Principles and Practice of Hospital Medicine. New York, NY: McGraw-Hill; 2012.

40

DIAGNOSTIC APPROACH

INFECTIOUS DISEASE

Acute HIV

ACUTE

LATENT

Opportunis tic infe ctions


a nd ma ligna ncie s

Acute s ymptoms

IMMUNODEFICIENCY

CD4 lymphocyte s

Anti-p24 a ntibodie s
Anti-gp120 a ntibodie s
Virus , vira l RNA, p24 a ntige n
0

2
3
4
5
Time a fte r infe ction (mo)

3 10
Time a fte r infe ction (y)

(Reproduced with permission rom Weiss RA. How does HIV cause AIDS? Science. 1993;260:1273.)

Figure 5-1. Laboratory pro le o HIV in ection over time.

41

INFECTIOUS DISEASE

DIAGNOSTIC APPROACH

Acute HIV
Case: A 34-year-old IV drug user presents with 2 weeks o ever, malaise,
and weight loss. You suspect a new diagnosis o HIV.
How might some patients with HIV rst present?1

5070% o patients may experience acute retroviral syndrome (ARS),


presenting as ever, lethargy, pharyngitis, rash, and lymphadenopathy, which
usually occurs 24 weeks a er in ection. T is resolves spontaneously.
What tests are available or diagnosing HIV?
ELISA or antibodies to HIV antigens (e.g., p24). Sensitive (> 99.5%) but not
highly speci c.2 Positive 37 weeks a er in ection
Western blot or antibodies to HIV antigens (eg., p24 and gp120)
Highly speci c. Positive 46 weeks a er in ection.
Direct detection o virus via RNA PCR or HIV antigen detection (p24
capture assay). Positive 1015 days a er in ection
How do you diagnose acute HIV in ection?1,2

T e HIV virus is undergoing high levels o replication viral load at


> 1 million copies/mL; may cause transient drop in CD4 cells as they are
in ected/killed
Anti-HIV antibodies have not yet developed; thus ELISA and Western
blots are likely to be alsely negative during this window period be ore
seroconversion
Use direct detection o virus via RNA PCR or HIV antigen detection
(p24 assay)
How do you diagnose HIV in patients who are not acutely in ected?
Anti-HIV antibodies are developed viral load drops to a set point
which is constant or years (~711 years in untreated patients). ELISA and
Western Blot are commonly used or screening in this population.
A higher set point corresponds with higher risk o progression to
symptomatic AIDS
Which patients with HIV in ection should receive antiretroviral therapy

(ART)?
AR should be initiated in the majority o patients with HIV in ection,
regardless o CD4 count
T e S AR trial showed that early initiation o AR in patients with HIV
and CD4 counts > 500 cells/L had improved outcomes compared to AR
de erred until CD4 count ell below 350 cells/L3

REFERENCES
1. Fauci AS, Lane H. Chapter 189. Human immunode ciency virus disease: AIDS and related disorders. In: Longo DL, Fauci AS,
Kasper DL, Hauser SL, Jameson J, Loscalzo J, eds. Harrisons Principles of Internal Medicine. 18th ed. New York, NY: McGraw-Hill;
2012. http://accessmedicine.mhmedical.com/content.aspx?bookid= 331&Sectionid= 40726947. Accessed April 23, 2014.
2. Chapter 45. Human immunode ciency virus. In: Levinson W, eds. Review of Medical Microbiology & Immunology. 12th ed. New
York, NY: McGraw-Hill; 2012. http://accessmedicine.mhmedical.com/content.aspx?bookid= 400&Sectionid= 42098509. Accessed
April 23, 2014.
3. INSIGH S AR Study Group. Initiation o antiretroviral therapy in early asymptomatic HIV in ection. NEJM. 2015;379(9):795.

41

TREATMENT APPROACH

INFECTIOUS DISEASE

AsymptomaticBacteriuria
P os itive urine culture in the
a bs e nce of:
Urina ry s ymptoms
S ys te mic s ymptoms
re la te d to urina ry
infe ction (fe ve rs , chills )

P re gna nt
Re na l tra ns pla nt re cipie nt
Unde rgoing inva s ive urologic

Ha s urina ry ca the te r

All othe r pa tie nts

proce dure

Tre a t

No a ntibiotics ne e de d
but re move ca the te r

Do not tre a t

(Adapted rom Figure 288-4 rom Long DL, Fauci AS, Kasper DL, Hauser SL, Jameson JL, Loscalzo J. Harrisons
Principles o Internal Medicine. 18th ed.)

Figure 5-2. Management o asymptomatic bacteriuria.

42

INFECTIOUS DISEASE

TREATMENT APPROACH

AsymptomaticBacteriuria
Case: A 34-year-old male paraplegic with chronic indwelling catheter has
no urinary symptoms but is ound to have 3+ WBC on UA and a culture
with > 10 5 c u/mL o Escherichia coli.
How is asymptomatic bacteriuria de ned?

1. Absence o symptoms o U I (dysuria, requency, hematuria, back pain) or


more general symptoms o in ection ( ever, malaise, etc.)
2. Isolation o a single organism in urine in the ollowing quantities:
a. Women: > 2 consecutive clean-catch specimens with 105 c u/mL
b. Men: single clean-catch specimen with 105 c u/mL
c. Catheter sample: single sample o 102 c u/mL
How common is asymptomatic bacteriuria?
Rare in men but airly common in women (27%).1 Prevalence increases
with age (women older than age 80: 20%; men older than age 75: 615%)1
and sexual activity
Also increased in emale diabetic patients, spinal cord injury patients, and
patients with indwelling urethral catheters. Prevalence is NO increased in
pregnancy1
Which patients with asymptomatic bacteriuria should be treated?

reat only pregnant patients, renal transplant patients, and patients


undergoing urologic procedures
All other patients, including diabetics, patients with spinal cord injury,
patients with chronic indwelling oley, patients undergoing joint
arthroplasty, and elderly patients need not receive antibiotics
Data shows treatment o these patients does not a ect outcomes or
subsequent development o U I symptoms and instead may lead to
emergence o resistant bacteria2,3
Why might patients not bene t rom treating asymptomatic bacteriuria?4

A lack o symptoms may re ect the act that the organisms involved are less
virulent and thus rarely progress to serious in ection
It has been proposed that colonization with uroprotective strains o E. coli
may protect against more invasive pathogens in spinal cord injury patients
Patients in studies who received antibiotics recolonized shortly a er treatment

REFERENCES
1. Nicolle LE. Asymptomatic bacteriuria: when to screen and when to treat. Infect Dis Clin North Am. 2003;17:367394.
2. Leone M, Perris AS, Granier I, et al. A randomized trial o catheter change and short course o antibiotics or asymptomatic bacteriuria in catheterized ICU patients. Intensive Care Med. 2007;33:726729.
3. Warren JW, Anthony WC, Hoopes JM, Muncie HL Jr. Cephalexin or susceptible bacteriuria in a ebrile, long-term catheterized
patients. JAMA. 1982;248:454458.
4. Nicolle LE, Bradley S, Colgan R, et al. In ectious Diseases Society o America guidelines or the diagnosis and treatment o asymptomatic bacteriuria in adults. Clin Infect Dis. 2005;40:643654.

42

CLASSIFICATION

INFECTIOUS DISEASE

Cellulitis and Erysipelas

Diagnosis

Likely Pathogen

Treatment

Nonpurulent cellulitis
without MRSA risk actorsa

-hemolytic
streptococci, MSSA

All oral:
Cephalexin
Cephadroxil
Dicloxacillin
Clindamycin

Nonpurulent cellulitis with


MRSA risk actors
OR
Purulent cellulitis
(localized)

-hemolytic
streptococci, MSSA,
MRSA

All oral:
Clindamycin
Trimethoprimsulfamethoxazole
Minocycline
Doxycycline
Linezolid

Purulent cellulitis with


signsofsystemictoxicity

MSSA, MRSA,
-hemolytic
streptococci

Vancomycin(IV)
Linezolid(IVororal)

Mild erysipelas

-hemolytic
streptococci

All oral:
Penicillin
Amoxicillin(alloral)

Erysipelas with signs o


systemictoxicity

-hemolytic
streptococci

Ceftriaxone(IV)

MRSA risk actors include recent hospitalization or long-term care acility, recent antibiotic use, HIV
in ection, IV drug use, hemodialysis, shared sports equipment, military service.

Figure 5-3. Description o the common types o skin and so t tissue


in ections.

43

INFECTIOUS DISEASE

CLASSIFICATION

Cellulitis and Erysipelas


Case: A 34-year-old male presents with a di use rash on his leg concerning
or a skin in ection.
What are the di erent types o skin and so tissue in ections?

Cellulitis, erysipelas, impetigo, asciitis, olliculitis/abscess, uruncle/carbuncle,


animal/human bite
How do erysipelas and cellulitis di er in appearance?
Cellulitis a ects deeper layers; may be associated more with swelling,
erythema; may be purulent or nonpurulent
Clinical course develops over a ew days time
Erysipelas a ects only the super cial skin layers and thus has a more
distinctive raised appearance above surrounding skin with a clear line o
demarcation
Clinical course may be more acute in onset with evers/chills
What are the most likely pathogens or cellulitis and erysipelas, and how

does this inf uence antibiotic choice?13


Cellulitis:
Nonpurulent without MRSA risk actors: -hemolytic streptococcus
(73%)4 or MSSA (see image)
Note: trimethoprim-sul amethoxazole is NO adequate empiric
treatment or suspected streptococcus
Nonpurulent with MRSA risk actors or purulent: MRSA, MSSA,
-hemolytic streptococcus (see image)
Erysipelas: -hemolytic streptococcus (see image)
When should cultures be sent?

Blood cultures, needle aspirations, biopsies are NO help ul in mild in ections (e.g., blood cultures positive in less than 5% o cases)5
Obtain cultures o blood, pus, bullae in patients with systemic toxicity or
extensive skin involvement, underlying immunode ciency, or recurrent/
persistent cellulitis
How long should patients with erysipelas and cellulitis be treated or?
Most patients can be treated or 510 days, tailored to response to antibiotics.

REFERENCES
1. Halilovic J, Heintz BH, Brown J. Risk actors or clinical ailure in patients hospitalized with cellulitis and cutaneous abscess. J Infect.
2012;65:128134.
2. Moran GJ, Krishnadasan A, Gorwitz RJ, et al. Methicillin-resistant S. aureus in ections among patients in the emergency department. N Engl J Med. 2006;355:666674.
3. Stevens DL, Bisno AL, Chambers HF, et al. Practice guidelines or the diagnosis and management o skin and so -tissue in ections.
Clin Infect Dis. 2005;41:13731406.
4. Jeng A, Beheshti M, Li J, et al. T e role o beta-hemolytic streptococcus in causing di use, nonculturable cellulitis: a prospective
investigation. Medicine (Baltimore). 2010;80:217226.
5. Perl B, Gottehrer NP, Raveh D, et al. Cost-e ectiveness o blood cultures or adult patients with cellulitis. Clin Infect Dis. 1999;
29:14831488.

43

CLASSIFICATION

INFECTIOUS DISEASE

Community-Acquired Pneumonia

PORT/PS I S c o ring S ys te m

CURB-65 S c o ring S ys te m

Age (s ubtra ct 10 for wom e n)

Nurs ing home

10

Ne opla s m (active , not skin)

30

Cirrhos is or chronic he pa titis

20

CHF, CVA, chronic re na l dis e a s e *

10

Alte re d me nta l s ta tus

20

Re s pira tory ra te 30

20

S ys tolic BP < 90

20

Temp < 35 or 40C

15

P uls e 125/min

10

Po int S c o re

Mo rtality

Arte ria l pH < 7.35

30

0.7%

BUN 30 mg/dL (11 mmol/L)

20

3.2%

S e rum s odium < 130

20

3%

Glucos e 250 mg/dL (13.9 mmol/L)

10

17%

He ma tocrit < 30

10

41.5%

pO 2 < 60

10

57%

P le ura l e ffus ion

10

Po int S c o re **

Clas s

Mo rtality

70

II

< 1%

7190

III

2.8%

91130

IV

8.2%

> 130

29.2%

One point for e a ch of the following:



Confus ion

Ure a > 19 mg/dL (7 mmol/L)

Re s pira tory ra te > 30

Blood pre s s ure < 90/60 (e ithe r


s ys tolic or dia s tolic va lue )

65 ye a rs of a ge

S core of 2 indica te s ne e d for hos pita liza tion.

*10 points for e a ch dia gnos is


Cla s s IV-V indica te s ne e d for hos pita liza tion
**Fore go ca lcula tion a nd a s s ign pa tie nt to cla s s I (morta lity of 0.1%) if 50 ye a rs old, no s ignifica nt co-morbiditie s a nd norma l
vita l s igns with inta ct me nta l s ta tus

(Musher DM. Chapter 189. Community-Acquired Pneumonia. In: McKean SC, Ross JJ, Dressler DD, Brotman
DJ, Ginsberg JS. eds. Principles and Practice o Hospital Medicine. New York, NY: McGraw-Hill; 2012.)

Figure 5-4. The PORT/PSI vs. CURB-65 scoring systems or


community-acquired pneumonia.

44

INFECTIOUS DISEASE

CLASSIFICATION

Community-Acquired Pneumonia
Case: A 72-year-old man with a history o type II diabetes mellitus presents
with 2 days o ever, productive cough, and an inf ltrate on chest x-ray.
What validated prognostic scores are available to help determine the risk

o short-term mortality in patients with community-acquired pneumonia


(CAP)?
POR /PSI score: varying points given or 20 di erent variables
CURB-65 score: one point or meeting each o 5 di erent variables
Although easier to calculate, the CURB-65 score is less sensitive and less
speci c than the POR /PSI score1
How can the PORT/PSI and CURB-65 scores be used to guide care?2

Predict 30-day mortality risk


Also requently used to help determine which patients to admit to the
hospital
POR /PSI score:
Class I-II outpatient management
Class III consider observation unit vs. outpatient management
Class IV-V admit to hospital
CURB-65 score:
Score < 2 outpatient management
Score 2 admit to hospital
How can we predict which patients with CAP require ICU admission?3

Calculate the SMAR -COP score


SBP (2 pts), multilobar (1 pt), albumin (1 pt), RR (1 pt), tachycardia
(1 pt), con usion (1 pt), oxygenation (2 pts), arterial pH (2 pts)
Score 3 indicates need or ICU admission (correctly identi ed 84.8% o
patients that needed intensive respiratory + / vasopressor support)
Outper ormed both the POR /PSI and CURB-65 scores
What antibiotics should be started empirically or a patient with CAP?4
Outpatients: i no recent antibiotic use and no signi cant comorbidities
doxycycline or macrolide. I in a region with increased macrolide-resistant
Streptococcus pneumoniae, then macrolide monotherapy is not recommended
(consider below regimens)
Inpatients: anti-pneumococcal uoroquinolone or beta-lactam + macrolide

REFERENCES
1. Musher DM. Chapter 189. Community-acquired pneumonia. In: McKean SC, Ross JJ, Dressler DD, Brotman DJ, Ginsberg JS, eds.
Principles and Practice of Hospital Medicine. New York, NY: McGraw-Hill; 2012.
2. Mandell LA, Wunderink R. Chapter 257. Pneumonia. In: Longo DL, Fauci AS, Kasper DL, Hauser SL, Jameson J, Loscalzo J, eds.
Harrisons Principles of Internal Medicine. 18th ed. New York, NY: McGraw-Hill; 2012.
3. Chalmers JD, Singanayagam A, Hill A . Predicting the need or mechanical ventilation and/or inotropic support or young adults
admitted to the hospital with community-acquired pneumonia. Clin Infect Dis. 2008;47(12):15711574.
4. Mandell LA, Wunderink RG, Anzueto A, et al.; In ectious Diseases Society o America; American T oracic Society. In ectious Diseases Society o America/American T oracic Society consensus guidelines on the management o community-acquired pneumonia
in adults. Clin Infect Dis. 2007;44(Suppl 2):S27S72.

44

INFECTIOUS DISEASE

DIAGNOSTIC APPROACH

Fever of Unknown Origin


Labo rato ry te s ting
Ele ctrolyte s , BUN/Cr, CBC with diffe re ntia l, s me a r,
ES R, CRP , urina lys is , live r function te s ts , mus cle
e nzyme s , Tre pone ma l IgG, HIV, CMV, EBV, ANA,
RF, S P EP , P P D, TS H
Culture s : Blood 3, urine , s putum, body fluids a s
a ppropria te

Dis c o ntinue all me dic atio ns


CT c he s t/abdo me n/pe lvis

Dire c te d
inve s tig atio n

ECHO
Nuc le ar s c ans

(67 Ga, 111 In

PMN, FDG PET)

Le g do pple rs

If ag e >50, te mpo ral arte ry bio ps y

No diag no s is

Figure 5-5. Diagnostic algorithm or ever o unknown origin.


45

INFECTIOUS DISEASE

DIAGNOSTIC APPROACH

Fever of Unknown Origin


Case: A 77-year-old emale is admitted with 1 month o cyclic evers.
A basic outpatient workup has been negative.
What are the diagnostic criteria o

ever o unknown origin (FUO)?1


emperature o 101F on several occasions
Duration o ever > 3 weeks
Uncertain diagnosis a er 1 week o study in the hospital
What etiologies o ever should be considered a er an initial workup is
negative?
In ection: bacterial (e.g., endocarditis, tuberculosis, paraspinal abscesses, etc.),
viral (e.g., CMV, HIV), parasitic (e.g., trypanosomes), ungal (e.g.,
histoplasmosis, coccidioidomycosis), zoonoses (e.g., Lyme, Ehrlichia, Coxiella)
Neoplasm: leukemia, lymphoma, renal cell carcinoma
Nonin ectious in ammatory: temporal arteritis, adult Stills, sarcoidosis
Miscellaneous: pulmonary embolism, hereditary periodic ever, drugs,
hyperthyroidism
Many cases will remain undiagnosed2
How do you workup FUO?

1. Obtain a detailed history, including recent travel/exposures, pattern o


ever, accompanying symptoms
2. Basic laboratory testing (see chart)
3. Discontinue all medications
4. I ever persists, obtain advanced testing (see chart)
What are your options or patients who remain undiagnosed a er an

extensive FUO evaluation?3,4


T ere are two general approaches:
1. Watch ul waiting
2. Empiric therapy: NSAIDs, steroids, colchicine, anti- B treatment, antibiotics
Patients who remain undiagnosed a er an extensive evaluation generally
have a good prognosis. T ere ore, avoid empiric therapy i possible.

REFERENCES
1. Petersdor RG, Beeson PB. Fever o unexplained origin: report on 100 cases. Medicine (Baltimore). 1961;40:130.
2. de Kleign EM, Vandenbroucke JP, van der Meer JW. Fever o unknown origin (FUO). A prospective multicenter study o 167 patients
with FUO, using xed epidemiologic entry criteria. T e Netherlands FUO Study Group. Medicine (Baltimore). 1997;76:392400.
3. Gel and JA, Callahan MV. Chapter 18. Fever o Unknown Origin. In: Longo DL, Fauci AS, Kasper DL, Hauser SL, Jameson J,
Loscalzo J, eds. Harrisons Principles of Internal Medicine. 18th ed. New York, NY: McGraw-Hill; 2012. http://accessmedicine.
mhmedical.com/content.aspx?bookid= 331&Sectionid= 40726728. Accessed March 12, 2014.
4. Knockaert DC, Dujardin KS, Bobbaers HJ. Long-term ollow-up o patients with undiagnosed ever o unknown origin. Arch Intern
Med. 1996;156:618620.

45

INFECTIOUS DISEASE

CLASSIFICATION

Fungal Markers

1,3--D-Glucan

Fungal in ections
associated with
a positive result

Galactomannan

Candidemia/invasive
candidiasis

Aspergillosis

Pneumocystis jirovecii
pneumonia ( ormerly PCP)

Cryptococcosis

Aspergillosis
(less sensitive/speci c than
galactomannan assay)

Histoplasmosis

Histoplasmosis and other


endemic mycoses
(e.g., coccidioidomycosis,
blastomycosis)

Other mold in ections


(e.g., Penicillium species)

Bacteremia (especially rom Pseudomonas aeruginosa or


Streptococcus pneumoniae)
Causes o
alse -positive
results

Hemodialysis with cellulose


membranes

Gra t-versus-hostdisease o the GI tract

Immunoglobulin or albumin
in usion with cellulose lters

Gluconate-containing IV
uids (e.g., Plasmalyte)

Gauze in contact with


mucosal sur aces
(Ross JJ. Chapter 192. Candida and Aspergillus. In: McKean SC, Ross JJ, Dressler DD, Brotman DJ, Ginsberg
JS. eds. Principles and Practice o Hospital Medicine. New York, NY: McGraw-Hill; 2012.)

Figure 5-6. Causes o positive ungal marker testing.

46

INFECTIOUS DISEASE

CLASSIFICATION

Fungal Markers
Case: A 32-year-old woman who recently underwent a bone marrow
transplant presents with ever o unknown etiology without any localizing
symptoms. A ungal in ection is considered.
What ungal markers are used to diagnose an invasive ungal in ection?

1,3--D-glucan and galactomannan (serum assay tests)


Components o the cell wall o speci c ungi
What ungal in ections are most speci cally associated with a positive glucan
vs. galactomannan result?1
Positive glucan: most speci c or candidemia/invasive candidiasis
Positive galactomannan: most speci c or aspergillosis
Are there alse positive results or each ungal marker?1

Yes: or both ungal markers


Glucan:
Bacteremia, hemodialysis with cellulose membranes, immunoglobulin or albumin in usion with cellulose lters, or gauze in contact with
mucosal sur aces
Galactomannan:
Bacteremia, gra -versus-host-disease o the GI tract or gluconatecontaining IV uids (e.g., plasmalyte)
Fungal markers are sent on the above patient and return with a positive

glucan. You are concerned or candidemia/invasive candidiasis. What are


the next steps?
Additional diagnostic testing
Blood cultures (i not already sent) or isolate susceptibility testing
Biopsy localized sites o involvement (e.g., skin)
Empiric antibiotics2
I non-neutropenic and stable uconazole
I neutropenic and/or unstable echinocandin (e.g., caspo ungin),
amphotericin B ormulation or voriconazole

REFERENCES
1. Ross JJ. Chapter 192. Candida and aspergillus. In: McKean SC, Ross JJ, Dressler DD, Brotman DJ, Ginsberg JS, eds. Principles and
Practice of Hospital Medicine. New York, NY: McGraw-Hill; 2012.
2. Pappas PG, Kau man CA, Andes D, et al.; In ectious Diseases Society o America. Clinical practice guidelines or the management
o candidiasis: 2009 update by the In ectious Diseases Society o America. Clin Infect Dis. 2009;48(5):503535.

46

CLASSIFICATION

INFECTIOUS DISEASE

Hepatitis C, Acute vs. Chronic

Acute

Chronic

Anti-HCV
ALT

HCV RNA (P CR)

Months

Ye a rs
Time a fte r e xpos ure

(Friedman LS. Chapter 16. Liver, Biliary Tract, & Pancreas Disorders. In: Papadakis MA, McPhee SJ, Rabow
MW. eds. CURRENT Medical Diagnosis &Treatment 2014. New York, NY: McGraw-Hill; 2014.)

Figure 5-7. Laboratory pro le o acute vs. chronic hepatitis C.

47

INFECTIOUS DISEASE

CLASSIFICATION

Hepatitis C, Acute vs. Chronic


Case: A 58-year-old healthy man with distant IV drug use asks about
hepatitis C screening.
What are the means o transmission o hepatitis C?1

Percutaneous (IV drug use [majority o cases], tattoos, body piercing)


Blood/clotting actor trans usion (prior to HCV donor screening ~ pre-1992)
Blood exposure (hemodialysis, healthcare workers)
Sexual and perinatal transmission are possible but relatively inef cient
compared to the other risk actors listed above
Who should be screened or chronic hepatitis C?
USPS F recommends screening (anti-HCV antibody; i positive
con rmatory HCV RNA) in high-risk populations (current or past IVDU,
sex with an IVDU, blood trans usion prior to 1992), or one-time in adults
born between 1945 and 1965
How does acute vs. chronic hepatitis C di er in terms o symptoms and

lab results?2
Acute (within 6 months o hepatitis C exposure)
Symptoms: majority are asymptomatic (84% o cases); minority may
experience atigue, jaundice, ever, nausea/vomiting, or RUQ pain
Labs: elevation in aminotrans erases (o en 1020 normal)
HCV RNA turns positive within 18 weeks a er exposure
Anti-HCV antibody turns positive within 26 months a er exposure
Chronic (develops in 85% o patients with acute hepatitis C)
Symptoms: also generally asymptomatic, but ~20% progress to cirrhosis
and 15% may have extra-hepatic mani estations (e.g., cryoglobulinemia)
Labs: uctuation o aminotrans erases (may be normal depending on
timing); anti-HCV antibody and HCV RNA both detected
What does the lab pro le o a positive anti-HCV but negative HCV RNA

suggest?2
Past exposure to HCV but spontaneous recovery (~15% o acute hepatitis C
cases) or alse positive anti-HCV (e.g., patients with rheumatoid actor)
What are the various genotypes o hepatitis C?3
otal o six di erent genotypes (testing should be sent a er diagnosis is
con rmed)
Genotype 1 is the most common (70% o HCV in ections in US)
Genotypes 2 and 3 account or the majority o remaining HCV in ections

REFERENCES
1. Friedman LS. Chapter 16. Liver, biliary tract, & pancreas disorders. In: Papadakis MA, McPhee SJ, Rabow MW, eds. CURRENT
Medical Diagnosis & Treatment 2014. New York, NY: McGraw-Hill; 2014.
2. Ruther ord A, Dienstag JL. Chapter 39. Viral hepatitis. In: Greenberger NJ, Blumberg RS, Burako R, eds. CURRENT Diagnosis &
Treatment: Gastroenterology, Hepatology, & Endoscopy. 2nd ed. New York, NY: McGraw-Hill; 2012.
3. Dienstag JL. Chapter 304. Acute viral hepatitis. In: Longo DL, Fauci AS, Kasper DL, Hauser SL, Jameson J, Loscalzo J, eds. Harrisons
Principles of Internal Medicine. 18th ed. New York, NY: McGraw-Hill; 2012.

47

PHYSICAL EXAM

INFECTIOUS DISEASE

Meningitis

A Ke rnig s ign

Involunta ry hip a nd
kne e fle xion

B Brudzins ki s ign

Physical Exam Finding

Sensitivity (Rule -Out)

Classic Triad Components1


Fever

85%

Neck stif ness

70%

Altered mental status

67%

Absence o all 3 ( ever, neck stif ness,


AND altered mental status)

99100%

Specif c Exam Maneuvers24


Kernig sign

5%

Brudzinski sign

5%

Jolt accentuation

2197%

(GreenbergDA,Amino MJ,SimonRP.Chapter1.NeurologicHistory&Examination.In:GreenbergDA,
Aminof MJ, Simon RP. eds. Clinical Neurology, 8e. New York, NY: McGraw-Hill; 2012.)

48

Figure 5-8. The Kernig and Brudzinski signs (above); the utility o
physicalexam ndingsinrulingoutacutemeningitis(below).

INFECTIOUS DISEASE

PHYSICAL EXAM

Meningitis
Case: A 20-year-old male college student with no past medical history presents
with ever and con usion. You are concerned or acute bacterial meningitis.
What is the classic presentation o acute meningitis?1

riad o ever, altered mental status, and neck sti ness (although only seen
in less than 2/3 o patients with meningitis)
How might the exam help in the evaluation o a patient with suspected
meningitis?
Primarily use ul to rule-out potential disease and avoid unnecessary lumbar
puncture (LP)
Speci city o exam is less use ul because will still need LP to con rm dx
What physical exam maneuvers can be used to diagnose meningitis?1
Kernig sign: passively ex the hip to 90 and bring the knee to exion at 90
and then passively extend the knee. I pain in hamstrings + resistance to
extension positive test
Brudzinski sign: passively ex the neck
Corresponding involuntary hip and knee exion positive test
Jolt accentuation: patient rotates head horizontally at requency o
23 rotations/sec
I baseline headache worsens positive test
What aspects o the physical exam are most help ul or ruling out

meningitis?14
Absence o all 3 components o triad ( ever, altered mental status, AND
neck sti ness) is highly sensitive
Fever (85%), neck sti ness (70%), and altered mental status (67%)
individually have relatively low sensitivities or ruling-out meningitis
Speci c exam maneuvers have very poor sensitivities
Kernig (5%), Brudzinski (5%), and jolt (initial study reported 97%
sensitivity, but recent data with larger sample population showed a
sensitivity o only 21%)
You proceed with obtaining a lumber puncture or the 20-year-old college stu-

dent. What empiric antibiotics should be started while awaiting the results?5,6
Vancomycin + a third-generation cephalosporin (e.g., ce riaxone or ce otaxime)
2004 IDSA guidelines also recommend the addition o IV dexamethasone
in adults with suspected or proven pneumococcal meningitis

REFERENCES
1. Attia J, Hatala R, Cook DJ, Wong JG. Does this adult patient have acute meningitis?. JAMA. 1999;282(2):175181.
2. T omas KE, Hasbun R, Jekel J, Quagliarello VJ. T e diagnostic accuracy o Kernigs sign, Brudzinskis sign, and nuchal rigidity in
adults with suspected meningitis. Clin Infect Dis. 2002;35(1):4652.
3. Uchihara , sukagoshi H. Jolt accentuation o headache: the most sensitive sign o CSF pleocytosis. Headache. 1991;31(3):167171.
4. Nakao JH, Ja ri FN, Shah K, Newman DH. Jolt accentuation o headache and other clinical signs: poor predictors o meningitis in
adults. Am J Emerg Med. 2014;32(1):2428.
5. unkel AR, Hartman BJ, Kaplan SL, et al. Practice guidelines or the management o bacterial meningitis. Clin Infect Dis. 2004;
39(9):12671284.
6. unkel AR, Hartman BJ, Kaplan SL, et al. Practice guidelines or the management o bacterial meningitis. Clin Infect Dis. 2004;
39(9):12671284.

48

INFECTIOUS DISEASE

PHYSICAL EXAM

Osteomyelitis and DiabeticUlcers

Test

Positive Likelihood
Ratio (95% CI)

Negative Likelihood
Ratio (95% CI)

Exam Findings
Ulcer area > 2 cm 2

7.2 (1.149)

0.48 (0.310.76)

Positive probe-to-bone
test

6.4 (3.611)

0.39 (0.200.76)

Boneexposure

9.2 (0.57146)a

0.70 (0.530.92)

11.0 (1.679)

0.34 (0.061.90)a

Laboratory Tests
ESR > 70 mm/hr

Not statistically signi cant.


(Data rom Butalia S, Palda VA, Sargeant RJ, Detsky AS, Mourad O. Does this patient with diabetes have
osteomyelitisofthelowerextremity?JAMA. 2008;299(7):806813.)

Figure 5-9.Utilityofexam ndingsandlaboratorytestingin


diagnosing osteomyelitis in the presence o a diabetic ulcer.

49

INFECTIOUS DISEASE

PHYSICAL EXAM

Osteomyelitis and DiabeticUlcers


Case: A 58-year-old man with long-standing type II diabetes mellitus is
noted to have a 3 2 cm ulcer on the sole o his oot with surrounding
erythema.
What are the di erent types o lower extremity ulcers and their typical

eatures?1
Venous: typically above the medial malleoli with irregular borders
Arterial: toes/shins with pale borders, punched-out appearance, and very
pain ul
Diabetic: sites o increased pressure (e.g., sole o oot or heel)
Which physical exam ndings and laboratory tests are most use ul or

diagnosing (ruling-in) vs. excluding (ruling-out) osteomyelitis in patients


with a diabetic ulcer?2
Exam ndings:
Rule-in (high positive likelihood ratio [LR])
Ulcer area > 2 cm 2 (positive LR = 7.2)
Positive probe-to-bone test (positive LR = 6.4)
Bone exposure (positive LR = 9.2, but not statistically signi cant)
Rule-out (low negative LR)
Ulcer area < 2 cm 2 (negative LR = 0.48)
Negative probe-to-bone test (negative LR = 0.39)
Not use ul: presence/absence o ulcer in ammation
Laboratory tests:
Rule-in (high positive LR)
ESR > 70 mm/hr (positive LR = 11)
Rule-out (low negative LR)
ESR < 70 mm/hr (negative LR = 0.34, but not statistically signi cant)
Not use ul: swab culture or presence/absence o leukocytosis
I the physical exam and laboratory testing are concerning or osteomyelitis,

what tests would you order next?2


Imaging: plain lm and/or MRI
Plain lm (positive LR = 2.3, negative LR = 0.63)
MRI (positive LR = 3.8, negative LR = 0.14)
Negative MRI makes osteomyelitis much less likely

REFERENCES
1. Section 17. Skin signs o vascular insuf ciency. In: Wol K, Johnson R, Saavedra AP, eds. Fitzpatricks Color Atlas and Synopsis of
Clinical Dermatology. 7th ed. New York, NY: McGraw-Hill; 2013.
2. Butalia S, Palda VA, Sargeant RJ, Detsky AS, Mourad O. Does this patient with diabetes have osteomyelitis o the lower extremity?
JAMA. 2008;299(7):806813.

49

INFECTIOUS DISEASE

DIAGNOSTIC APPROACH

Tuberculosis (Latent) Infection

Tuberculosis Skin
Test (TST)

Inter eron-Gamma
Release Assay (IGRA)

Type o testing

Skin

Whole blood

Test detection

Delayed-hypersensitivity
reaction to tuberculin
protein derivative (PPD)

T-cell release o IFN- in


responsetoexposureto
highly speci c TB antigens

Positive result

Skin induration

IFN- detected

Sensitivity1

77%

90%a

Specif city1

97%b

93%a

Advantages

Inexpensive
Preferredtestif< 5 years
old

Noboostingphenomenon
Singlebloodtestand
objective result within
24 hours
Betterspeci city(than
TST) in recently BCGvaccinated individuals

Disadvantages

alse positive rate i


recent BCG vaccination,
non-TB mycobacterial
in ection, or boosting
phenomenon
Requiresmultiplevisits
and subjective result

Expensive
Requiresblooddraw

Test characteristics or T-SPOT. TB assay


In non-BCG vaccination populations (TST alse positive rate is higher a ter BCG vaccination, although
reaction wanes a ter 10 years rom vaccination)

Figure 5-10. TST vs. IGRA testing or latent tuberculosis in ection.

50

INFECTIOUS DISEASE

DIAGNOSTIC APPROACH

Tuberculosis (Latent) Infection


Case: A 30-year-old man with newly diagnosed acute myeloid leukemia is
seen in clinic prior to stem cell transplantation. He undergoes screening
or latent TB in ection.
What are the two tests or latent TB in ection and how do they work?1,2

uberculosis skin test ( S ) with puri ed protein derivative (PPD)


PPD is injected subcutaneously and a delayed-type hypersensitivity
reaction (i.e., skin induration) occurs i there was prior in ection with B
Inter eron-gamma release assay (IGRA)
wo assays availableQuantiFERON- B Gold-In- ube and -SPO . B
Both assays are blood tests that measure -cell release o IFN- in
response to exposure to highly speci c B antigens
IFN- detected i prior B in ection
What are the advantages and disadvantages o the di erent testing modalities?
S
Advantages: low cost; pre erred in children < 5 years old
Disadvantages: high alse positive rate (i recent BCG vaccination,
non- B mycobacterial in ection, or boosting phenomenonprior PPD
tests increase reactivity); requires multiple visits; result is subjective
IGRA
Advantages: single blood test with objective result; better speci city
(vs. S ) in patients with recent BCG vaccination (prior BCG vaccination
does not cause alse positive IGRA results)
Disadvantages: expensive; requires blood draw; time-sensitive processing
What level o induration constitutes a positive TST?3
5 mm: i HIV in ection, recent contact with active B, chest x-ray with signs
o prior B, or history o organ transplant or immunosuppression
10 mm: i IV-drug user, resident/employee in high-risk setting, risk o
progression to active B, < 4 years old, or recent arrival rom B-endemic
area
15 mm: i no known risk actors or B
Can TST or IGRA testing be used to diagnose active TB?2
No: both tests have limited accuracy in diagnosing active B

REFERENCES
1. Pai M, Zwerling A, Menzies D. Systematic review: -cell-based assays or the diagnosis o latent tuberculosis in ection: an update.
Ann Intern Med. 2008;149(3):177184.
2. Raviglione MC, OBrien RJ. Chapter 165. uberculosis. In: Longo DL, Fauci AS, Kasper DL, Hauser SL, Jameson J, Loscalzo J, eds.
Harrisons Principles of Internal Medicine. 18th ed. New York, NY: McGraw-Hill; 2012.
3. Phan VD, Poponick JM. Chapter 70. uberculosis. In: intinalli JE, Stapczynski J, Ma O, Cline DM, Cydulka RK, Meckler GD, eds.
Tintinallis Emergency Medicine: A Comprehensive Study Guide. 7th ed. New York, NY: McGraw-Hill; 2011.

50

INTENSIVE CARE

TREATMENT APPROACH

Acute RespiratoryDistress Syndrome

Goa ls
Initia te
volume /pre s s ure -limite d
ve ntila tion

Oxyge na te

Tida l volume 6 mL/kg ide a l


body we ight ba s e d on he ight
P la te a u pre s s ure 30 cm H2 O
RR 35 bre a ths pe r minute
Titra te P EEP a nd FiO 2
to a chieve :
Pa O 2 >60 mmHg
S pO 2 88 95%

Diure s is
(if MAP > 65 mmHg)

PEEP = Positive end expiratory pressure.


(Data rom Caironi P, Tognoni G, Masson S. Albumin replacement in patients with severe sepsis or septic
shock. N Engl J Med 2014; 370: 1412 21.)

Figure 6-1. Initial management o ARDS.

51

INTENSIVE CARE

TREATMENT APPROACH

Acute RespiratoryDistress Syndrome


Case: A 22-year-old male presents with 3 days o cough, SOB, and ever.
His oxygen saturation is 83% on a 100% nonrebreather ace mask, and
an ABG is 7.45/35/55. You suspect acute respiratory distress syndrome
(ARDS).
How is ARDS de ned?1

According to the 2012 Berlin de nition, there are three levels o ARDS:
while measured on PEEP > 5
Mild ARDS (PaO2/FiO2 = 201300 mmHg), moderate ARDS (PaO2/FiO2
= 101200), and severe ARDS (PaO2/FiO2 < 100)
Hypoxemia must not be ully explained by cardiogenic pulmonary edema
Respiratory symptoms must have begun within 1 week o clinical insult
Bilateral opacities consistent with pulmonary edema must be present on
CXR or C , and not ully explained by pleural e usions, lobar collapse, or
pulmonary nodules
What conditions may lead to ARDS?
Pulmonary: pneumonia, chemical inhalation, near-drowning, aspiration
Nonpulmonary: trauma, pancreatitis, sepsis, burns
How should ARDS be managed?

T ere are three major components to minimizing lung injury in ARDS:


1. Ventilate with a low tidal volume (V ) strategy: ARDSNet2 compared
low V (68 mL/kg o ideal body weight to maintain a plateau pressure
o 2530 cm H 2O) with conventional V (1012 mL/kg o ideal body
weight to maintain a plateau pressure o 4550 cm H 2O). Mortality
was signi cantly lower (31% vs. 40%) in the low V strategy.
2. Oxygenate with higher PEEP to prevent alveolar collapse and to minimize FiO2. Meta-analysis o PEEP trials3 showed that patients have
improved oxygenation with higher PEEP but only patients with PaO2/
FiO2 < 200 mmHg have a mortality bene t. Esophageal pressures
may be used as a surrogate or pleural pressures to help determine an
individuals best PEEP4.
3. Manage uid status: ARDS leads to increased pulmonary vascular
permeability leading to poor intravascular oncotic pressure and increase in extravascular lung water. Maintaining normal/low le atrial
pressure minimalizes pulmonary edema and improves pulmonary
mechanics.

REFERENCES
1. ARDS De nition ask Force. Acute respiratory distress syndrome: the Berlin De nition. JAMA. 2012;307:25262533.
2. Brower RG, Matthay MA, Morris A, et al. Ventilation with lower tidal volumes as compared with traditional tidal volumes or acute
lung injury and the acute respiratory distress syndrome. New Engl J Med. 2000;342(18):13011308.
3. Briel M, Meade M, Mercat A, et al. Higher vs. lower positive end-expiratory pressure in patients with acute lung injuiry and acute
respiratoyr distress syndrome: systemic review and meta analysis. JAMA. 2010;303:865873.
4. almor D, Sarge , Malhotra A, et al. Mechanical ventilation guided by esophageal pressure in acute lung injury. N Engl J Med.
2008;359:20952104.

51

TREATMENT APPROACH

INTENSIVE CARE

Acute RespiratoryDistress Syndrome: RefractoryHypoxemia

Strategy
Ventilator strategies
Nonventilator strategies

Improves
Mortality?

Higher PEEP strategy

Yes1

Recruitment maneuvers

No 2

Neuromuscular blockade (NMB)

Yes3

Inhaled vasodilators (inhaled NO,


inhaled prostacyclins)

No 4

Prone positioning

Yes5

ECMO

Unclear6

(Data rom Caironi P, Tognoni G, Masson S. Albumin replacement in patients with severe sepsis or septic shock.
N Engl J Med 2014; 370: 1412 21.)

Figure 6-2. Ventilator based and nonventilator based strategies to


improve oxygenation.

52

INTENSIVE CARE

TREATMENT APPROACH

Acute RespiratoryDistress Syndrome: RefractoryHypoxemia


Case: A 34-year-old patient is treated or ARDS a ter near drowning.
Despite low tidal volume ventilation, he remains hypoxemic.
What approaches can be used to improve oxygenation in ARDS?

Ventilator based:
Increasing FiO2 and PEEP
Recruitment maneuvers: Opens atelectatic alveoli via 1030 second bursts
o elevated pressure (e.g., 3040 cm H 2O)
Nonventilator based:
Neuromuscular blockade: improves patientventilator asynchrony by
decreasing resistance o chest wall and diaphragm
Inhaled pulmonary vasodilators (nitric oxide and prostacyclins): improves
per usion primarily to ventilated alveoli to improve V-Q match
Prone positioning: recruits alveoli in dependent areas to improve V-Q match
ECMO: blood is circulated by a mechanical pump through a membrane
oxygenator and returned to the patient
Which o these strategies improve mortality?

Increasing FiO2 and PEEP: mortality bene t seen in higher PEEP strategy
vs. lower PEEP strategy in ARDS patients without causing higher rates o
barotrauma (adjusted RR, 0.90; 95% CI, 0.811.00)1
Recruitment maneuvers: no di erence in 28-day mortality2
Neuromuscular blockade: mortality bene t seen with cisatracurium at
28 days (p = 0.05), more ventilator ree days (p = 0.04), and ewer pneumothoraces (p = 0.01) without di erence in ICU-acquired paresis3
Inhaled pulmonary vasodilators: no mortality bene t has been shown 4
Prone positioning: mortality bene t in ARDS (HR 0.39, 95% CI 0.250.63).5
Recommended only in centers experienced in the use o prone-positioning
ECMO: CESAR trial showed reduced death or disability at 6 months
(RR 0.69, 95% CI 0.050.97) when patients were trans erred to a acility
with the capacity to provide ECMO (although ECMO was not always
provided)6

REFERENCES
1. Briel M Meade M, Mercat A, et al. Higher vs. lower positive end expiratory pressure in patients with acute lung injury and acute
respiratory distress syndrome: systematic review and meta-analysis. JAMA. 2010;303:865873.
2. Hodgson C, Keating JL, Holland AE, et al. Recruitment manoeuvres or adults with acute lung injury receiving mechanical ventilation. Cochrane Database Syst Rev. 2009; 3:CD 006667.
3. Papazian L, Forel JM, Gacouin A, et al. Neuromuscular blockers in early acute respiratory distress syndrome. N Engl J Med.
2010;363:11071116.
4. Ahikari NK, Burns KE, Friedrich JO, et al. E ect o nitric oxide on oxygenation and mortality in acute lung injury: systematic
review and meta-analysis. BMJ. 2007;334:779.
5. Guerin C, Reignier J, Richard JC, et al. Prone positioning in severe acute respiratory distress syndrome. N Engl J Med.
2013;368:21592168.
6. Peek GJ, Mug ord M, iruvoipati R, et al. Ef cacy and economic assessment o conventional ventilator support versus extracorporeal membrane oxygenation or severe adult respiratory ailure (CESAR): a multicenter randomised controlled trial. Lancet.
2009;374:13511363.

52

EVIDENCE-BASED MEDICINE

INTENSIVE CARE

Albumin in Fluid Resuscitation

Albumin
45%
40%
35%
30%
25%
20%
15%
10%
5%
0%

P = 0.09
P = 0.10
P = 0.87
22.80%24.50%
P = 0.09

35.30%
30.70%

42.40%
39.30%

20.90% 21.10%

3.20% 2.50%
ICU morta lity

Hos pita l
morta lity

28 da y
morta lity

Albumin
60%

P = 0.72

S a line

28 da y morta lity 28 da y morta lity


in s e ve re s e ps is in ARDS group
group

S a line

P = 0.85*

52.70% 53.30%

50%
40%
30% 29.80%

30%
20%

13.90% 13.50%

10%

2.60% 2.80%

0%
No orga n
fa ilure

1 orga n
fa ilure

2 orga n
fa ilure

3 orga n
fa ilure

0.70%

0.60%

4 orga n
fa ilure

0.10% 0%

5 orga n
fa ilure

*P-value comparing the numbers o patients with no organ ailure or organ ailure o one, two, three, our
or ve organs in the albumin vs. saline group.
(Longo DL, Fauci AS, Kasper DL, et al. Harrisons Principles o Internal Medicine, 18E. New York,
NY: McGraw Hill; 2012.)

Figure 6-3. Mortality rates (above) and organ ailure (below) in the
SAFE trial.

53

INTENSIVE CARE

EVIDENCE-BASED MEDICINE

Albumin in Fluid Resuscitation


Case: A 78-year-old man with sepsis presents with a low mean arterial
pressure. You consider whether to begin resuscitation with saline or
albumin.
What is the goal o early uid resuscitation in shock?

Shock leads to hypoper usion o organs that can lead to ischemic injury and
multiorgan system ailure
Resuscitation is necessary to improve oxygen delivery
What can be given or uid resuscitation?1
1. Crystalloids: saline solutions, bu ered solutions (e.g., Lactated Ringers)
Inexpensive, widely available, but may not remain in the vasculature;
normal saline can cause a nonanion gap metabolic acidosis
2. Colloids: suspensions o molecules with a carrier solution (e.g., albumin,
starches)
T eoretically may expand intravascular volume better than crystalloids
because they are retained in the intravascular space and maintain oncotic
pressure
Is albumin better than normal saline or resuscitation?2

No. T e SAFE rial compared the e ects o giving critically ill patients
either 4% albumin or normal saline or intravascular uid resuscitation
Patient in the albumin arm received less uid overall, but had no di erences in mean arterial pressures
T ere was no signi cant di erence in ICU, hospital, or 28-day mortality.
Among patients with severe sepsis, there was a nonsigni cant trend
toward reduction in 28-day mortality with albumin (see image)
T ere was also no signi cant di erence in rates o organ ailure (see image)
I a person with severe sepsis has a low albumin, does albumin replacement

improve mortality?
No. T e ALBIOS rial compared crystalloid alone with crystalloid plus
albumin given to a target serum albumin o 30 g/L in patients with severe
sepsis or septic shock3
T ere was no di erence in survival at 28 or 90 days

REFERENCES
1. Myburgh JA, Mythen MG. Critical care medicine: resuscitation uids. N Engl J Med. 2013;369:12431251.
2. T e SAFE Study Investigators. A comparison o albumin and saline or uid resuscitation in the intensive care unit. N Engl J Med.
2004;350:22472256.
3. Caironi P, ognoni G, Masson S. Albumin replacement in patients with severe sepsis or septic shock. N Engl J Med. 2014;370:
14121421.

53

cl

54

ch

INTENSIVE CARE
TREATMENT APPROACH

Mechanical Ventilation

INTENSIVE CARE

TREATMENT APPROACH

Mechanical Ventilation
Case: A 43-year-old male is admitted with severe respiratory ailure.
Mechanical ventilation is initiated.
What are the major reasons to initiate mechanical ventilation?

Improve oxygenation, improve ventilation, or decrease work o breathing


Intubation is requently used as airway protection, though this is an
imper ect protection
What are the basic parameters that can be adjusted on a ventilator?
FiO2: titrate 21100% to improve oxygenation. High FiO2 may O2 toxicity
Positive end-expiratory pressure (PEEP): titrate rom 5 to 20 cm H 2O to
minimize expiratory alveolar collapse, improve oxygenation. Higher PEEP
may alveolar injury and decrease cardiac preload, but PEEP may be
indicated in ARDS
Inspiratory pressure: titrate in pressure-preset modes to change patients tidal
volume and improve CO2 ventilation. Higher pressures may alveolar injury.
idal volume (V ): titrate in volume-preset modes to improve CO2 ventilation. Higher V may elevated peak inspiratory pressure (PIP) and plateau
pressures (see Peak Inspiratory Pressures card)
Respiratory rate (RR): titrate to improve CO2 ventilation; at high respiratory
rates, breaths may begin to stack (inspiration started be ore expiration
complete) higher pressures
What two major ways ventilator be set to deliver a breath, and how are the

most common ventilator modes designed?


Breaths delivered can be measured by volume delivered or pressure
delivered
I the volume is set, the pressure becomes the dependent variable
(a ected by airway resistance, lung/chest wall compliance)
I the pressure is set, the volume becomes the dependent variable
See image or characteristics o most common ventilator modes
How do you choose a mode o ventilation?

Most commonly used mode may be institution-dependent


Use an assist-control mode (volume-cycled or pressure-cycled) or greater
control over ventilation. Use pressure-support ventilation or patients ready
to breathe spontaneously entirely on their own.

54

55

E R US S E RP YA WRI A

ve

xh

vo

ve

xp

<

<

ve

<

INTENSIVE CARE
DIAGNOSTIC APPROACH

Mechanical Ventilation: PeakInspiratoryPressures

INTENSIVE CARE

DIAGNOSTIC APPROACH

Mechanical Ventilation: PeakInspiratoryPressures


Case: A 23-year-old male is intubated or respiratory ailure. The respiratory
therapist notes a peak inspiratory pressure (PIP) o 55.
What two properties o the mechanically ventilated patient contribute to PIPs?

Resistance: the pressure due to the opposition to ow caused by the orces o


riction, and is determined by the dimension o the airway, viscosity o the
gas, and pattern o air ow
Compliance: the ability o the lung to stretch to accommodate more volume
What causes an elevated airway resistance?
Problems that inhibit the movement o air into the alveoli: secretions,
bronchospasm, kink in endotracheal tubing
What causes decreased pulmonary compliance?
Problems that inhibit the expansion o alveoli to its ull volume: pus or
pulmonary uid that reduce sur actant unction, pneumothorax, atelectasis,
abdominal distention, one-sided bronchus intubation, large pleural e usion
How do you identi y the relative contribution o resistance and

compliance to high PIPs?13


On a patient that is ully supported on volume-control ventilation with a
constant ow rate, measure a plateau pressure by pausing the respiratory
cycle at the end o inspiration (an end-inspiratory hold) be ore
expiration is allowed to proceed.
As air ow has ceased, the component o pressure caused by airway
resistance (Presistance) is removed and any resulting pressure is thus caused
by decreased compliance only (Pcompliance)
Compliance = V /(Pplateau PEEP)

Resistance = (PIP Pplateau)/V, where V is the inspiratory ow rate


For ease o calculation, set a constant ow rate o 1 L/sec
T is is derived rom Ohms law, where V = IR, or R = V/I
Goals or mechanically ventilated patients:
PIP < 40 cm H 2O, Pplateau < 30 cm H 2O, airway resistance < 10 cm
H 2O/L/sec, respiratory compliance: 50100 mL/cm H 2O
T ese measurements can only be per ormed on patients who are not
actively breathing because contraction o respiratory system muscles can
contribute to total compliance

REFERENCES
1. Marino Paul M. T e ICU Book. 4th ed. Philadelphia, PA: Wolters Kluwer; 2014.
2. Kreit JW. Mechanical Ventilation. New York, NY: Ox ord University Press; 2013.
3. Hess DR, Kacmarek RM. Essentials of Mechanical Ventilation. New York, NY: McGraw Hill Education Medical; 2014.

55

56

INTENSIVE CARE
TREATMENT APPROACH

Oxygen Supplementation

INTENSIVE CARE

TREATMENT APPROACH

Oxygen Supplementation
Case: A 78-year-old emale is admitted with pneumonia. O2 saturation is
86% on room air.
What is rst-line oxygen therapy and what are its advantages/disadvantages?

Nasal cannula: 16 L/min 1,2


Provides lowest dose o oxygen least oxygen toxicity
Poses less delirium and aspiration risk over ace mask
Oral intake is not limited by nasal cannula
However, the e ective FiO2 is extremely variable based on the amount o
room air a patient also takes in, which increases with high tidal volume
and high respiratory rate; the maximum FiO2 is about 40%
In patients with normal tidal volume and respiratory rate, each L/min
increases FiO2 by ~4%
How can the e cacy o nasal cannula be augmented?
Reservoir systems (e.g., Oxymizer pendant): oxygen is stored in a reservoir
during exhalation, making that oxygen available as a bolus upon the next
inhalation. T is allows a reduced oxygen ow to provide a higher oxygen
percentage without requiring a ace mask3
What are other ways oxygen can be administered?

Simple ace mask: 612 L/min ( ow rates > ~6 LPM are needed to clear
exhaled gas rom mask to prevent rebreathing CO2)
Venturi masks: exchangeable mask adapters deliver a set FiO2 by entraining a variable amount o room air into delivered oxygen; pre erred or
patients or whom a lower FiO2 is pre erred (e.g., COPD patients)
Nonrebreather masks (NRB): nontitratable and delivers ~100% FiO2 at a
high ow rate; considered bridge to HFNC or intubation
High ow nasal cannula (HFNC): oxygen delivered at both high concentrations and high velocities ensures a high concentration o oxygen
What is the ef ect o HFNC on positive end-expiratory pressure (PEEP)?

At ow rates > 35 LPM, HFNC can provide PEEP up to 6 cm H 2O


What risks do excess oxygen pose?
FiO2 > 60% can cause oxidative in ammatory-medicated cell injury (e.g.,
ARDS)

REFERENCES
1. Bateman N , Leach RM. ABC o oxygen. Acute oxygen therapy. BMJ. 1998;317:798801.
2. Basuaye EA, Stone N, Corris PA, Gibson GJ. Variability o inspired oxygen concentration with nasal cannulas. T orax.
1992;47:609611.
3. Collard P, Wautelet F, Delwiche JP, et al. Improvement o oxygen delivery in severe hypoxaemia by reservoir cannula. Eur Respir J.
1989;2:778781.

56

EVIDENCE-BASED MEDICINE

INTENSIVE CARE

Sepsis
Patients

Multicenter US trial with 1,341 patients:


Age 18 with suspected sepsis
2 SIRS criteria
Lactate 4 mmol/L OR SBP< 90 a ter IVF challenge o
1 L over 30 min

Intervention

Randomized to one o three arms:


1. Protocol based early goal directed therapy (EGDT)
2. Protocol based standard therapy
3. Usual care (nonprotocolized targets)
All patients received early antimicrobials, low tidal volume
ventilation, moderate glycemic control, and were subjected
to conservative trans usion thresholds

Outcomes

Primary outcome
60dayin-hospitalmortality:nodi erence(p = 0.83)
Secondary outcomes
Nodi erenceindeathby90days,durationoforgansupport,
ICU or hospital length o stay, or serious adverse events

40
35

p = 0.66
p = 0.83

31.9

33.7
30.8

30
25
20

21
18.2

18.9

15
10
5
0

In hos pita l de a th by 60 da ys
De a th by 90 da ys
EGDT-like protocol
S ta nda rd the ra py protocol Us ua l ca re

SIRS = systemic inf ammatory response syndrome


(ProCESS Investigators, Yealy DM, Kellum JA, et al. A randomized trial o protocol based care or early
septic shock. N Eng J Med. 2014;370(10):16831693.)

Figure 6-7. Outcomes o sepsis managed via protocolized care vs.


57 usual care.

INTENSIVE CARE

EVIDENCE-BASED MEDICINE

Sepsis
Case: A 63-year-old emale presents with productive cough. Her vitals are
T 102.3, HR 115, BP 74/43, RR 18, and O2 saturation o 84%. Her WBC count
is 14,300 cells/mm3 .
What is SIRS? How does SIRS dif er rom sepsis, severe sepsis, and septic shock?

SIRS: wo o our eatures: (1) emperature > 100.4F or < 96.8F; (2) heart
rate > 90 bpm; (3) WBC < 4,000 cells/mm 3 or 12,000 cells/mm 3 or > 10%
immature orms; (4) respiratory rate > 20 breaths/min or PaCO2 < 32 mm Hg
Sepsis = SIRS due to known or presumed source o in ection
Severe sepsis = Sepsis + one or more organ dys unction
Septic shock = Sepsis + MAP < 65 mmHg unresponsive to uid challenge
How do you manage septic shock?
1. Source control: early administration o antibiotics increases survival1
2. Maintain organ per usion via uids and pressors:
raditionally guided using early goal-directed therapy (EGD , or Rivers
Protocol)2 in which a central line is placed and therapy is titrated to
achieve sequential goals o CVP, then MAP, and then central venous O2
saturation (ScvO2)
3. Steroids i necessary (see Steroid Use in Sepsis card)
Does EGDT actually improve outcomes?3

A single-center trial in 2001 (with above-average mortality) showed a


reduction in mortality with EGD compared to non-EGD (30.5% vs.
46.5%)2
T e 2014 ProCESS trial was a large multicenter trial that compared
outcomes o three types o management
Protocol-based EGD : Central line inserted to monitor sequential CVP,
MAP, and ScvO2 goals
Protocol-based standard therapy: Central line inserted only i access
required; uid resuscitation guided by SBP, shock index (HR:SBP ratio),
and clinical signs o per usion
Usual care: No speci ed protocolized targets
Protocol-based therapy did not result in better outcomes, also con rmed in
the ARISE and ProMISe trials.4,5 T e lack o di erence between arms may have
been due to the early recognition and treatment o sepsis or all patients.
As a result, sepsis management should ocus on aggressive source control
and adequate resuscitation up ront to maintain organ per usion rather
than on protocolized goals.

REFERENCES
1. Kumar A, Roberts D, Wood KE, et al. Duration o hypotension be ore initiation o e ective antimicrobial therapy is the critical
determinant o survival in human septic shock. Crit care Med. 2006;34:15891596.
2. Rivers E, Nguyen B, Havstad S, et al. Early goal directed therapy in the treatment o severe sepsis and septic shock. N Eng J Med.
2001;345(19):13681377.
3. ProCESS Investigators, Yealy DM, Kellum JA, et al. A randomized trial o protocol-based care or early septic shock. N Eng J Med.
2014;370(10):16831693.
4. Huang D , Angus DC, Barnato A, et al.; ProCESS/ARISE/ProMISe Methodology Writing Committee. Harmonizing international
trials o early goal-directed resuscitation or severe sepsis and septic shock: methodology o ProCESS, ARISE, and ProMISe. Intensive Care Med. 2013;39:17601775.
5. Peake SL, Delaney A, Bailey M, et al.; T e ARISE Investigators, ANZICS Clinical rials Group. Goal-directed resuscitation or
patients with early septic shock. N Eng J Med. 2014;371(16):14961506.

57

TREATMENT APPROACH

INTENSIVE CARE

Steroid Use in Sepsis

S e ve re s e ptic s hock

Fluid re s us cita tion, a ntimicrobia ls ,


a nd va s opre s s ors (if ne e de d)

S ys tolic blood pre s s ure re ma ins low


(e .g., <90 mmHg) for more tha n a n
hour de s pite a de qua te fluid
re s us cita tion a nd ma xima l
va s opre s s or a dminis tra tion

YES

Initia te e mpiric s teroids :


hydrocortis one 50 mg IV q6h or
100 mg IV q8h, us ua lly for 57 da ys

NO

Continue curre nt ma na ge me nt

Figure 6-8. Indications or use o steroids in sepsis.

58

INTENSIVE CARE

TREATMENT APPROACH

Steroid Use in Sepsis


Case: A 78-year-old male is admitted to the ICU with urosepsis on maximum
doses o three vasopressors. Despite having received 12L NS, his systolic
blood pressure remains 80. You wonder whether to administer steroids.
How are cortisol levels normally af ected during critical illness?

T e hypothalamic-pituitary-adrenal (HPA) axis is activated loss o diurnal variation increased production o cortisol, reduced cortisol breakdown, reduced renal clearance due to reduced renal unction and reduced
cortisol binding globulin increased cortisol as high as 4050 g/dL13
How should critically ill patients be assessed or adrenal insu ciency?

See Adrenal Insuf ciency: Diagnostic Approach or discussion on assessing adrenal insuf ciency in noncritically ill and limits o cortisol assays
In critically ill patients, a random cortisol < 25 g/dL may suggest adrenal
insuf ciency. However, current recommendations are to treat empirically
in patients with persistently low blood pressure a er uids and maximal
uids and vasopressors rather than per orm laboratory assessment. T is
is because:
otal cortisol may less accurately re ect active cortisol as cortisol binding globulin levels may uctuate widely.4 Serum cortisol levels may also
correlate only moderately with tissue cortisol levels.5
High-dose AC H stimulation test (250 g cosyntropin) is inadequately
sensitive to predict steroid responsiveness.6,7
Which patients may bene t rom empiric treatment with steroids?

wo trials address the idea o using low-dose (physiologic) steroids (e.g.,


hydrocortisone 50 mg IV q6h) in severely septic patients: the French rial
and the COR ICUS trial7,8
aken together, these trials suggest that steroids may decrease time to
vasopressor weaning in all patients, but may only have a mortality bene t
in patients with severe septic shock (de ned as systolic blood pressure
< 90 mm Hg or more than an hour despite adequate uid resuscitation and
vasopressor administration)

REFERENCES
1. Lamberts SW, Bruining HA, de Jong FH. Corticosteroid therapy in severe illness. N Engl J Med. 1997;337:12851292.
2. Shenker Y, Skatrud JB. Adrenal insuf ciency in critically ill patients. Am J Respir Crit Care Med. 2001;163:15201523.
3. Cooper MS, Stewart PM. Corticosteroid insuf ciency in acutely ill patients. N Engl J Med. 2003;348:727734.
4. Hamrahian AH, Oseni S, Ara ah BM. Measurements o serum ree cortisol in critically ill patients. N Engl J Med. 2004;350:
16291638.
5. Vassiliadi DA, Ilias I, xanela M, et al. Interstital cortisol obtained by microdialysis in mechanically ventilated septic patients:
correlations with total and ree serum cortisol. J Crit Care. 2013;28:158165.
6. Marik PE, Zaloga GP. Adrenal insuf ciency during septic shock. Crit Care Med. 2003;31:141145.
7. Sprung CL, Annane D, Keh D, et al.; COR ICUS Study Group. Hydrocortisone therapy or patients with septic shock. N Engl J Med.
2008;358:111124.
8. Annane D, Sebille V, Charpentier C, et al. E ect o treatment with low doses o hydrocortisone and udrocortisone on mortality in
patients with septic shock. JAMA. 2002;288:862871.

58

EVIDENCE-BASED MEDICINE

INTENSIVE CARE

TransfusionThresholds in the ICU

30%

P = 0.11

25%
20%

P = 0.05

P = 0.23

28.1%

26.5%

23.3%

22.7%

22.2%

P = 0.29

18.7%

16.2%
15%

13.4%

10%
5%
0%

30 da y morta lity 60 da y morta lity

ICU morta lity

Re s trictive Tra ns fus ion S tra te gy

Hos pita l morta lity

Libe ra l Tra ns fus ion S tra te gy

Primary Outc o me s
P = 0.12
60%

54.3%
49.0%

50%
40%

P = 0.22

P <0.01

29.0%
25.4%

30%
20%

P = 0.38

21.0%
13.2%

10.0%11.9%

10%

P = 1.00
2.4% 2.4%

0%

Ca rdia c

P ulmona ry

Infe ctious

Re s trictive Tra ns fus ion S tra te gy

He ma tologic

Any
complica tion

Libe ra l Tra ns fus ion S tra te gy

Co mplic atio ns o c c uring in e ac h g ro up


(Hbert PC, Wells G, Blajchman MA, et al. A multicenter, randomized, controlled clinical trial o trans usion
requirements in critical care. Trans usion Requirements in Critical Care Investigators, Canadian Critical Care
Trials Group. N Engl J Med. 1999;340(13):1056.)

Figure 6-9. Comparison o outcomes o restrictive vs. liberal


trans usion strategies in ICU non bleeding patients.
59

INTENSIVE CARE

EVIDENCE-BASED MEDICINE

TransfusionThresholds in the ICU


Case: A 73-year-old male is admitted to the ICU with urosepsis. His
hemoglobin is 8.2 g/dL on admission. You consider whether to give a
blood trans usion.
What are potential complications o blood trans usions?

Allergic and immune trans usion reactions, volume overload, and in ection
are most well-known (see card on rans usion Reactions). Reversal o
protective physiologic responses to blood loss may also occur (e.g.,
vasoconstriction) (see card on rans usions or GI bleed).
What should the trans usion threshold be in most ICU patients?

RICC rial, 19991: 838 euvolemic patients with hemoglobin < 9 g/dL in
the rst 72 hours o admission; excluded patients with chronic anemia or
active bleeding; randomized to either a restrictive (goal hemoglobin
> 7 g/dL) or a liberal (goal hemoglobin > 10 g/dL) strategy
No di erence in 30-day, 60-day, ICU, or hospital mortality
Cardiac events (e.g., pulmonary edema and MI) were more common
with the liberal trans usion strategy, otherwise no di erence in other
complications
In subgroup analysis, patients with lower severity o illness (APACHE II
< 20) or younger age (< 55) had decreased rates o in-hospital mortality
with restrictive strategy
rials in other non-bleeding patient populations have shown similar
results (e.g., critically ill pediatric patients,2 patients with cardiovascular
risk actors undergoing hip surgery,3 and patients undergoing cardiac
surgery4,5 )
How would concurrent acute coronary syndrome change management?

T e trans usion threshold or this group is still debated:


In the RICC trial and FOCUS trial, patients with cardiac disease treated
with a restrictive trans usion threshold did not are worse
However, a small randomized trial o 110 acute coronary syndrome patients ound that a trans usion threshold o hemoglobin > 10 g/dL (liberal
strategy) was associated with a lower risk o death, myocardial in arction,
or unscheduled revascularization (10.9% vs. 25.5%, p = 0.054) compared
to a trans usion threshold o hemoglobin > 8 g/dL (restrictive strategy)6

REFERENCES
1. Hbert PC, Wells G, Blajchman MA, et al. A multicenter, randomized, controlled clinical trial o trans usion requirements in critical
care. rans usion Requirements in Critical Care Investigators, Canadian Critical Care rials Group. N Engl J Med. 1999;340:1056.
2. Lacroix J, Hebert PC, Hutchison JS, et al.; RIPICU Investigators. rans usion strategies or patients in pediatric intensive care
units. N Engl J Med. 2007;356:16091619.
3. Carson JL, errin ML, Noveck H, et al. FOCUS Investigators. Liberal or restrictive trans usion in high-risk patients a er hip surgery. N Engl J Med. 2011;365:24532462.
4. Bracey AW, Radovancevic R, Riggs SA, et al. Lowering the hemoglobin threshold or trans usion in coronary artery bypass procedures: e ect on patient outcome. ransfuion. 1999;39:10701077.
5. Hajjar LA, Vincent JL, Galas FR, et al. rans usion requirements a er cardiac surgery: the RACS randomized controlled trial.
JAMA. 2010;304:15591567.
6. Carson JL, Brooks MM, Abbott JD, et al. Liberal versus restrictive trans usion thresholds or patients with symptomatic coronary
artery disease. Am Heart J. 2013;165:964971.

59

60

INTENSIVE CARE
TREATMENT APPROACH

Vasopressors

INTENSIVE CARE

TREATMENT APPROACH

Vasopressors
Case: A 56-year-old male presents to the ICU with sepsis. He has been
f uid resuscitated, but mean arterial pressure remains low. You consider
initiation o vasopressors.
What types o vasopressors are commonly used?

Inodilators: dobutamine
Inopressors: epinephrine, dopamine, norepinephrine
Vasopressors: phenylephrine, vasopressin
What are the sites o action o vasopressors?

Vascular walls:
Vasopressin and -1 receptors induce peripheral vasoconstriction
-2 receptors induce peripheral vasodilation; dopamine receptors induce
vasodilation in renal, splanchnic, coronary, and cerebral vasculature
T e nal e ect is determined by the sum o the combined receptor
activity
Heart: -1 receptors induce chronotropy and inotropy
What vasoactive agent is pre erred as rst-line therapy in shock?

T e SOAP II rial compared norepinephrine and dopamine or treatment


o shock1
No di erence in 28-day or 12-month mortality, although arrhythmias were
more common in the dopamine group
Dopamine increased mortality in patients with cardiogenic shock
Surviving Sepsis Guidelines now recommend norepinephrine as rst line
therapy or septic and cardiogenic shock; dopamine should be reserved or a
very select group (i.e., low risk o arrhythmias or bradycardia)
When should other agents be added?
Vasopressin: Most commonly used as a second-line agent. Intended as
a replacement or exhausted endogenous vasopressin in septic shock
(0.030.04 units/min). It can be given as a second agent to decrease the
dose o the rst agent 2
T ird line agent depends on the patients hemodynamic status:
Dopamine: use ul or inotropy in patients with a low risk o
tachyarrhythmias
Dobutamine: use ul to increase inotropy but may worsen hypotension
Phenylephrine: use ul to increase peripheral vasoconstriction alone in
patients who do not need inotropic support (e.g., neurogenic shock)

REFERENCES
1. De Backer D, Biston P, Devriendt J , et al. Comparison o dopamine and norepinephrine in the treatment o shock. N Engl J Med.
2010;362(9):779789.
2. Russell JA, Walley KR, Singer J, et al. Vasopressin versus norepinephrine in usion in patients with septic shock. N Engl J Med.
2008;358(9):877887.

60

DIAGNOSTIC APPROACH

NEPHROLOGY

AcidBase Disorders
pH <7.4
Step 1

Acide mia

Step 2

Che ck P a CO 2
a nd HCO 3

pH >7.4
Che ck pH

Alka le mia

Che ck P a CO 2
a nd HCO 3

P a CO 2

HCO 3

P a CO 2

HCO 3

Re s pira tory
a cidos is

Me ta bolic
a cidos is

Re s pira tory
a lka los is

Me ta bolic
a lka los is

Step 3

Che ck
compe ns a tion

Ina ppropria te

Cons ide r a dditiona l


dis orde r

Ele va te d

Cons ide r a nion ga p


me ta bolic a cidos is

Appropria te

Step 4

Che ck a nion ga p

Che ck de lta -de lta

Step 5

Reproduced, with permission, rom Stern SC, Ci u AS, Altkorn D. Symptom to Diagnosis: An
Evidence-Based Guide. 2nd ed. New York, NY: McGraw Hill Education; 2010. Figure 4 1.

1 Disorder

1 Abnormality

Response

Expected
Compensation
(1 Abnormality :
Response)a

Metabolic acidosis

HCO3-

PaCO2

1 : 1.2b

Metabolic alkalosis

HCO3-

PaCO2

1 : 0.7

Respiratory acidosis

PaCO2

HCO3-

10 : 1 (acute) vs.
10 : 3.5 (chronic)

Respiratory alkalosis

PaCO2

HCO3-

10 : 2 (acute) vs.
10 : 4 (chronic)

Units (mEq/L or HCO3- , mm Hg or PaCO2).


b
Alternatively, can use Winters Formula: PaCO2 = 1.5 (HCO3- ) + 8 2.
a

Figure 7-1. Diagnostic algorithms or evaluation o acid base


disorders (above) and expected compensation o acid base disorders
61 (below).

DIAGNOSTIC APPROACH

NEPHROLOGY

AcidBase Disorders
Case: A 32-year-old man with type 1 diabetes mellitus presents with
vomiting and lethargy. Initial labs are notable or a HCO3 - o 20 and an
ABG with a pH o 7.30 and PaCO2 o 37.
What are the general types o acidbase disorders?

Metabolic acidosis and alkalosis; respiratory acidosis and alkalosis


What are the steps in the evaluation o an acidbase disorder?1

1. Check the pH on the ABG to determine the primary disorder


2. Compare the HCO3- and the PaCO2 to determine i the primary process
is respiratory or metabolic
3. Check or appropriate compensation: i actual value is less than or greater
than the expected compensation an additional process exists in addition to the 1 disorder
For example, metabolic acidosis with a higher than expected PaCO2
Inappropriate respiratory compensation (i.e., respiratory acidosis is
also present)
4. Check or an anion-gap acidosis: i anion gap is elevated a metabolic
acidosis is also present (regardless o pH or HCO3- )
Anion gap = Na+ - (HCO3- + Cl- )
Correction or hypoalbuminemia = observed anion gap
+ 2.5(4-observed albumin) expressed in g/dL
5. Check the deltadelta (change in anion gap divided by change in HCO3- )
i there is an anion-gap acidosis
Delta Anion Gap
Delta HCO3-

Calculated AG - Expected AG
24 - measured HCO3-

AG / HCO3- = 12 pure anion gap metabolic acidosis


AG / HCO3- < 1 additional non-AG metabolic acidosis
AG / HCO3- > 2 additional metabolic alkalosis
Practice using the step-wise approach with the ollowing labs or the case

vignette:
1. Na+ 131, K+ 4.0, Cl- 92, HCO3- 20, BUN 56, Cre 1.0, Glu 380. Albumin 4.0
2. ABG: pH 7.30, PaCO2 37, PaO2 90
3. Answer Primary anion-gap metabolic acidosis with appropriate respiratory compensation and coexisting metabolic alkalosis (delta-delta > 2).

REFERENCE
1. Chapter 4. I have a patient with an acid-base abnormality. How do I determine the cause? In: Stern SC, Ci u AS, Altkorn D, eds.
Symptom to Diagnosis: An Evidence-Based Guide. 2nd ed. New York, NY: McGraw-Hill Education; 2010.

61

CLASSIFICATION

NEPHROLOGY

Acid-Base Disorders: Non-Gap MetabolicAcidosis

De cre a s e d
s e rum HCO 3

Impa ire d re na l
e xcre tion of a cid

Re na l
los s e s

Gl
los s e s

Dilutiona l
e ffe ct

Type II RTA
Ace ta zola mide
P os thype rve ntila tion

Dia rrhe a
P a ncre a tic
fis tula
Ure te ra l
dive rs ion

S a line IV
a dminis tra tion

Type I RTA
Type IV RTA
Adre na l
ins ufficie ncy

Incre a s e d
a cid inta ke

TP N
HCl
a dminis tra tion

Figure 7-2. The di erent causes o a non gap metabolic acidosis.

62

CLASSIFICATION

NEPHROLOGY

Acid-Base Disorders: Non-Gap MetabolicAcidosis


Case: A 62-year-old emale with a history o breast cancer is admitted with
hypercalcemia. She is administered IV normal saline and develops a low
serum HCO3 - with a normal anion gap.
How does a non-gap metabolic acidosis develop?

1. Decreased serum bicarbonate levels (via renal loss, GI loss, or dilutional e ect)
2. Impaired excretion o acid by the kidneys
3. Increased acid intake
What are the possible etiologies o a non-gap metabolic acidosis?1

Cause

Mechanism

Hyperalimentation ( TPN) Amino acids metabolized by liver into acid


HCl administration (IV)

Increased acid intake (rare treatment o metabolic


alkalosis)

Acetazolamide

HCO3- resorption in the proximal tubule

Adrenal insu ciency

mineralocorticoid activity renal acid


secretion (like type IV RTA)

Renal tubular acidosis

Proximal (type II): HCO3- resorption in the


proximal tubule
Distal (type I or IV): acid secretion in the collecting
duct

Diarrhea

Gastrointestinal losses o HCO3-

Ureteral diversion (e.g.,


ureterosigmoidostomy)

Colon has anion exchanger that absorbs chloride


( rom diverted urine) and secretes HCO3- (into
diverted urine)

Pancreatic stula

HCO3 rich f uid o pancreatic enzymes is lost in


the GI tract

Post hyperventilation

Respiratory alkalosis kidneys compensate by


wasting HCO3- respiratory alkalosis resolves
HCO3- until regenerated

Saline administration
(IV)

Serum HCO3- concentration is diluted by in usion


o large amounts o IV f uid that lack HCO3-

REFERENCE
1. Galla JH, Kurtz I, Kraut JA, Lipschik GY, Macrae JP. Chapter 5. Acidbase disorders. In: Lerma EV, Berns JS, Nissenson AR, eds.
CURREN Diagnosis & reatment: Nephrology & Hypertension. New York, NY: McGraw-Hill; 2009.

62

DIAGNOSTIC APPROACH

NEPHROLOGY

Acid-Base Disorders: Renal Tubular Acidosis

Norma l a nion ga p me ta bolic a cidos is


Urine Anion Ga p = [Na +] + [K+] [Cl-]

Appro priate re nal re s po ns e :


no RTA

Type 1
pH >6
+
s e rum K
Type 2
pH <5.5
+
s e rum K

Type 4
+
K

RTA

S e rum [K+]

De cre a s e d

Incre a s e d

Urine pH

<5.5

>6

Pro ximal (Type 2) RTA

Dis tal (Type 1) RTA

Hypo aldo s te ro ne
(Type 4) RTA

Figure 7-3. Diagnostic algorithm or renal tubular acidosis.

63

NEPHROLOGY

DIAGNOSTIC APPROACH

Acid-Base Disorders: Renal Tubular Acidosis


Case: A 63-year-old male on i os amide is ound to have a non-anion gap
metabolic acidosis. You consider whether this is due to a renal tubular
acidosis (RTA).
What is the appropriate kidney response to acidemia?

Increase excretion o acid in the orm o ammonium chloride (NH 4+ Cl- )


How is an RTA diagnosed?13
Calculate a urine anion gap (UAG) = [Na+ ] + [K+ ] [Cl- ]. NH 4+ and
Cl- should be present in higher than usual amounts i kidney is working
normally to excrete NH 4+
i. Negative UAG: urine Cl- appropriately elevated to match (unmeasured)
NH 4+ excretion to maintain electrical neutrality no R A
ii. Positive UAG: inadequate NH 4+ excretion R A present
How do you di erentiate Type 1, Type 2, and Type 4 RTAs?13

1. Check serum potassium:


(a) High potassium = ype 4 (distal/ hypoaldosterone) R A: low
aldosterone inability o distal tubule to produce NH 4+ and excrete
potassium. May be due to drugs (e.g., K+ -sparing diuretics, NSAIDs,
or diabetes mellitus) in addition to hypoaldosteronism
2. Check urine pH:
(a) Urine pH < 5.5 = ype 2 (proximal) R A: Inability to reabsorb
HCO3 in proximal tubule HCO3 wasted in urine serum HCO3
decreases low ltered serum HCO3 ultimately low urine HCO3
with acidic pH
Acidic urine in distal tubule K+ wasting
O en associated with proteinuria, phosphaturia, glycosuria (i.e.,
Fanconis syndrome). May be due to meds (teno ovir, i os amide),
amyloid, MM, heavy metal toxicity
(b) Urine pH > 6 = ype 1 (distal) R A: inability to secrete H + in distal
tubule
Due to H + channel dys unction (e.g., Sjogrens, SLE) or increased
permeability o distal tubule cells to H + (e.g., amphotericin B, lithium)
How should each o the RTAs be managed?

ype 2: repletion o HCO3; may need to counteract HCO3 wasting with


thiazide
ype 1: correct hypokalemia be ore acidosis; potassium citrate treats both
ype 4: discontinue o ending agent; treat hypoaldosteronism with
udrocortisone

REFERENCES
1. Chapter 9. Regulation o acid-base balance. In: Eaton DC, Pooler JP, eds. Vanders Renal Physiology, 8th ed. New York, NY: McGrawHill; 2013.
2. Kau man D, Kitching AJ, Kellum JA. Chapter 77. Acid-base balance. In: Hall JB, Schmidt GA, Wood LH, eds. Principles of Critical
Care. 3rd ed. New York, NY: McGraw-Hill; 2005.
3. Koeppen BM, Stanton BA. Renal Physiology. 4th ed. Philadelphia, PA: Elsevier; 2007.

63

64

c)

>

NEPHROLOGY
CLASSIFICATION

Acute IntrinsicKidneyInjury

NEPHROLOGY

CLASSIFICATION

Acute IntrinsicKidneyInjury
Case: A 72-year-old woman undergoes a coronary angiogram and
post-catheterization is ound to have a serum creatinine o 2.5 mg/ dL, up
rom a baseline o 1.0 mg/ dL. Urinalysis is normal.
What is the de nition o acute kidney injury (AKI)?1

T ere is no universal de nition, but most agree that a creatinine rise o


0.5 mg/ dL or a 25% increase in serum creatinine within 48 hours is
consistent with AKI
What are the most common causes o AKI?1
Prerenal (decreased renal blood ow), intrinsic (de ect within kidney
parenchyma), and postrenal (obstruction o normal urine ow)
What are the major causes o intrinsic renal disease and how do you

di erentiate them?1,2
ubular disease: acute tubular necrosis (A N)
ubular obstruction is a less common cause (e.g., tumor lysis syndrome)
Glomerular disease: glomerulonephritis (GN)
May be primary (isolated to kidney such as IgA nephropathy) or
secondary ( rom systemic diseases such as SLE, Goodpastures disease,
small-vessel vasculitis, etc.)
Interstitial disease: interstitial nephritis
Most common o ending medications include NSAIDs, antibiotics,
PPIs, or anti-convulsants
Vascular disease
Macrovascular: renal in arction rom aortic dissection, thromboembolism,
or renal artery aneurysm
Microvascular: thrombotic microangiopathies (e.g., P/ HUS/ DIC) or
atheroembolic disease (e.g., cholesterol emboli)
Di erentiate based on clinical eatures, urinalysis, and speci c testing
(see table)
You suspect cholesterol embolization syndrome as the cause o AKI. How

would you manage this patient?


Supportive care (no speci c therapy): avoid nephrotoxins, await renal
recovery
Secondary prevention o cardiovascular disease: ASA, statin, anti-H N,
glycemic control, smoking cessation

REFERENCES
1. Yaqub M, Molitoris BA. Chapter 9. Acute kidney injury. In: Lerma EV, Berns JS, Nissenson AR, eds. CURREN Diagnosis & reatment: Nephrology & Hypertension. New York, NY: McGraw-Hill; 2009.
2. Waikar SS, Bonventre JV. Chapter 279. Acute kidney injury. In: Longo DL, Fauci AS, Kasper DL, Hauser SL, Jameson J, Loscalzo J, eds.
Harrisons Principles of Internal Medicine. 18th ed. New York, NY: McGraw-Hill; 2012.

64

65

ch

NEPHROLOGY
EVIDENCE-BASED MEDICINE

Erythropoietin in ChronicKidneyDisease

NEPHROLOGY

EVIDENCE-BASED MEDICINE

Erythropoietin in ChronicKidneyDisease
Case: A 63-year-old woman with stage 4 CKD presents with atigue and
is noted to have a hemoglobin o 8.5 mg/dL. You are considering starting
erythropoietin therapy.
How does chronic kidney disease (CKD) lead to anemia?1

Decreased production o erythropoietin (stimulus or red blood cell


production) by diseased kidneys normocytic, normochromic anemia
What labs must be checked prior to the initiation o erythropoietin?1
Baseline iron studies (serum iron, trans errin, and erritin levels)
I iron de cient, need to pursue potential cause o iron-de ciency (e.g., GI
blood loss) and replete iron stores prior to initiating erythropoietin
How can iron be administered to patients with CKD and iron-de ciency?1
IV iron ormulations are pre erred (more rapid repletion and better patient
adherence compared to PO errous sul ate)
1 g o iron will improve Hct by 10% over 3 months
IV iron gluconate (Ferrlecit) + iron sucrose (Veno er) are pre erred due to
reduced risk o anaphylaxis compared to IV iron dextran
Is a higher hemoglobin target pre erred in CKD?2

CHOIR trial: RC o 1,432 patients with predialysis CKD comparing


erythropoietin titrated to a hemoglobin target o 13.5 vs. 11.3 g/dL
Risk o primary outcome: composite end-point (death, myocardial
in arction, hospitalization or CHF, and/or stroke) increased in highhemoglobin target group vs. low target (hazard ratio o 1.34)
Quality o li e improvement was not signi cantly di erent between
the two groups (based on three di erent scoring systems including the
Linear Analogue Sel -Assessment score)
What hemoglobin target do the guidelines recommend or CKD?3

2012 KDIGO guidelines recommend a hemoglobin target o 1011.5 g/dL


What are the possible reasons or ailure to respond to erythropoietin
treatment?1
Iron de ciency, chronic in ammation, severe hyperparathyroidism (can lead
to bone marrow brosis), hemoglobinopathies, B12/ olate de ciencies

REFERENCES
1. Provenzano R. Chapter 18. Anemia & chronic kidney disease. In: Lerma EV, Berns JS, Nissenson AR, eds. CURREN Diagnosis &
reatment: Nephrology & Hypertension. New York, NY: McGraw-Hill; 2009.
2. Singh AK, Szczech L, ang KL, et al.; CHOIR Investigators. Correction o anemia with epoetin al a in chronic kidney disease. N Engl
J Med. 2006;355(20):20852098.
3. Andrassy KM, Kidney Disease: Improving Global Outcomes (KDIGO) CKD Work Group. KDIGO 2012 Clinical Practice Guideline or the Evaluation and Management o Chronic Kidney Disease. Kidney Int. 2013;3:1150.

65

DIAGNOSTIC APPROACH

NEPHROLOGY

Hypernatremia
Hype rnatre mia
(Na + >145 mEq/L)

As s e s s vo lume s tatus

Hypo vo le mia
Tota l body wa te r
Tota l body Na +

Euvo le mia
Tota l body wa te r
No cha nge in tota l body
Na + > 20/mmol/L

UNa
> 20 mmol/L

UNa
< 20 mmol/L

Re nal lo s s e s
Os motic or loop
diure tic
P os tobs truction
Intrins ic re na l
dis e a s e

Extrare nal lo s s e s
Exce s s s we a ting
Burns
Dia rrhe a
Fis tula e

Hype rvo le mia


Tota l body wa te r
Tota l body Na +

UNa
> 20 mmol/L

S o dium g ains
P rima ry
hype ra ldos te ronis m
Cus hings s yndrome
Na Cl ta ble ts

UNa
va ria ble

Re nal lo s s e s
Dia be tic ins ipidus
Hypodips ia

Extrare nal lo s s e s
Ins e ns ible los s e s :
(Re s pira tory, de rma l)

(Adapted with permission rom Johnson RJ, Freehaly J. Comprehensive Clinical Nephrology. 2nd ed. Mosby,
2003.)

Figure 7-6. Diagnostic algorithm or hypernatremia.


66

NEPHROLOGY

DIAGNOSTIC APPROACH

Hypernatremia
Case: A 23-year-old emale with an eating disorder is admitted with chronic
laxative abuse. Among other electrolyte disturbances, she is ound to have
a serum Na o 154 mEq/L.
How is hypernatremia de ned?1

Sodium concentration > 145 mEq/L


What is the bodys appropriate response to hypernatremia?
Release o antidiuretic hormone (ADH) to increase tubular resorption o
ree water decrease in serum sodium
What is the rst step in evaluating hypernatremia?

Assess uid status to exclude hypertonic uid gain


Most other causes are associated with normal or hypovolemic uid status
How does assessment o urine sodium guide urther evaluation?
Hypervolemic: an elevated urine sodium con rms sodium overload
Hypovolemic: a urine sodium > 20 mmol/L suggests renal losses o ree
water; a urine sodium < 20 mmol/L suggests extrarenal losses o ree water
Euvolemic: a urine sodium is less help ul. A urine osmolality > 700 suggests
extrarenal losses, while a urine osmolality < 700 suggests renal losses
How do you distinguish nephrogenic rom central diabetes insipidus?

Administer ADH. An increase in urine osmolality suggests central diabetes


insipidus, while an unchanged urine osmolality suggests nephrogenic
diabetes insipidus.
How should hypovolemic hypernatremia be treated?
I patient is in shock or has severe hypernatremia, give normal saline to
resuscitate the patient (as normal saline may be hypotonic relative to serum
sodium in this case)
Otherwise, treat the underlying cause (e.g., remove o ending medications)
Calculate the water de cit = 0.6 (body weight in kg) ([Na+ ]/1401)
Administer enterally ( ree water) or parenterally (D5W), and check [Na+ ]
every 23 hours to ensure a steady slow rate o correction
Correct Na+ at a rate no aster than 10 mEq/L/day or 0.5 mEq/L/hr to avoid
cerebral edema

REFERENCE
1. Mount DB. Chapter 45. Fluid and electrolyte disturbances. In: Longo DL, Fauci AS, Kasper DL, Hauser SL, Jameson J, Loscalzo J,
eds. Harrisons Principles of Internal Medicine. 18th ed. New York, NY: McGraw-Hill; 2012.

66

>

vo

yp

vo

yc

yp

yn

<

vo

yp

>

yn

67

<

>

NEPHROLOGY
DIAGNOSTIC APPROACH

Hyponatremia

NEPHROLOGY

DIAGNOSTIC APPROACH

Hyponatremia
Case: A 78-year-old emale is admitted with severe diarrhea. Her serum
Na+ is 127 mEq/L.
What is the rst step in the workup o hyponatremia?

Assess tonicity:
Isotonic hyponatremia = pseudohyponatremia (e.g., increased lipids/
protein) or hypotonic hyponatremia + hypertonic combination
Hypertonic hyponatremia = hyperglycemia, mannitol, other osmotically
active agent pulling water out o cells diluting Na+ (normal total body Na+ ).
Na+ by 1.6 mEq/L or every 100 mg/dL in glucose above 100 mg/dL
Hypotonic hyponatremia: the most common type o hyponatremia
How does the patients volume status and urine sodium guide urther

evaluation?1
Hypervolemia: increase in water is greater than the increase in sodium
UNa > 20: inability o kidneys to excrete water (renal ailure)
UNa < 20: kidneys seeing decreased e ective volume and actively
attempting to hold onto water by holding onto Na+ (e.g., CHF, cirrhosis)
Hypovolemia: body attempts to recover Na+ and water losses by holding
onto water to maintain blood pressure via release o ADH
UNa< 20: extrarenal losses that kidney makes up or (e.g., secretory
diarrhea, vomiting, burns)
UNa> 20: renal losses (e.g., thiazides loop diuretics; suspect
hypoaldosteronism i hyperkalemic)
Euvolemia: excess ADH release
Hypothyroid state, glucocorticoid de ciency, SIADH, psychogenic
polydipsia
How does urine osmolality help you identi y the cause o euvolemic

hyponatremia?
Uosm > 100 mOsm/kg = SIADH, hypothyroid, steroids
Uosm < 100 mOsm/kg = psychogenic polydipsia, low solute intake
What are common causes o SIADH?
Malignancy (e.g., lung, GI tract, GU tract, lymphomas), pulmonary
in ections, CNS in ections, drugs (e.g., SSRIs, CAs, narcotics), pain,
nausea, stress

REFERENCE
1. Mount DB. Chapter 45. Fluid and Electrolyte Disturbances. In: Longo DL, Fauci AS, Kasper DL, Hauser SL, Jameson J, Loscalzo J,
eds. Harrisons Principles of Internal Medicine. 18th ed. New York, NY: McGraw-Hill; 2012.

67

ac

68

NEPHROLOGY
CLASSIFICATION

Nephrolithiasis

NEPHROLOGY

CLASSIFICATION

Nephrolithiasis
Case: A 43-year-old woman presents with colicky groin pain. You suspect
she has a kidney stone.
What are the our major types o kidney stones?1,2

Calcium, uric acid, struvite, cysteine; 80% o stones are calcium-containing


stones
How are calcium stones ormed?1,2

Most are calcium-oxalate stones; there are three major mechanisms:


1. Excess renal calcium excretion: idiopathic hypercalciuria, primary
hyperthyroidism, sarcoidosis, excess vitamin D
2. Excess renal oxalate excretion: dietary hyperoxaluria (beets, spinach,
rhubarb, chocolate, tea); enteric hyperoxaluria (gastric bypass
surgery ree atty acids in gut binding to calcium in gut allows
ree oxalate to be absorbed in gut hyperoxaluria)
3. Low renal citrate excretion: citrate normally chelates calcium in the
kidney and prevents calcium binding with oxalate. Acidosis in distal
tubules citrate resorption hypocitraturia decreased calcium
chelation hypercalciuria
How are struvite stones ormed?
Formed in setting o in ection by Proteus or Klebsiella, which convert urea
to ammonium orms crystals with Mg and PO4
How are uric acid stones ormed?
Uric acid crystallizes at acidic pH (metabolic syndrome, chronic diarrhea,
and gout)
I kidney stones are suspected, what diagnostic tests should be ordered?

Labs: strained urine, can check serum calcium, urine pH


Plain abdominal radiography: detects radiopaque stones only
Noncontrast helical C : detects all stones, including radiolucent uric acid
stones
Note: abdominal ultrasound is not recommended (misses ureteral stones)
Af er treating the underlying cause, how are calcium stones prevented?
Fluid intake to achieve urine output > 2.5 L/day. Salt restriction to < 2 g/day
T iazide improves calcium resorption in proximal tubule
Dietary calcium encourages Ca-oxalate binding in gut decreased urine
oxalate
Paradoxically, decreasing calcium intake INCREASES calciuria

REFERENCES
1. Asplin JR, Coe FL, Favus MJ. Chapter 287. Nephrolithiasis. In: Longo DL, Fauci AS, Kasper DL, Hauser SL, Jameson J, Loscalzo J,
eds. Harrisons Principles of Internal Medicine. 18th ed. New York, NY: McGraw-Hill; 2012.
2. Stamatelou KK, Francis ME, Jones CA, Nyberg LM, Curhan GC. ime trends in reported prevalence o kidney stones in the United
States: 19761994. Kid Int. 2003;63:18171823.

68

69

ca

NEPHROLOGY
EVIDENCE-BASED MEDICINE

Renal ArteryStenosis

NEPHROLOGY

EVIDENCE-BASED MEDICINE

Renal ArteryStenosis
Case: A 66-year-old man with CAD and re ractory, new-onset hypertension
is noted to have an abdominal bruit on exam. Renal ultrasound conf rms
renal artery stenosis on the le t side.
What are the possible clinical presentations o hemodynamically signi cant

RAS?1,2
Hypertension: re ractory to multiple agents or new-onset (age < 30 years or
age > 55 years)
Multiple episodes o ash pulmonary edema
Acute kidney injury, especially a er starting an ACE inhibitor (particularly
in bilateral RAS)
Audible abdominal bruit on the a ected side
What are the main causes o RAS?1
Atherosclerosis o the renal artery (vast majority)
Fibromuscular dysplasia (rare; primarily in women < 40 years)
What are the di erent modalities available to diagnose RAS?1
Doppler ultrasonography (high sensitivity/speci city but operator-dependent)
C or MR angiography (also high accuracy + noninvasive, but requires
contrast)
Renal angiography (gold-standard but invasive, expensive, and requires
contrast)
What are the major treatment options or atherosclerotic RAS?1,2
Medical management (aspirin, statin, anti-hypertensives, smoking cessation)
Angioplasty with or without stenting (o en curative or bromuscular
dysplasia)
Surgical bypass is the most invasive and much less requently employed
How does medical therapy compare to stenting in the management o

RAS?3
T e CORAL trial compared medical therapy alone vs. medical therapy +
renal artery stenting in 947 patients with RAS (60% stenosis) +
hypertension or CKD
Primary outcome (composite o death rom cardiac/renal causes, MI,
stroke, CHF hospitalization, progressive renal insuf ciency, or need or
renal-replacement therapy) not signi cantly di erent between the two
arms
Small reduction in systolic blood pressure or intervention vs. control
group (2.3 mm Hg, P = 0.03)
Most common stenting complication: arterial dissection (2.2%)

REFERENCES
1. Watnick S, Dirkx . Chapter 22. Kidney disease. In: Papadakis MA, McPhee SJ, Rabow MW, eds. CURREN Medical Diagnosis &
reatment 2014. New York, NY: McGraw-Hill; 2014.
2. Elliott WJ, Kalahasti P, Lau SM, Nally JV, Gomez-Sanchez CE. Chapter 42. Secondary hypertension. In: Lerma EV, Berns JS, Nissenson AR, eds. CURREN Diagnosis & reatment: Nephrology & Hypertension. New York, NY: McGraw-Hill; 2009.
3. Cooper CJ, Murphy P, Cutlip DE, et al.; CORAL Investigators. Stenting and medical therapy or atherosclerotic renal-artery
stenosis. N Engl J Med. 2014;370(1):1322.

69

NEPHROLOGY

EVIDENCE-BASED MEDICINE

Renal Ultrasound in Acute KidneyInjury

Odds Ratio o
Hydronephrosis on
Renal Ultrasound
(95% CI)

P-value

Point
Score

History o hydronephrosisa

11.1 (3.041.3)

< 0.001

History o recurrent urinary tract


in ections

2.7 (0.88.5)

0.10

Diagnosis consistent with possible


obstruction b

2.4 (1.24.6)

0.01

Absence o prerenal AKI

2.3 (0.96.2)

.10

Nonblack race

2.1 (1.04.4)

0.06

Absence o CHF

2.1 (0.85.2)

0.12

Absence o exposure to
nephrotoxic medicationsc

2.1 (1.03.85)

0.053

Patient Characteristic

Total Score

Risk o Hydronephrosis (%)

Low (3.1)

Moderate (10.7)

High (16.1)

Hydronephrosis documented in medical record or imaging within 2 years (automatic placement in high
risk group).
b
History o BPH, pelvic/ abdominal cancer, neurogenic bladder, single unctional kidney, or prior pelvic
surgery.
c
Aspirin (> 81 mg/ d), diuretic, ACE inhibitor or IV vancomycin.
a

(Data rom Licurse A, Kim MC, Dziura J, et al. Renal ultrasonography in the evaluation o acute kidney
injury: developing a risk strati cation ramework. Arch Intern Med. 2010;170(21):19001907.)

Figure 7-10. Likelihood o detecting hydronephrosis on renal


ultrasound in acute kidney injury based on patient characteristics.

70

EVIDENCE-BASED MEDICINE

NEPHROLOGY

Renal Ultrasound in Acute KidneyInjury


Case: A 56-year-old woman with ovarian cancer is noted to have an elevated
creatinine to 2.5 mg/dL, up rom a baseline o 1.0 mg/dL 1 week ago. You
are concerned or renal obstruction.
What are the general causes o acute kidney injury (AKI)?

Prerenal (decreased renal per usion), intrinsic (de ect within kidney
parenchyma), and postrenal (obstruction o normal urine ow)
What are the rst steps in diagnosis and management o AKI?
Assess volume status and consider trial o uids (i not in decompensated
CHF)
Discontinue any nephrotoxic medications
Obtain urinalysis with sediment
Place bladder catheter or per orm a bladder scan to assess or urinary
retention
You decide to order a renal ultrasound. What nding is consistent with a
diagnosis o renal obstruction and how of en is it ound?1,2
Hydronephrosis (renal U/S has 90% sensitivity/speci city o detection)
Relatively rare ( ound in 110% o patients with AKI)
What actors increase the likelihood o

nding hydronephrosis in a

patient with AKI?


A retrospective study identi ed risk actors or hydronephrosis awarded
each risk actor 1 point, and then organized the sum o risk actors into
3 di erent risk groups (see tables)
Hydronephrosis identi ed in only 3.1% o low-risk patients (compared
to 10.7% in moderate-risk group and 16.1% in high-risk group) with a
negative likelihood ratio o 0.27
When hydronephrosis was urther strati ed to hydronephrosis necessitating an intervention, the prevalence in the low-risk group dropped
to 0.4% with a negative likelihood ratio o 0.13
2

What causes o renal obstruction may lead to acute kidney injury (AKI)?3

Intrinsic de ects: bilateral ureteral calculi or blood clots, BPH, prostate or


bladder cancer, urethral stricture or tumor
Extrinsic de ects: pregnancy, tumor (uterus, cervix, colon, rectum),
endometriosis, retroperitoneal brosis

REFERENCES
1. Chapter 69. Hydronephrosis. In: Usatine RP, Smith MA, Chumley HS, Mayeaux EJ Jr, eds. T e Color Atlas of Family Medicine.
2nd ed. New York, NY: McGraw-Hill; 2013.
2. Licurse A, Kim MC, Dziura J, et al. Renal ultrasonography in the evaluation o acute kidney injury: developing a risk strati cation
ramework. Arch Intern Med. 2010;170(21):19001907.
3. Sei er JL. Chapter 289. Urinary tract obstruction. In: Longo DL, Fauci AS, Kasper DL, Hauser SL, Jameson J, Loscalzo J, eds.
Harrisons Principles of Internal Medicine. 18th ed. New York, NY: McGraw-Hill; 2012.

70

DIAGNOSTIC APPROACH

NEUROLOGY

Benign Paroxysmal Positional Vertigo (BPPV)

S TEP 1: TURN HEAD 45 TO RIGHT

S TEP 2: MOVE PATIENT S UPINE


with RIGHT s ide do wn

P os te rior ca na l

45

vi

Gra vity

S a gitta l
pla ne

Pa ra lle l

P os te rior
ca na l

S upe rior
ca na l

Rota tory nys ta gmus ca uda lly


a nd to the right (a ffe cte d e a r)
Utriculus
P os te rior ca na l
Utriculus
P os te rior
Pa rticle s
now a ligne d
ca na l
with s a gitta l
P os te rior ca na l a mpulla
a mpulla
S upe rior
pla ne
ca na l
P os te rior
ca na l
Pa rticle s

(Ropper AH, Samuels MA, Klein JP. Chapter 15. Dea ness, Dizziness, and Disorders o Equilibrium. In: Ropper AH,
Samuels MA, Klein JP. eds. Adams &Victor's Principles o Neurology, 10e. New York, NY: McGraw Hill; 2014.)

Figure 8-1. A positive Dix Hallpike maneuver per ormed on the right
ear.

71

NEUROLOGY

DIAGNOSTIC APPROACH

Benign Paroxysmal Positional Vertigo (BPPV)


Case: A 55-year-old emale presents with new-onset dizziness that began
acutely upon awakening on the morning o presentation. Your leading
diagnosis is BPPV.
What symptoms suggest BPPV?1

Brie attacks, usually 1020 seconds, sometimes up to 90 seconds


Occurs with a ew seconds latency a er change in position
Frequently noted to occur in the middle o night or early morning due to
shi ing position during sleep or upon awakening; can also occur upon
looking up
May occur in clusters or weeks at a time, then asymptomatic or years in
between
It is the most common cause o positional vertigo in the population
What is thought to be the mechanism o BPPV?2
Otoliths that are normally xed become ree- oating in endolymphatic uid
in the semicircular canals, causing perception o movement
T e posterior canal is more commonly a ected than horizontal or anterior
canals
How is the DixHallpike maneuver per ormed?1

1. Stand on right side o patient, rotate patients head 45 degrees to right


(aligns right posterior semicircular canal with sagittal plane o body)
2. Move patient rom seated to supine position with neck extended back
30 degrees
3. Observe or up to 1 minute while supine, and then again a er returning to
upright
4. Repeat on the other side
A positive test = upbeat and rotatory nystagmus (toward right when right
ear is down) a er a ew seconds latency, typically lasts < 30 seconds
A ected side is the side that is down when nystagmus is noted
Nystagmus should also be (a) reversible (i.e., reverses upon sitting up
again) and (b) atigable (i.e., no longer elicited a er three to our trials)
How sensitive and speci c is the DixHallpike maneuver?3

A single study estimated sensitivity to be 79% (95% CI 6594) with a speci city
o 75% (95% CI 33100)

REFERENCES
1. Chapter 15. Dea ness, dizziness, and disorders o equilibrium. In: Ropper AH, Samuels MA, eds. Adams and Victors Principles of
Neurology. 9th ed. New York, NY: McGraw-Hill; 2009.
2. Furman JM, Cass SP. Benign paroxysmal positional vertigo. N Engl J Med. 1999;341:15901596.
3. Halker RB, Barrs DM, Wllik KE. Establishing a diagnosis o benign paroxysmal positional vertigo through the Dix-Hallpike and
side-lying maneuvers: a critically appraised topic. Neurologist. 2008;14:201204.

71

72

co

NEUROLOGY
PHYSICAL EXAM

Coma

NEUROLOGY

PHYSICAL EXAM

Coma
Case: A 49-year-old male was resuscitated in the f eld a ter a cardiac event.
He has undergone cooling, but a ter rewarming, he remains unresponsive.
How is coma de ned?

A state o unarousable unresponsiveness


What are the key elements o the neurological examination or coma?
Motor: ollowing commands, localizing to pain, exing/extending to pain,
or no response
Arousability: eye opening (spontaneous, to pain, or remaining closed), eye
tracking
Brainstem re exes: corneal, pupillary, vestibulo-ocular, cold calorics,
cough/gag
Respirations: regular breathing pattern, irregular pattern, apnea
How are the brainstem re exes tested in the comatose patient?

Pupillary: shine a light in one eye; observe or symmetric pupil


constriction
Corneal: apply a stimulus to the cornea; observe or symmetric acial
response
Vestibulo-ocular: gently shake head side-to-side; observe or eyes
remaining xated
Cold calorics: instill cold water in one ear; watch or slow phase toward
ear and ast phase away
Cough/gag: stimulate the posterior pharynx to cause gag or cough
How do you assess the severity o coma?

T ere are two coma scales to grade prognosis:


Glasgow coma scale (GCS)1: scores rom 1 to 5 each o eye opening, verbal
response, and motor response. otal score range 315. GCS score > 8
associated with low in-hospital mortality
FOUR score24: scores rom 0 to 4 each o eye opening, motor response,
brainstem re exes, and respirations. otal score 016. FOUR score > 12
associated with low in-hospital mortality
T is scale has the ability to assess patients that may be intubated (GCS
has verbal component), and includes a brainstem examination. It has
potentially improved inter-rater reliability compared to the GCS

REFERENCES
1. Cowan JA, Jr, T ompson B. Chapter 36. Neurosurgery. In: Doherty GM, eds. CURRENT Diagnosis & Treatment: Surgery. 13th ed.
New York, NY: McGraw-Hill; 2010.
2. Wijdicks EF, Bamlet WR, Maramattom BV. Validations o a new coma scale: T e FOUR score. Ann Neurol. 2005;58:585593.
3. Kornbluth J, Bhardwaj A. Evaluation o coma: A critical appraisal o popular scoring systems. Neurocrit Care. 2011;14:134143.
4. Fischer M, Regg S, Czaplinski A, et al. Inter-rater reliability o the Full Outline o Unresponsiveness score and the Glasgow Coma
Scale in critically ill patients: a prospective observational study. Critical Care. 2010;14:R64.

72

TREATMENT APPROACH

NEUROLOGY

Delirium
S us pe cte d De lirium

Es ta blis h ba s e line
cognitive function a nd
a cuity of a ny cha nge s

Cognitive a s s e s s me nt
(e .g., CAM, MoCA)

Rule out de me ntia ,


de pre s s ion, ma nia ,
a cute ps ychos is

De lirium confirme d

Ide ntify a nd a ddre s s a ll


pote ntia l unde rlying ca us e s
or contributing fa ctors

Ma na ge me nt of
de lirium s ymptoms

P ote ntia l ca us e or
contributor ide ntifie d

Furthe r options :
- La bora tory te s ts
(Che mis try, CBC, TFTs ,
B12, drug le ve ls ,
LFTs /a mmonia , cortis ol,
ABG, infe ctious s cre e n,
EKG)
- Bra in ima ging, LP , or
EEG

Tre a t
a ccordingly

All pa tie nts

S e ve re a gita tion
pre s e nt

Nonpha rma cologic s tra te gie s :


- Re orie nta tion a nd e ncoura ge fa mily
involve me nt
- Us e of s itte rs
- Eye gla s s e s a nd he a ring a ids
- Uninte rrupte d pe riod for s le e p a t night
- Avoid phys ica l re s tra ints
- Ma inta in mobility a nd s e lf-ca re
- Us e of light/da rk to pre s e rve s le e pwa ke cycle s

P ha rma cologic ma na ge me nt:


Only in s e ve re ly a gita te d pa tie nts a t ris k
of inte rruption of e s s e ntia l me dica l ca re
or pos ing s a fe ty ha za rd to the ms e lve s or
s ta ff
- Ha lope ridol 0.251.0 mg, IM, IV
- Ma y re pe a t Q2030 mins ., che ck VS
- Ma x 35 mg in 24 hr
- Divide loa ding dos e by 50% a nd give
in divide d dos e s for 24 hr, the n ta pe r
- Avoid othe r ps ychoa ctive drugs
including be nzodia ze pine s , unle s s
de lirium is s e conda ry to a lcohol
withdra wa l or nonconvuls ive s e izure s

(Adapted with permission rom Inouye SK. Delirium and other mental status problems in the older patient.
In: Goldman L, Bennett JC, eds. Cecil Textbook o Medicine. 21st ed. Philadelphia, PA: WB Saunders; 2000:18.)

Figure 8-3. Diagnostic and treatment algorithm or delirium.


73

NEUROLOGY

TREATMENT APPROACH

Delirium
Case: A 78-year-old emale, admitted with a new hip racture, becomes
agitated overnight.
What are the eatures o delirium?

T e diagnosis o delirium by the Con usion Assessment Method (CAM)


requires eatures 1 and 2 and either 3 or 41:
1. Acute onset or uctuating course
2. Inattention
3. Disorganized thinking
4. Altered level o consciousness (including hyperactive or hypoactive)
What are the risk actors or delirium?
Predisposing risk actors include: sensory impairments (vision/hearing),
dementia, age, severe underlying illness, alcohol abuse
Precipitating risk actors include medications (especially anticholinergics,
benzodiazepines, steroids), in ection, dehydration, sleep deprivation,
malnutrition, immobilization or racture at time o admission, metabolic
derangements, decreased mobility, constraining medical equipment (IV poles,
Foley, restraints)
What is the workup o delirium?
Con rm the diagnosis o delirium
Identi y contributing actors:
Review medications
Basic physical/laboratory assessment: physical exam/vital signs, electrolytes, nger stick glucose, ABG, CBC, urinalysis, urine culture, blood
culture, CXR
ailor urther lab investigation to initial ndings
Manage symptoms
How should delirium be managed?2

Nonpharmacologic means should be instituted rst: amily involvement,


preserve circadian rhythms, reorientation, eyeglasses/hearing aids,
encourage mobility
Pharmacologic: see ront o card. All antipsychotics are associated with
higher mortality in elderly (primarily cardiac e ect); haloperidol is
associated with highest mortality o all antipsychotics. Use as low dose or
the shortest time period possible

REFERENCES
1. Inouye SK. Delirium in older persons. N Engl J Med. 2006;354:11571165.
2. Josephson S, Miller BL. Chapter 25. Con usion and Delirium. In: Longo DL, Fauci AS, Kasper DL, Hauser SL, Jameson J, Loscalzo
J, eds. Harrisons Principles of Internal Medicine. 18th ed. New York, NY: McGraw-Hill; 2012.

73

TREATMENT APPROACH

NEUROLOGY

Migraine

P OUND P os itive

S e ve rity

MILD

MODERATE TO
S EVERE

NS AIDS
+ /- a ntie me tics

NS AIDS + a ntie me tics


P e rs is te nt s ymptoms
Add a tripta n
S till pe rs is te nt or ve ry
s e ve re s ymptoms
Cons ide r:
- Ke torola c (or dihydroe rgota mine )
with me toclopra mide
- De xa me tha s one

Figure 8-4. Treatment algorithm or migraine headache.

74

NEUROLOGY

TREATMENT APPROACH

Migraine
Case: A 34-year-old emale presents with a unilateral, pounding headache
associated with sensitivity to light.
What eatures most strongly suggest a diagnosis o migraine?

Pulsating headache, lasting between 4 and 72 hOurs without medications,


Unilateral, Nausea, Disabling (POUND)
What therapies are use ul to treat migraines?

Mild: Simple analgesics, e.g., NSAIDs, acetaminophen, and aspirin


pre erred
Low rate o adverse e ects, less expensive, shown to be e ective2
Patients not responding to one NSAID may still respond to a di erent
NSAID, and equal ef cacy has been demonstrated or multiple NSAIDs
Can add oral antiemetic or added abortive e ect and to control nausea
Prochlorperazine and metoclopramide not in erior to sumatriptan in
resolving migraine.3,4 Must monitor or Q prolongation
Moderate/severe: In addition to above, add a triptan
Sumatriptan with naproxen may be more e ective than either agent
alone5. Sumatriptan is available in non-oral orms (SC, rectal, nasal) all
with demonstrated ef cacy. Patients not responding to one triptan may
still respond to a di erent triptan
I still persistent or very severe, ketorolac IV or dihydroergotamine
1 mg IV with metoclopramide 10 mg IV may help. A single dose o
dexamethasone (1025 mg IV) may reduce recurrence
What is the overall strategy or timing and dosing o acute migraine

treatment?
reat early (goal is or complete resolution in 12 hours)
A larger single dose tends to work better than repetitive small doses
What is the role o narcotics in treating acute migraine?
Recommended only as a last resort: not more e ective than other treatments
Have no anti-in ammatory or neurovascular e ect on migraine
pathophysiology
Associated with increased risk or development o chronic migraine, and
higher rate o return to ED in 7 d (11.5 vs. 4.9, p = 0.011)6

REFERENCES
1. Michel P, Henry P, Letenneur L, et al. Diagnostic screen or assessment o the IHS criteria or migraine by general practitioners.
Cephalgia. 1993;13:5459.
2. Law S, Derry S, Moore RA. Naproxen with or without an antiemetic or acute migraine headaches in adults. Cochrane Database Syst
Rev. 2013;10:CD009455.
3. Kelly NE, epper DE. Rescue therapy or acute migraine, part 1triptans, dihydroergotamine, and magnesium. Headache.
2012;52:114128.
4. Colman I, Brown MD, Innes GD, Gra stein E, Roberts E, Rowe BH. Parental metoclopramide or acute migraine: meta-analysis
o randomised controlled trials. BMJ. 2004;329:13691373.
5. Law S, Derry S, Moore RA. Sumatriptan plus naproxen or acute migraine attacks in adults. Cochrane Database Syst Rev. 2013;
10:CD008541.
6. Colman I, Rothney A, Wright SC, Zilkalns B, Rowe BH. Use o narcotic analgesic in the emergency department treatment o
migraine headache. Neurology. 2004;62:16951700.

74

75

rs

73

i
tA
Ibh
i
n
n
onaI
1
b
a
imr
lbw
i
I
H
i
t
l
m
a
e
y
sletii
tym
l
k
o
t
e
tuo
narbvo
y
e
n
f
oo
loocsofl
A
w
l
f
o
o
r
eCf
ebm
ncw
rN
o
p
o
nt
rS
orew
m
s
s
o
Acdm
bsnqoicsiae
l
G
o
anvetue
oidFesam
s
z
t
a
n
r
s
s
i
teuom
cm

er

i
uz

s.

NEUROLOGY
EVIDENCE-BASED MEDICINE

Neuroimaging Before Lumbar Puncture

NEUROLOGY

EVIDENCE-BASED MEDICINE

Neuroimaging Before Lumbar Puncture


Case: A 43-year-old man is admitted with concern or meningitis. You plan
on per orming a lumbar puncture.
Why would you consider neuroimaging?

Initially, universal neuroimaging was recommended be ore lumbar puncture


to identi y intracranial abnormalities that may put a patient at risk o brain
herniation when cerebrospinal uid was removed on lumbar puncture
On whom do you order neuroimaging?

Gopal et al. identi ed our eatures: (1) altered mental status (LR 2.2,
95% CI 1.53.2), (2) ocal neurologic exam (LR 4.3, 95% CI 1.910),
(3) papilledema (LR 11.1, 95% CI 1.1115), and (4) overall clinical
impression (LR 18.8, 95% CI 4.843), which together identi y patients at
greatest risk o LP complications1
Hasbun et al. ound the presence o certain eatures (age > 60 years,
immunocompromised state, history o CNS disease, seizure within
1 week, or a ocal neurologic exam nding) identi es patients likely to
have an abnormal C 2 (see image)
Absence o any o these eatures accurately predicted a normal head
C 97% o the time and absent mass e ect 100% o the time
78% o patients received neuroimaging; a high percentage o these
C s were normal (76%) and use o above eatures to in orm need or
imaging would have decreased requency by 41%
What is the major downside to universal neuroimaging be ore LP?

Head C delayed time to LP (5.3 vs. 3.0 hr, p < 0.001 per Hasbun et al.) and
antibiotics (2.8 hr delay per Gopal et al.). Delay in antimicrobials > 3 hr a er admission is an independent risk actor or mortality (OR 14.1, 95% CI 3.950.9)3
Diagnostic power o CSF analysis may be limited when antibiotics given
empirically prior to LP
Others: cost, unnecessary radiation exposure

REFERENCES
1. Gopal AK, Whitehouse JD, Simel DL, Corey GR. Cranial computed tomography be ore lumbar puncture: a prospective clinical
evaluation. Arch Intern Med. 1999:159;26812685.
2. Hasbun R, Abrahams J, Jekel J, Quagliarello VJ. Computed tomography o the head be ore lumbar puncture in adults with suspected meningitis. N Engl J Med. 2001;345:17271733.
3. Auburtin M, Wol M, Charpentier J. Detrimental role o delayed antibiotic administration and penicillin-nonsusceptible strains in
adult intensive care unit patients with pneumococcal meningitis: the PNEUMOREA prospective multicenter study. Crit Care Med.
2006;34:27582765.

75

76

NEUROLOGY
PHYSICAL EXAM

Nystagmus

NEUROLOGY

PHYSICAL EXAM

Nystagmus
Case: A 23-year-old emale with ear pain and hearing loss is ound to have
horizontal nystagmus.
Is nystagmus ever normal?

Yes. ypes o physiologic nystagmus include:1


1. Caloric-induced nystagmus (irrigation o external auditory canal with
warm or cold water produces nystagmus)
2. Optokinetic (eyes response to continually moving object)
3. End-point nystagmus (on extreme lateral gaze, eyes atigue, and dri
back to center; quick phase nystagmus corrects this to continue gazing
laterally)
What conditions cause nystagmus?2
Peripheral nystagmus (vestibular lesions): inner ear disorders such as
Menieres disease, labyrinthitis, vestibular neuritis, benign paroxysmal
positional vertigo
Central nystagmus (brainstem or cerebellar lesion): mass or stroke, drugs
(sedatives, anticonvulsants), alcoholism
How can you dif erentiate peripheral rom central causes o nystagmus by

history and on exam?1


Peripheral: usually more acute, severe, short lasting, but recurrent; usually
with mixed directional components; o en associated with hearing loss or
tinnitus
Fast phase points away rom the side o the lesion
Central: usually more mild and chronic; usually purely horizontal, vertical,
or rotational, and usually with CNS symptoms rather than hearing loss
Fast phase changes with direction o gaze (e.g., le -beating when
gazing le , right beating when gazing right). T e combined presence
o (1) bidirectional nystagmus, (2) vertical ocular misalignment (skew
deviation), and (3) normal vestibulo-ocular re ex is highly predictive o
a posterior ossa in arct.3
When should imaging be obtained on a patient with nystagmus?

Central orms o nystagmus, nystagmus without an identi able cause, or


nystagmus associated with a ocal neurologic de cit
MRI is the pre erred imaging modality o choice

REFERENCES
1. Ropper AH, Samuels MA. Chapter 14. Disorders o ocular movement and pupillary unction. In: Ropper AH, Samuels MA, eds.
Adams & Victors Principles of Neurology. 10th ed. New York, NY: McGraw-Hill; 2014.
2. Riordan-Eva P, Hoyt WF. Chapter 14. Neuro-Ophthalmology. In: Riordan-Eva P, Cunningham E Jr, eds. Vaughan & Asburys
General Ophthalmology. 18th ed. New York, NY: McGraw-Hill; 2011.
3. Kattah JC, alkad AV, Wang DZ. HIN S to diagnose stroke in the acute vestibular syndrome. Stroke. 2009;40:35043510.

76

<

&

co

>

co

ce

77

NEUROLOGY
DIAGNOSTIC APPROACH

Seizure Versus Syncope

NEUROLOGY

DIAGNOSTIC APPROACH

Seizure Versus Syncope


Case: A 78-year-old man is brought to the emergency room a ter a
witnessed transient loss o consciousness associated with jerking
movements.
What characteristics do seizure and syncope share?1

ransient loss o consciousness lasting seconds to minutes


Convulsive syncope and seizures can both be associated with abnormal limb
jerking (up to 15% o patients with syncope)
How do you distinguish between seizure and syncope?2

T e Syncope Symptom Study (Sheldon et al., JACC, 2002) evaluated


671 patients with de nitive gold-standard proven syncope or seizure
(e.g., tilt-table positive syncope, documented V -induced syncope,
electrographically documented heart block induced syncope, or
EEG-documented epilepsy)
Patients completed 117 item questionnaire, and likelihood ratios or
seizure were calculated or each variable (see ront o card)
How do you workup a suspected seizure?3,4

According to the joint recommendations o the American Academy o


Neurology and the American Epilepsy society, a patient should have the
ollowing:
Selected lab tests: lumbar puncture may be indicated to rule out an in ectious cause o seizure. Electrolytes, glucose, CBC, toxicology screens, renal,
and liver unction tests rarely help diagnose a cause o seizure but may
in orm choice o antiepileptic drug
EEG: a normal EEG does not rule out epilepsy as an EEG is only abnormal
in 50% o patients presenting rst with seizure, and only hal o these have
epilepti orm discharges. Sensitivity is increased when repeated or a er
patient has undergone sleep deprivation 3
May help prognosticate risk o recurrence. Epilepti orm discharges on
EEG doubles the risk o seizure recurrence
Neuroimaging: C is appropriate in emergency situations, but an MRI o
the brain is more sensitive than C in identi ying structural lesions5

REFERENCES
1. Palac SM. Chapter 212. Seizures. In: McKean SC, Ross JJ, Dressler DD, Brotman DJ, Ginsberg JS, eds. Principles and Practice of
Hospital Medicine. New York, NY: McGraw-Hill; 2012.
2. Sheldon R, Rose S, Ritchie D, et al. Historical criteria that distinguish syncope rom seizures. J of Am Coll Cardiol. 2002;40:
142148.
3. Krumholz A, Wiebe S, Gronseth G, et al. Practice Parameter: evaluating an apparent unprovoked rst seizure in adults (an evidence-based review): report o the Quality Standards Subcommittee o the American Academy o Neurology and the American
Epilepsy Society. Neurology. 2007;69(21):19962007.
4. French JA, Pedley A. Initial Management o Epilepsy. N Engl J Med. 2008;359:166167.
5. Bronen RA, Fulbright RK, Spencer DD, et al. Re ractory epilepsy: comparison o MR imaging, C , and histopathologic ndings in
117 patients. Radiology. 1996;201:97105.

77

'

kg

zo

kg

kg

>

rk

ve

kg

kg

kg

ve

>

va

ze

78

<

NEUROLOGY
TREATMENT APPROACH

Status Epilepticus

NEUROLOGY

TREATMENT APPROACH

Status Epilepticus
Case: A 36-year-old male with a history o seizures has a generalized
convulsive seizure.
How is status epilepticus (SE) de ned?

raditional de nition: continuous seizure > 30 minutes or repetitive seizures


without interval awakening1
Practical de nition: treatment/prevention o SE begins when seizure
> 5 minutes
Likelihood o success ul treatment o SE inversely related to duration
aim to aggressively treat SE early
What should be done rst or a patient who is seizing?
Basic li e support: ensure circulation, airway, breathing, check vital signs
Obtain IV access, start empiric O2, and place on cardiac monitoring
Check labs: ngerstick blood glucose, CBC, electrolytes, BUN/Cr, ABG,
antiepileptic drug levels, toxicology screen
Consider IV thiamine i alcoholic history suspected (prior to dextrose)
What are the rst-line drug treatments or a patient that is seizing?2

Benzodiazepines: lorazepam pre erred over diazepam because o longer


anti-seizure e ect. IM or PR i no IV access
IV osphenytoin or IV phenytoin: osphenytoin is prodrug or phenytoin
and can be administered aster (phenytoin can cause hypotension when
administered at aster rates)
How should re ractory SE be treated?2,3
Further treatment causes respiratory suppression and hypotension, so
patient should be intubated
Antiepileptics: phenobarbital or valproic acid
IV anesthetics: propo ol or midazolam
What is the prognosis o generalized convulsive SE?1

Overall 30-day mortality: 20%, but is heavily dependent on cause


I seizure is due to anoxic brain injury, 30-day mortality is 70%
Seizures due to subtherapeutic antiepileptics are rarely atal
What are the classic complications o SE?
Rhabdomyolysis (monitor CK, rehydrate, alkalinize urine)
Hyperthermia

REFERENCES
1. Lowenstein DH, Alldredge BK. Status epilepticus. N Engl J Med. 1998;338:970976.
2. reiman DM, Meyer PD, Walton NY, et al. A comparison o our treatments or generalized convulsive status epilepticus. N Engl J
Med. 1998;339:792798.
3. Bassin SL, Fountain NB, Bleck P. Chapter 64. Seizures in the Intensive Care Unit. In: Hall JB, Schmidt GA, Wood LH, eds. Principles of Critical Care. 3rd ed. New York, NY: McGraw-Hill; 2005.

78

TREATMENT APPROACH

NEUROLOGY

IntravenousTPAin Patients with IschemicStroke


P a tie nt a rrive s with s us pe cte d s troke

Time of ons e t <4.5 hours *


Age 18 years

P ote ntia l ca ndida te for IV TP A

Abs o lute
c o ntraindic atio ns
Hea d tra uma /s troke within
3 months

P re vious suba ra chnoid or


intra cra nia l he morrha ge

Intra cra nia l ne opla s m or


va s cula r ma lforma tion

Re cent CNS surgery


Arteria l puncture a t non-

Re lative
c o ntraindic atio ns

Ra pidly improving or mild

s ymptoms
S e izure a t ons et of s troke
GI or urina ry tra ct ble e ding
Myoca rdia l infa rction in the
pa s t 3 months
Ma jor surgery or tra uma in
the pa s t 14 da ys

No
e xc lus io n
c rite ria

compre s sible site within 7 da ys

P la telets <100,000
INR >1.7, P T >15 se conds
BP >185/110
Re cent he pa rin us e with P TT

Cons ide r IV TP A in
pa tie nts with a dis a bling
ne urologic de ficit
(e .g., comple te he mia nops ia ,
s e ve re a pha s ia , e tc.)

Adminis te r
IV TP A

Do not a dminis te r IV TP A

Cons ide r intra -a rte ria l TP A or e mbole ctomy if ima ging re ve a ls ma jor
ce re bra l a rte ry occlus ion, e ve n if be yond window
*TPA is only approved by FDA or treatment within 3 hours, but many stroke centers treat patients
presenting 3 4.5 hours a ter onset.

Figure 8-9. Treatment algorithm or administration o TPA in patients


79 with ischemic stroke.

NEUROLOGY

TREATMENT APPROACH

IntravenousTPAin Patients with IschemicStroke


Case: A 67-year-old man presents to the emergency department 45 minutes
a ter the onset o dysarthria and right-sided weakness. Head CT is negative
or hemorrhage. Blood pressure is 155/70. You consider whether to give IV
TPA.
In which patients with ischemic stroke should IV TPA be considered?

Clinical diagnosis o ischemic stroke with neurologic de cits, onset


< 4.5 hours, age 18 years1,2
How is the onset o a stroke de ned?
ime rom when symptoms rst observed
I onset was not observed or occurred during sleep, onset de ned as time
when patient was last awake and neurologically normal
Can IV TPA be given outside the FDA-approved 3-hour window?
PA is only approved by FDA or treatment within 3 hours, but many stroke
centers treat patients presenting 34.5 hours a er onset
T e 3-hour window was based on the NINDS PA Stroke Study3
Later studies showed no bene t to PA when window extended to 6 hours
ECASS III trial: PA vs. placebo given between 34.5 hours a er onset4
Improved outcomes at 90 days (OR 1.34, 95% CI 1.021.76, p = 0.04)
What are contraindications to administering TPA or stroke?3,4

Head trauma/stroke within 3 months, current/prior intracranial hemorrhage


Arterial puncture at a noncompressible site within the past 7 days
Coagulopathy (platelets > 100 K, recent heparin use with elevated P ,
INR > 1.7)
Recent major surgery (within 18 days based on NINDS trial, within
3 months per ECASS III trial)
ECASS III: age > 80, severe symptoms (NIHSS score > 25), involvement
o > 1/3 o cerebral hemisphere (prone to hemorrhagic trans ormation),
anticoagulant use
How should blood pressure be managed in ischemic stroke?

Blood pressure is typically allowed to autoregulate to quite high values


Goal or most ischemic stroke patients: SBP < 220, DBP < 120
Patients receiving PA: < 185/110 be ore PA can be given
Maximum limits primarily help prevent possible hemorrhagic conversion

REFERENCES
1. Jauch EC, Cucchiara B, Adeoye O, et al. Part 11: adult stroke: 2010 American Heart Association guideline or cardiopulmonary
resuscitation and emergency cardiovascular care. Circulation. 2010;122(Suppl 3):S818S828.
2. Wechsler LR. Intravenous thrombolytic therapy or acute ischemic stroke. N Engl J Med. 2011;364:21382146.
3. T e National Institute o Neurological Disorders and Stroke rt-PA Stroke Study Group. issue plasminogen activator or acute
ischemic stroke. N Engl J Med. 1995;333:15811588.
4. Hacke W, Kaste M, Bluhmki E, et al. T rombolysis with alteplase 3 to 4.5 hours a er acute ischemic stroke. N Engl J Med.
2008;359:13171329.

79

EVIDENCE-BASED MEDICINE

NEUROLOGY

Transient IschemicAttack

Risk Factor

Point(s)

Age
60 years

Blood pressure
Systolic BP 140 mm Hg or Diastolic BP 90 mm Hg

Clinical eatures
Unilateral weakness with or without speech impairment

Speech impairment without unilateral weakness

Duration
60 min

1059 min

Diabetes mellitus

Risk category based on ABCD2 score


High Risk

67

Moderate Risk

45

Low Risk

03

(Data obtained rom Johnston SC, Rothwell PM, Nguyen Huynh MN, Giles MF, Elkins JS, Bernstein AL, Sidney S.
Validation and re nement o scores to predict very early stroke risk a ter transient ischaemic attack. Lancet.
2007;369(9558):283292.)

Figure 8-10. The ABCD2 score or the evaluation o a transient


ischemic attack.

80

NEUROLOGY

EVIDENCE-BASED MEDICINE

Transient IschemicAttack
Case: A 62-year-old male with hypertension and type 2 diabetes mellitus
presents with le t arm weakness and slurred speech. His symptoms lasted
or 30 minutes and then resolved.
What is the de nition o a transient ischemic attack (TIA)?

ransient episode o neurologic dys unction due to ischemia without


evidence o acute in arction
What is the clinical signi cance o a TIA?
Increased risk o developing recurrent stroke (i.e., acute in arction)
T ere ore, patients with IA should be urgently evaluated to identi y
modi able risk actors
How can we identi y patients who are at high risk o a recurrent stroke?1,2

ABCD2 score: prognostic score derived rom ve di erent risk actors at


presentation that estimates the risk o developing stroke within 2 days
High risk (score 67): 8.1% risk
Moderate risk (score 45): 4.1% risk
Low risk (score 03): 1.0%
Which patients with TIA should be hospitalized?2

T e AHA/ASA 2009 guidelines recommend hospitalization or patients with


IA who present within 72 hr o symptom onset i :
ABCD2 score 3
ABCD2 o 02 but uncertainty i diagnostic work-up can be completed
within 2 days as an outpatient
ABCD2 o 02 and other evidence that the IA was caused by ocal
ischemia
How should patients with TIA be evaluated?2
Blood tests (BMP, CBC, P /P , asting lipids, and glucose) + EKG
Brain imaging: MRI as pre erred modality (head C i MRI not possible)
Neurovascular evaluation: MR-angiography or C -angiography
Duplex U/S + transcranial Doppler also an option, but operator-dependent
Cardiac evaluation (i above work-up is unrevealing or cause o IA)
Echocardiography and cardiac monitoring to exclude atrial brillation

REFERENCES
1. Johnston SC, Rothwell PM, Nguyen-Huynh MN, et al. Validation and re nement o scores to predict very early stroke risk a er
transient ischaemic attack. Lancet. 2007;369(9558):283292.
2. Easton JD, Saver JL, Albers GW, et al. De nition and evaluation o transient ischemic attack. Stroke. 2009;40(6):22762293.

80

81

xi

yn

yp

ve

s.

PULMONARY
PHYSICAL EXAM

Abnormal Breathing Patterns

PULMONARY

PHYSICAL EXAM

Abnormal Breathing Patterns


Case: A 53-year-old male in the intensive care unit is admitted with decompensated heart ailure. His nurse notices he has periodic periods o apnea.
What are the most common abnormal respiratory patterns?

CheyneStokes: cycles o crescendo/decrescendo hyperpnea and apnea


Central hyperventilation: persistent deep hyperventilation
Apneustic: especially prolonged and deep inspiration, ollowed by apnea
Ataxic: irregularly irregular breathing interspersed with random pauses
What might each respiratory pattern signi y?

Many patterns indicate a neurologic lesion or a systemic abnormality.


See image or speci c causes.
CheyneStokes: 90% o patients have a cardiopulmonary disease1, ewer
patients have a hemispheric cerebral lesion. Also seen in normal elderly
patients during sleep. Little prognostic value.
Hyperventilation: most commonly secondary to an underlying metabolic
acidosis (e.g. DKA) or liver ailure; can be improved by treating the
underlying abnormality. Less commonly re ects a lesion in the midbrain.2
Apneustic: re ects inability to regulate inspiration due to brainstem damage, usually signi ying extremely poor prognosis.3
Ataxic: the only breathing pattern with true localizing value. Signi es
medullary stroke/lesion,3 or seen in agonal breathing be ore death
What is the physiologic basis or CheyneStokes respiration?4,5

Disrupted control o respiratory center in regulating PaCO2 due to abnormal


sensitivity o respiratory center to changes in PaCO2 . T eoretically, patient
may become apneic be ore center triggers breaths (undersensitivity) or
hyperventilates to small increases in PaCO2 (oversensitivity). wo pathways:
Neurogenic: loss o rontal lobe control over respiratory patterns
Cardiogenic: prolongation o lung-to-brain circulation time that causes
delayed and overshooting responses to changes in PaCO2
How should CheyneStokes be treated?
Usually no treatment is needed other than treating underlying cause
(e.g., CHF)
I present, treat hypocapnia and hypoxemia to reduce cerebral vasoconstriction

REFERENCES
1. Lee MC, Klassen AC, Resch JA. Respiratory pattern disturbances in ischemic cerebral vascular disease. Stroke. 1974;5(5):612616.
2. North JB, Jennet S. Abnormal breathing patterns associated with acute brain damage. Arch Neurol. 1974;31:338344.
3. Frank JI. Abnormal breathing patterns. In: Hanley DF, Einhaupl KM, Bleck P, Diringer MN; Neurocritical Care, eds. Heidelberg,
Germany: Springer-Verlag; 1994:366.
4. Rosengart AJ, Novakovic RL, Frank JI. Chapter 67. Coma, Persistent Vegetative State, and Brain Death. In: Hall JB, Schmidt GA,
Wood LH, eds. Principles of Critical Care. 3rd ed. New York, NY: McGraw-Hill; 2005.
5. Mann DL, Chakinala M. Chapter 234. Heart ailure and cor pulmonale. In: Longo DL, Fauci AS, Kasper DL, Hauser SL, Jameson J,
Loscalzo J, eds. Harrisons Principles of Internal Medicine. 18th ed. New York, NY: McGraw-Hill; 2012.

81

82

co

ym

yr

ve

ym

ym

ym

ye

ye

ye

ye

xa

xa

xa

<

xa

<

ve

ve

<

<

>

>

PULMONARY
TREATMENT APPROACH

ChronicAsthma and COPD

PULMONARY

TREATMENT APPROACH

ChronicAsthma and COPD


Case: A 70-year-old male with a history o COPD is admitted or his third
COPD exacerbation this year. His pulmonary unction tests show an
FEV1 /FVC o 55%, and an FEV1 o 45% predicted.
What are the mechanisms o action o medications used to treat asthma and

COPD?
Bronchodilatory: counteracts obstruction
wo types: 2-agonists and anticholinergics, both with short- and
long-acting orms
Anti-in ammatory: counteracts in ammation
Steroids come in oral or inhaled orms
Leukotriene-modi ying agents (used in asthma only)
What is the initial approach to mild chronic COPD and asthma?

Short-acting bronchodilators are rst line (short-acting -agonists


pre erred)
How does escalation o treatment di er in asthma and COPD?
Asthma: steroids (inhaled) are introduced earlier
COPD: long-acting -agonists or anticholinergics initiated a er
short-acting agents; steroids are only added a er maximizing long-acting
-agonists or anticholinergics1
Why does this treatment strategy make sense rom a pathophysiologic

perspective?
Asthma: punctuated by episodes o reversible in ammation that cause acute
airway obstruction. Earlier steroid use reduces exacerbations
COPD: chronic ongoing airway collapse rom (1) emphysematous destruction
o alveoli and lung parenchyma and (2) bronchiolitic in ammation. Patients
with worsening obstruction bene t more rom bronchodilators early.
What nonpharmacologic options should be o ered in asthma and COPD
patients?
Asthma: pneumococcal vaccine i age 19 and annual in uenza vaccine2
COPD: smoking cessation therapy, oxygen or patients with chronic
hypoxemia at rest (PaO2 55 mmHg or SpO2 88%), pneumococcal
vaccine and annual in uenza vaccine
Oxygen therapy is the only intervention shown to reduce mortality in
COPD
As disease progresses, can re er patients or pulmonary rehabilitation,
noninvasive positive pressure ventilation, lung reduction surgery, or
transplant

REFERENCES
1. Global strategy or the diagnosis, management, prevention o COPD: Revised 2014. Global initiative or Chronic obstructive lung
disease (GOLD). http://www.goldcopd.org (Accessed on 2014).
2. Centers or Disease Control and Prevention. Vaccines and Immunizations. http:// www.cdc.gov/vaccines/vpd-vac/pneumo/vaccin-short.htm. (Accessed on June 4, 2014).

82

PATHOPHYSIOLOGY

PULMONARY

COPDand Oxygenation

No rm al
O2
CO 2

Po o r Ve ntilatio n

150 m m Hg
0 m m Hg

O2
CO 2

150 m m Hg
0 m m Hg

Airw a y

O2
CO 2
O2
CO 2

100 m m Hg
40 m m Hg

O2
CO 2

40 m m Hg
45 m m Hg

Pu lm o n a ry
a rte rio le

Alve o lu s

Diffu s io n

De cre as e d O 2
Incre as e d CO 2
100 m m Hg
40 m m Hg

O2
CO 2

100 m m Hg
45 m m Hg

De cre as e d O 2
Incre as e d CO 2

Pu lm o n a ry
ve n u le

Hypo xic Pulm o nary Vas o co ns trictio n


O2
CO 2

S hunt aw ay fro m po o r ve ntilatio n

150 m m Hg
0 m m Hg
O2
CO 2

100 m m Hg
40 m m Hg

De cre as e d O 2
Incre as e d CO 2

De cre as e d O 2
Incre as e d CO 2
O2
CO 2

40 m m Hg
45 m m Hg

Im p ro ve d
ve n tila tio n -p e rfu s io n
m a tch in g

S hunt

(Levitzky MG. Chapter 4. Blood Flow to the Lung. In: Levitzky MG. eds. Pulmonary Physiology, 8e . New York,
NY: McGraw-Hill; 2013.)

Figure 9-3. Hypoxic pulmonary vasoconstriction: response to poorly


ventilated areas o the lung.

83

PULMONARY

PATHOPHYSIOLOGY

COPDand Oxygenation
Case: A 64-year-old woman with severe COPD is admitted with a COPD
are and is started on supplemental oxygen titrated to O2 sat o 100%. A
repeat ABG shows worsening hypercapnia.
What are some causes o poor ventilation within the lung?

Airway obstruction (e.g., COPD, asthma, oreign body), alveolar consolidation


(e.g., pneumonia, pulmonary edema) or localized atelectasis
What happens to pulmonary blood ow when there is poor ventilation

within the lung?1


Poor ventilation O2 and CO2 in the alveolar unit (panel B)
pulmonary vascular resistance in the hypoxic areas o the lung (hypoxic
pulmonary vasoconstriction (panel C) venous blood is pre erentially
shunted toward areas o lung with ventilation to restore ventilation/
per usion matching (panel D)
How does overoxygenating a patient with COPD a ect PaCO2 levels?

PaCO2 levels are increased (hypercapnia) due to:


1. Reversal o hypoxic pulmonary vasoconstriction 2: O2 delivery to obstructed
areas o the lung reversal o hypoxic pulmonary vasoconstriction
increased blood ow to poorly ventilated areas o the lung
2. Haldane e ect3: oxygenation o hemoglobin oxyhemoglobin levels
that have decreased CO2 carrying capacity compared to deoxygenated
hemoglobin less CO2 carried rom tissue back to lungs to be exhaled
higher PaCO2
3. Hypoxemic respiratory drive: hypoxia plays a small role in stimulating
respiration. Reversing hypoxia may decrease respiratory drive and worsen
hypercapnia (although this is controversial)
Is there a mortality bene t o long-term oxygen therapy (LTOT) in COPD?4
Yes: the NO
and MRC trials both showed a mortality bene t o L O or
patients with COPD and severe resting hypoxemia
Current indications or continuous L O include severe resting hypoxemia
(PaO2 55 mm Hg or SaO2 88%) or moderate resting hypoxemia (PaO2
5559 or SaO2 = 89%) with evidence o right-sided heart ailure, pulmonary
hypertension or hematocrit > 56%

REFERENCES
1. Chapter 4. Blood ow to the lung. In: Levitzky MG, eds. Pulmonary Physiology. 8th ed. New York, NY: McGraw-Hill; 2013.
2. Reilly JJ Jr, Silverman EK, Shapiro SD. Chapter 260. Chronic obstructive pulmonary disease. In: Longo DL, Fauci AS, Kasper DL,
Hauser SL, Jameson J, Loscalzo J, eds. Harrisons Principles of Internal Medicine. 18th ed. New York, NY: McGraw-Hill; 2012.
3. Chapter 7. ransport o oxygen and carbon dioxide in the blood. In: Levitzky MG, eds. Pulmonary Physiology. 8th ed. New York,
NY: McGraw-Hill; 2013.
4. Chesnutt MS, Prendergast J, avan E . Chapter 9. Pulmonary disorders. In: Papadakis MA, McPhee SJ, Rabow MW, eds. CURREN Medical Diagnosis & reatment 2014. New York, NY: McGraw-Hill; 2014.

83

TREATMENT APPROACH

PULMONARY

COPDExacerbation

Mild Exacerbation
Bronchodilators

Moderate or Severe
Exacerbation

Add or increase dose and/or requency o


short-acting bronchodilators
( 2-agonists and anticholinergics)

Steroids

Prednisone 40 mg 5 days

Antibiotics

NOT indicated

57-day course recommended

Treatment location

Outpatient

Hospitalize i 1 :
FEV1 50% predicted
Advanced age
Signi cant comorbidities
Worsening respiratory
acidosis
Failure to respond to initial
management
Marked increase in intensity
o symptoms
Frequent exacerbations

Figure 9-4. Treatment o a COPD exacerbation.

84

PULMONARY

TREATMENT APPROACH

COPDExacerbation
Case: A 77-year-old emale with severe COPD presents with a worsening
productive cough and shortness o breath. Her husband has recently had
an upper respiratory in ection.
How is a COPD exacerbation de ned?

T e Winnepeg criteria de nes major and minor criteria:


Major: Increase in sputum volume, increase in sputum purulence, or
worsening o baseline dyspnea
Minor criteria: recent URI, unexplained ever, increase in wheezing and
cough, increase in respirations or heart rate
An exacerbation can be mild (one major, one minor criteria), moderate (two
major), or severe (all three major criteria).
Who should receive steroids and/or antibiotics, and or how long?

Steroids: GOLD guidelines recommend prednisone 40 mg per day or


5 days or all exacerbations1
Steroids have been shown to reduce risks o early and late relapse,
reduce length o hospital stay, and improve lung unction 2
T e REDUCE trial showed that a 5-day course o prednisone 40 mg daily
was nonin erior to a 14-day course.3 Oral is nonin erior to IV steroids.
Antibiotics: GOLD guidelines recommend antibiotics or moderate or
severe exacerbations only.1 Antibiotic choice and duration is not wellde ned. In a meta-analysis, antibiotics were shown 4:
In ICU patients: antibiotics reduced re-exacerbation within 7 days to
1 month (OR 0.25, 95% CI 0.160.39), and mortality (OR 0.20, 95%
CI 0.060.62)
In hospitalized non-ICU patients: antibiotics reduced treatment ailures
only
In outpatients: antibiotics had a small but inconsistent e ect on relapse only
What interventions have been shown to NOT be e ective in shortening the

course o exacerbations or improving the outcome o patients?5


Mucolytic agents, chest physiotherapy, and mechanical percussion o the
chest
IV bronchodilators such as methylxanthines (theophylline) are not superior
to inhaled 2-agonists and anticholinergics.

REFERENCES
1. Global strategy for the diagnosis, management, and prevention of COPD: Revised 2014. Global initiative or Chronic obstructive lung
disease (GOLD). http://www.goldcopd.org (Accessed on June 4, 2014).
2. Niewoehner DE, Erbland ML, Deupree RH, Department o Veterans A airs Cooperative Study Group, et al. E ect o systemic
glucocorticoids on exacerbations o chronic obstructive pulmonary disease. N Engl J Med. 1999;340:1941.
3. Leuppi JD, Schuetz P, Bingisser R, et al. Short-term vs conventional glucocorticoid therapy in acute exacerbations o chronic
obstructive pulmonary disease: the REDUCE randomized clinical trial. JAMA. 2013;309(21):22232131.
4. Vollenweider DJ, Jarrett H, Steurer-Stey CA, Garcia-Aymerich J, Puhan MA. Antibiotics or exacerbations o chronic obstructive
pulmonary disease. Cochrane Database Syst Rev. 2012;12:CD010257.
5. Quon BS, Gan WQ, Sin DD. Contemporary management o acute exacerbations o COPD: a systematic review and meta-analysis.
Chest. 2008;133(3):756766.

84

85

ca

cs

<

PULMONARY
DIAGNOSTIC APPROACH

Interstitial Lung Disease

PULMONARY

DIAGNOSTIC APPROACH

Interstitial Lung Disease


Case: A 65-year-old male presents with progressive shortness o breath.
You suspect a orm o interstitial lung disease.
What initial testing should be done to evaluate or interstitial lung disease?

Lab testing: serology or rheumatologic disease (ANA, RF, ANCA)


Pulmonary unction testing: most have a restrictive de ect (but an
obstructive de ect can be seen in sarcoid-associated ILD, combined
COPD/ILD, hypersensitivity pneumonitis). DLCO is typically low
Imaging: CXR, while universally per ormed, is too insensitive to diagnose
ILD, and correlates poorly with disease stage1
All patients should have a chest C per ormed
In which patients is a lung biopsy warranted?

Patients with IPF usually are accurately diagnosed and staged by classic
C ndings
All other patients should be biopsied, including:
Patients with imaging consistent with other orms o ILD beyond IPF
Patients with concern or in ection, malignancy, or connective tissue
disease
What are the methods o obtaining lung tissue, and what are the pros/cons

or each?
1. ransbronchial biopsy: obtained via exible bronchoscopy.
Sa e (mortality < 0.05%, pneumothorax 0.72%)2
May miss diseases that are spatially heterogeneous; biopsy samples are
generally a ew millimeters in size and subject to crush arti act
Historically better or central lesions, but peripheral lesions are accessible
with the aid o ultrasound and new virtual 3D navigational systems
2. Surgical biopsy:
issue architecture is better preserved
Contraindications: DLCO < 35%, patients not suited or general anesthesia
a. Video-assisted thoracoscopic lung surgery (VA S): a minimally invasive
orm o thoracic surgery with lower morbidity than open thoracotomy
b. Open thoracotomy: surgical incision o 56 cm
May be pre erred over VA S i bleeding diathesis present or severe pleural
disease (better able to control bleeding)

REFERENCES
1. Pipavath S, Godwin JD. Imaging o interstitial lung disease. Clin Chest Med. 2004;25:455465.
2. Bradley B, Branley HM, Egan JJ, et al. Interstitial lung disease guideline: the British T oracic Society in collaboration with the
T oracic Society o Australia and New Zealand and the Irish T oracic Society. T orax. 2008;63(Suppl 5):v1v58.

85

86

yt

ke

yp

yp

ve

ve

yt

ya

PULMONARY
PATHOPHYSIOLOGY

Obstructive Sleep Apnea

PULMONARY

PATHOPHYSIOLOGY

Obstructive Sleep Apnea


Case: A 65-year-old obese man with hypertension complains o daytime
sleepiness. His wi e reports that he is a heavy snorer.
Which sleep observation is most use ul or diagnosing obstructive sleep

apnea (OSA)?1
Nocturnal gasping/choking is most predictive o a diagnosis o OSA
(LR = 3.3, 95% CI 2.1 4.6), whereas snoring is not use ul (LR = 1.1, 95%
CI 1.0 1.1)
What is the de nition o OSA?2
Unexplained excessive daytime sleepiness and 5 obstructed breathing
events/hour o sleep
Obstructed breathing event may be apnea (no breathing or 10 seconds)
or hypopnea (ventilation reduced by 50% or at least 10 seconds)
What is the pathophysiology o OSA?2
In OSA, the airway is narrowed at baseline (due to obesity, change in jaw
shape, tissue in ltration, etc.) but held open by increased airway muscle
activity during wake ulness
During sleep, the dilating airway muscles relax narrowed airway
(especially during inspiration) is no longer held patent airway collapse
What are the consequences o untreated OSA?2

Short-term: symptoms o disturbed sleep


Daytime sleepiness, impaired concentration, cognitive per ormance
Long-term: increased catecholamine levels with each arousal rom apnea/
hypopnea
Increased sympathetic tone hypertension (estimated to risk o
myocardial in arction by ~20% and risk o stroke by ~40%) and
tachyarrhythmias
Does treatment o OSA improve hypertension?3

Yes. In the HIPARCO trial, 194 patients with OSA and resistant hypertension
were randomized to CPAP or no CPAP. Primary outcome change in 24-hr
mean blood pressure a er 12 weeks o treatment
Main nding Mean blood pressure by 3.1 mm Hg (P = 0.02) in the
treatment group. Diastolic blood pressure also by 3.2 mm Hg (P = 0.005)

REFERENCES
1. Myers KA, Mrkobrada M, Simel DL. Does this patient have obstructive sleep apnea?: T e Rational Clinical Examination systematic
review. JAMA. 2013;310(7):731741.
2. Douglas NJ. Chapter 265. Sleep apnea. In: Longo DL, Fauci AS, Kasper DL, Hauser SL, Jameson J, Loscalzo J, eds. Harrisons Principles of Internal Medicine. 18th ed. New York, NY: McGraw-Hill; 2012.
3. Martnez-Garca MA, Capote F, Campos-Rodrguez F, et al.; Spanish Sleep Network. E ect o CPAP on blood pressure in patients
with obstructive sleep apnea and resistant hypertension: the HIPARCO randomized clinical trial. JAMA. 2013;310(22):24072415.

86

87

>

<

>

>

>

>

>

>

ve

PULMONARY
DIAGNOSTIC APPROACH

Pleural Ef usions

PULMONARY

DIAGNOSTIC APPROACH

Pleural Ef usions
Case: A 41-year-old male is admitted or ever and ound to have a right
pleural ef usion.
What are the two broad categories o e usions?

ransudate: due to imbalance in hydrostatic and oncotic pressures


Volume overload, e.g., CHF, nephrotic syndrome, cirrhosis
A pleural uid N -proBNP o > 1,500 pg/mL is diagnostic o CHF1
Exudate: due to lung/pleural in ammation or impaired lymphatics
In ection, malignancy, connective tissue diseases, chylothorax, trauma
How does Lights criteria help you di erentiate between causes o

e usions?2
T e e usion is exudative i one o the ollowing is met:
Pleural uid protein/serum protein > 0.5
Pleural uid LDH/serum LDH > 0.6
Pleural uid LDH > 2/3 upper limit o labs normal serum LDH
A pleural e usion that is exudative based on LDH alone (not protein) is
more suggestive o a parapneumonic or malignant e usion
T e sensitivity o Lights criteria is excellent (98%)2, but many transudates
will be misdiagnosed as exudates by Lights criteria (speci city is only
83%)2
I suspecting transudate despite Lights criteria, calculate the serum to
pleural uid protein gradient. I this gradient is > 3.1 g/dL, the patient
actually has a transudative e usion
What pleural uid studies help you identi y a speci c cause o an exudate?

Cell count/di erential, glucose, pH, amylase, gram stain, bacterial culture
Consider cytology or B marker
What pleural uid criteria can be used to diagnose a parapneumonic
e usion?
1. Loculated pleural uid
2. Pleural uid pH< 7.2
3. Pleural uid glucose < 3.3 mmol/L
4. Positive gram stain or culture o the pleural uid
5. Presence o pus in the pleural space

REFERENCES
1. Long DL, Fauci AS, Kasper DL, Hauser SL, Jameson JL, Loscalzo J. Harrisons Principles of Internal Medicine. McGraw-Hill. 2012.
2. Light RW. Pleural e usion. NEJM. 2002;346:19711977.

87

TREATMENT APPROACH

PULMONARY

PulmonaryEmbolism

P ULMONARY EMBOLIS M
CONFIRMED

Hypotens ive?
Morbidly obes e?
Very underweight?
Creatinine clearance <30 mL/min?
Increas ed ris k of bleeding?

YES

NO

LOW MOLECULAR
WEIGHT HEPARIN
WITH WARFARIN BRIDGE
OR
DIRECT ORAL ANTICOAGULANT

UNFRACTIONATED HEPARIN

Pers is tently hypotens ive?

NO

YES
Contraindications to thrombolytic therapy?
(intracranial neoplas m,
recent s urgery <10 days ,
platelets <100k, recent s troke, s evere
uncontrolled hypertens ion)

YES

CONSIDER
EMBOLECTOMY

NO

THROMBOLYSIS

Clinical improvement?

NO

YES

Figure 9-8. Management o acute pulmonary embolus.


88

PULMONARY

TREATMENT APPROACH

PulmonaryEmbolism
Case: A 37-year-old emale with metastatic breast cancer presents with
hypotension, pleuritic chest pain, and SOB; a pulmonary embolus (PE) is
diagnosed on CT angiography.
What steps should be taken to stabilize the patient rst?

Ensure adequate oxygenation, treat hypotension with IVF or pressors i


necessary. Monitor or cardiogenic shock.
What is rst-line therapy or acute PE, and what evidence supports this?
In general, lowmolecular-weight heparin (LMWH) is pre erred over IV
un ractionated heparin (UFH) or SC UFH
T ree meta-analyses showed mortality bene t o LMWH over UFH,
decreased recurrent V E disease, and decreased major bleeding1
LMWH also has greater bioavailability, ewer doses/day, and xed dosing
as compared to IV or SC UFH
LMWH and ondaparinux not compared head-to-head or PE. For DV :
no di erence in mortality, bleeding, or recurrent disease2
IV UFH pre erred i : patient is persistently hypotensive, has increased risk
o bleeding, is morbidly obese (concern or subcutaneous absorption), i
thrombolysis is considered, or i Cr clearance < 30 mL/min (LMWH not
well studied in this group, dosing must be monitored in renal ailure, and
P easier to monitor than anti-Xa)
When should thrombolytic therapy or embolectomy be considered?

T rombolytic therapy considered or patients with persistent hypotension


Improves RV unction and PA blood pressure in the short term, but no
evidence to demonstrate decreased mortality1
Embolectomy considered i thrombolytic therapy is contraindicated or
ails
When should long-term anticoagulation be initiated?

War arin should be started on same day as heparin and overlapped or


minimum o 5 days until INR therapeutic > 24 hours
Rivaroxaban: FDA approved, small decrease in risk o major bleeding
compared to war arin (1.1% vs. 2.2%, hazard ratio 0.49, 95% CI 0.310.79)3.
Dabigatran and apixaban may be superior to war arin or DV 4,5, but
insuf cient data or PE

REFERENCES
1. Kearon C, Akl EA, Comerota AJ, et al.; antithrombotic therapy or V E disease: antithrombotic therapy and prevention o thrombosis. Chest. 2012:141:e 419s.
2. Buller HR, Davidson BL, Decousus H, et al.; Matisse Investigators. Fondaparinux or enoxaparin or the initial treatment o symptomatic deep venous thrombosis: a randomized trial. Ann Intern Med. 2004;140:867873.
3. Buller HR, Prins MH, Lensin AW, et al.; EINS EIN-PE Investigators. Oral rivaroxaban or the treatment o symptomatic pulmonary embolism. N Engl J Med. 2012;366:12871297.
4. Agnelli G, Buller HR, Cohen A, et al.; AMPLIFY Investigators. Oral apixaban or the treatment o acute venous thromboembolism.
N Engl J Med. 2013;369:799808.
5. Schulman S, Kearon C, Kakkar AK; RE-COVER Study Group. Dabigatran versus war arin in the treatment o acute venous thromboembolism. N Engl J Med. 2009;361:23422352.

88

DIAGNOSTIC APPROACH

PULMONARY

PulmonaryFunctionTests

FEV1 /FVC ratio


(a ctual, not %
predicted)

70% FEV1 /FVC:


NORMAL or
RESTRICTIVE

RESTRICTIVE:
TLC <80%
predic ted

<70% FEV1 /FVC:


OBSTRUCTIVE

NORMAL:
TLC 80%
pre dicted

Mild:
FEV1 % pre d
>80%

Mode ra te:
FEV1 % pred
5080%

Irrevers ible FEV1


defec t

Revers ible FEV1


defe ct

COPD

As thma

Severe:
FEV1 % pred
3050%

Very s eve re:


FEV1 % pred
<30%

Exa mple pulmona ry function tes ting

IC

IC

VC

VC
VT
ERV

FRC
FRC

RV RV

FRC

(Re produce d, with permis s ion, from Weinbe rger


SE. Principles of Pulm onary Me dicine, 4th e d.
Philad elphia, PA: Saunders 2004.)

TLC

Unit

Ac tual

% Pre d

FEV1

0.48

26

FVC

1.70

72

FEV1 /
FVC
TLC

28

37

5.03

85

RV

3.07

140

FRC

3.94

149

DLCO

mL/
mm
Hg/
min

16.9

51

Figure 9-9. Interpretation o pulmonary unction tests.


89

PULMONARY

DIAGNOSTIC APPROACH

PulmonaryFunctionTests
Case: A 57-year-old male with 100 pack-year smoking history and lung
adenocarcinoma presents with hypoxemia. His pulmonary unction testing
is shown.
How do you assess the validity o pulmonary unction test results?

Reproducible: three similar ow-volume loops


Maximum e ort: evidenced by expiratory e ort lasting longer than
6 seconds
What are the steps to interpreting spirometry?

1. Evaluate FEV1/FVC (actual, not% predicted) ratio


< 70% = diagnostic o airway OBS RUC ION
70% = normal or suggestive o RES RIC IVE physiology
2. I obstructive, evaluate the reversibility o obstruction by administering
an inhaled bronchodilator and the severity o obstruction
An increase in FEV1% predicted by > 12% or 200 mL a er bronchodilator
suggests asthma
I you were to measure RV and FRC, they may be increased due to the
inability to ully exhale
3. I FEV1/FVC ratio > 70%, the next step is to con rm the presence o a
restrictive de ect by per orming lung volume measurements
LC < 80% predicted indicates a RES RIC IVE de ect
I RV is increased, this may indicate a concurrent obstructive de ect or
respiratory muscle weakness
What PFT abnormalities does the patient in the example have?

FEV1/FVC = 28% suggests an obstructive de ect. FEV1% predicted = 26%


indicates very severe obstructive de ect. High RV and FRC suggest inability
to ully exhale. Low DLCO and obstructive de ect suggest emphysematous
changes.
What are possible causes o a decrease in DLCO (%pred)?
DLCO measures the lungs ability to trans er gas
It can be decreased due to brosis o alveolar barrier, decrease in volume o
blood available to uptake oxygen (e.g., anemia, pulmonary hypertension),
decrease in sur ace or gas exchange (e.g., emphysema)

89

90

>

>

PULMONARY
CLASSIFICATION

PulmonaryHypertension

PULMONARY

CLASSIFICATION

PulmonaryHypertension
Case: A 57-year-old emale with scleroderma develops progressively
worsening shortness o breath.
How is pulmonary hypertension (PH) de ned?

Mean pulmonary artery pressure 25 mm Hg


What are the two major anatomic areas o pulmonary vasculature that can
lead to PH?1
Pre-capillary: a ects vasculature up to and including the pulmonary
capillaries pulmonary artery (PA) pressure, pulmonary vascular
resistance (PVR), but a normal pulmonary capillary wedge pressure (PCWP)
(re ects normal le -sided pressures)
Post-capillary: primarily a ects le -sided pressures PCWP PA
pressure. Secondarily, this can lead to right-sided heart ailure, i.e., mixed
pulmonary hypertension ( PA pressure, PVR, PCWP)
Which causes o PH are precapillary and which are postcapillary?

Precapillary:
WHO Group 1: Pulmonary arterial hypertension (PAH)1: diseases
that constrict pulmonary arteries and arterioles, including idiopathic or
amilial PAH, connective tissue disease, HIV, portal hypertension, and
drug e ects
WHO Group 3: Pulmonary hypertension secondary to hypoxemic
respiratory disease2: diseases that cause hypoxic vasoconstriction o
pulmonary vasculature, including COPD, interstitial lung disease, sleep
apnea, and restrictive lung de ects
WHO Group 43: Pulmonary hypertension secondary to chronic
thromboembolic disease: secondary to chronic obstruction o pulmonary arteries, leading to increased PVR
Postcapillary:
WHO Group 24: Pulmonary hypertension secondary to lef -sided
heart disease: includes diseases resulting in le -sided systolic, diastolic,
or valvular disease
What about WHO Group 5?

T is miscellaneous group can a ect all areas o the cardiopulmonary system,


and includes hematologic and vasculitic disorders

REFERENCES
1. Simonneau G, Gatzoulis MA, Adatia I, et al. Updated clinical classi cation o pulmonary hypertension. J Am Coll Cardiol.
2013;62:D34D41.
2. Seeger W, Adir S, Barbera JS, et al. Pulmonary hypertension in chronic lung diseases. J Am Coll Cardiol. 2013;62:D109D116.
3. Kim NH, Delcroix M, Jenkins DP, et al. Chronic thromboembolic pulmonary hypertension. J Am Coll Cardiol. 2013;62:D92D99.
4. Vahicery JL, Adir Y, Barbera JA, et al. Pulmonary hypertension due to le heart diseases. J Am Coll Cardiol. 2013;62:D100D108.

90

rl

ri

<

<

ri

91

ri

>

RHEUMATOLOGY
CLASSIFICATION

Acute Monoarticular Arthritis

RHEUMATOLOGY

CLASSIFICATION

Acute Monoarticular Arthritis


Case: A 60-year-old woman with type II diabetes presents with an acutely
pain ul and swollen right knee. You are preparing to per orm an
arthrocentesis.
What are the common causes o acute monoarthritis?1

1. Septic arthritis
Nongonococcal: Staphylococcus aureus, -hemolytic streptococci, Streptococcus pneumoniae, Lyme disease, or gram-negative in ection
Gonococcal: Neisseria gonorrhoeae in ection (can also present with migratory tenosynovitis with typical skin lesions). Di cult to grow in culture
2. In ammatory arthritis: crystal-induced (gout or pseudogout), spondyloarthropathy
3. Nonin ammatory: osteoarthritis
4. raumatic: hemarthrosis, meniscal tear or racture
When should an arthrocentesis be per ormed or an acute monoarthritis?
For any case o unexplained monoarthritis (need to exclude septic arthritis)
What is the major contraindication to per orming an arthrocentesis?1
Overlying cellulitis (risk o introducing in ection into joint space)
Anti-coagulation with war arin is not a contraindication (as long as INR is
< 3.0 and a small-gauge needle is used)
What synovial uid tests help you determine the cause o an e usion?2

Clarity
Color
WBC count
Gram stain or culture
Crystal analysis
How accurate is an elevated synovial uid WBC count or diagnosing septic

arthritis?3
Depends on level o elevation o synovial uid WBC count
WBC count > 50k/l: positive likelihood ratio o 7.7 or septic arthritis
WBC count > 100k/l: positive likelihood ratio o 28.0 or septic arthritis
How does the synovial uid neutrophil count change the likelihood o septic
arthritis?3
A neutrophil count 90% increases the likelihood ratio o septic arthritis by
3.4, whereas a neutrophil count < 90% decreases the likelihood ratio by 0.34

REFERENCES
1. Imboden JB. Chapter 4. Approach to the patient with arthritis. In: Imboden JB, Hellmann DB, Stone JH. eds. Current Rheumatology
Diagnosis & reatment. 3rd ed. New York, NY: McGraw-Hill; 2013.
2. Fye KH, Imboden JB. Chapter 2. Joint aspiration & injection. In: Imboden JB, Hellmann DB, Stone JH, eds. CURREN Rheumatology Diagnosis & reatment. 3rd ed. New York, NY: McGraw-Hill; 2013.
3. Margaretten ME, Kohlwes J, Moore D, Bent S. Does this adult patient have septic arthritis? JAMA. 2007;297(13):14781488.

91

CLASSIFICATION

RHEUMATOLOGY

Antinuclear Antibodies

Disease

ANA with High


Sensitivity (Rule -Out)

ANA with High


Speci city (Rule -In)

Systemic lupus
erythematosus (SLE)

ANA

Anti-dsDNA
Anti-Smith

Drug-induced SLE

ANA
Anti-histone

None

Mixed connective Tissue


disease (MCTD)

ANA
Anti-RNP

None

Sjogrens syndrome

None

Anti-Ro/SSA
Anti-La/SSB

Scleroderma (limited)

ANA

Anti-centromere

Scleroderma (systemic)

ANA

Anti-SCL-70

ANA, antinuclear antibodies; dsDNA, double-stranded DNA; RNP, ribonucleoprotein; SSA, Sjogrens
syndrome A; SSB, Sjogrens syndrome B.
(Modi ed, with permission, rom Imboden JB, Hellmann DB, Stone JH. CURRENT diagnosis &treatment:
Rheumatology. 3rd ed. New York, NY: McGraw-Hill Education; 2013. Table 3-3.)

Figure 10-2. Test characteristics o antinuclear antibodies in speci c


rheumatologic conditions.

92

RHEUMATOLOGY

CLASSIFICATION

Antinuclear Antibodies
Case: A 32-year-old woman presents with joint pain, atigue, and a acial
rash. You would like to send an antinuclear antibody (ANA) to diagnose
lupus.
What is an ANA?1

Any auto-antibody directed against a component o a cells nucleus


T e test captures many di erent antibodies, both generalized (e.g., ANA)
or speci c to a nuclear antigen (e.g., anti-Smith)
How common is a positive ANA in a healthy individual?1
An ANA titer o 1:40 occurs in 32% o healthy adults and a titer o 1:160
in 5% o healthy adults
What are some nuclear components that an ANA may be directed against?1
Histones, double- or single-stranded DNA, ribonucleoprotein (RNP)
complexes
For which conditions are di erent ANAs diagnostically use ul?1

Highly sensitive ANAs (help ul or ruling-out disease when antibody is


negative)
ANA sensitive or SLE, MC D, and scleroderma
Anti-histone antibody sensitive or drug-induced SLE
Anti-RNP antibody sensitive or MC D
Highly speci c ANAs (help ul or ruling-in disease when antibody is
positive)
Anti-dsDNA and anti-Smith antibodies speci c or SLE
Anti-centromere antibody speci c or scleroderma (limited)
Anti-SCL-70 antibody speci c or scleroderma (systemic)
Anti-Ro/SSA and anti-La/SSB antibodies speci c or Sjogrens
syndrome
How are ANA results reported?1

iter: 1:40 or higher is generally considered positive


Staining pattern: depends on the underlying type o ANA but usually nonspeci c, except or centromere pattern (highly speci c or limited scleroderma)
Utility o staining pattern has been largely replaced by the development o
antibody-speci c testing
Which ANAs are associated with a higher risk or multiorgan involvement?1
Anti-dsDNA: risk or lupus nephritis
Anti-SCL-70: risk or di use skin involvement and interstitial lung disease

REFERENCE
1. Nakamura MC, Imboden JB. Chapter 3. Laboratory diagnosis. In: Imboden JB, Hellmann DB, Stone JH. eds. Current Rheumatology
Diagnosis & reatment 3rd ed. New York, NY: McGraw-Hill; 2013.

92

DIAGNOSTIC APPROACH

RHEUMATOLOGY

Antiphospholipid Syndrome
Clinical Criteria
(at least one)

1. Vascular thrombosis: arterial, venous, or small vessel


thrombosis without in ammation in the vessel wall
2. Pregnancy morbidity (any o ):
a. Unexplained etal death 10th week o gestation
b. Preeclampsia, eclampsia, or placental insuf ciency
be ore 34th week o gestation
c. 3 unexplained spontaneous abortions be ore
10th week o gestation

Laboratory
Criteria (at least
one)

1. Lupus anticoagulant positive


2. Anti-cardiolipin IgG and/or IgM at medium to high titer
3. Anti- 2 glycoprotein-1 IgG and/or IgM titer > 99th
percentile

Patients must meet at least one laboratory criteria and clinical criteria within 5 years o each other
Laboratory positivity must be present on more than one occasion > 12 weeks apart

S te p 1: Functiona l clotting a s s ay

No prolonge d
clotting time

P rolonge d clotting
LAC* not confirme d

S te p 2: Mixing s tudy (1:1 pa tie nt/control)

Clotting time
doe s not
corre ct

Clotting time
corre cts
Factor deficiency

Inhibitor

S te p 3: Addition of exce s s phos pholipid

Clotting time
corre cts

Clotting time
doe s not
corre ct

Positive for LAC

Consider other factor inhibitor

*LAC = Lupus a nticoa gula nt


(Reproduced, with permission, rom Imboden JB, Hellmann DB, Stone JH. CURRENT Diagnosis &Treatment: Rheumatology. 3rd ed. New York: McGraw-Hill Education, 2013. Figure 23-1.)

93 Figure 10-3. Diagnosis o antiphospholipid syndrome.

RHEUMATOLOGY

DIAGNOSTIC APPROACH

Antiphospholipid Syndrome
Case: A 45-year-old emale presents with an unprovoked deep venous
thrombosis. She had a miscarriage at 20 weeks gestation 20 years ago.
You suspect antiphospholipid syndrome (APLS).
What types o vascular phenomena might be seen in patients with APLS?

Arterial: stroke; acute coronary syndrome; renal artery stenosis; in arcts


o spleen, bowel, pancreas, and adrenals; arterial thrombosis o upper and
lower extremities
Venous: DV , pulmonary embolism, intraabdominal vein thrombosis
(e.g., Budd-Chiari)
What is the di erence between primary and secondary APLS?1
1: Occurs without history o autoimmune disease
Can be due to in ections, malignancies, and drugs
2: Associated with autoimmune disease, usually systemic lupus erythematosus (one-third o patients have APLS antibodies), Sjogrens syndrome,
dermatomyositis, rheumatoid arthritis
What are the diagnostic criteria o APLS?1,2

At least one clinical and one laboratory criteria within 5 years o each other
Lupus anticoagulant: a unctional assay to detect a group o inhibitors
Anti-cardiolipin and anti- 2 glycoprotein-1 antibodies: common in low
titers (present in 110% o normal individuals); must be a high titer to be
pathologic
How is lupus anticoagulant detected?
Anticoagulant is a misnomer; lupus anticoagulant is a procoagulant in
vivo and anticoagulant in vitro (prolongs P in vitro)
Diagnosed with a two-step mixing study
How should APLS be managed?1

Laboratory criteria only: no treatment


Laboratory and clinical criteria: treat APLS patients with li elong war arin
(INR goal 2.53.5) aspirin 80 mg
Pregnant APLS patients: therapeutic low-molecular weight heparin and
aspirin 80 mg

REFERENCES
1. Moutsopoulos HM, Vlachoyiannopoulos PG. Chapter 320. Antiphospholipid antibody syndrome. In: Longo DL, Fauci AS, Kasper
DL, Hauser SL, Jameson J, Loscalzo J, eds. Harrisons Principles of Internal Medicine. 18th ed. New York, NY: McGraw-Hill; 2012.
2. Miyakis S, Lockshin MD, Atsumi , et al. International consensus statement on an update o the classi cation criteria or de nite
antiphospholipid syndrome (APS). J T romb Haemost. 2006; 4(2):295306.

93

94

yp

ys

ys

RHEUMATOLOGY
TREATMENT APPROACH

ChronicGout

RHEUMATOLOGY

TREATMENT APPROACH

ChronicGout
Case: A 62-year-old man with gout presents with his third are o gout
over the past 12 months.
What are the indications or hypouricemic therapy in patients with chronic

gout?1
Frequent gouty attacks ( 2x/year)
Gouty attack with stage 25 chronic kidney disease (CKD)
Evidence o chronic gouty arthritis (e.g., tophi, erosions on plain lms)
Uric acid stones
What agents are available to treat chronic gout and how do they work?2

Decrease joint in ammation: anti-in ammatory agents (e.g., low-dose


colchicine or NSAIDs)
Must be taken daily and with a urate-lowering therapy
Decrease uric acid production: xanthine oxidase inhibitors
Allopurinol: most commonly prescribed, but does have risk or severe
allergic reactions (especially with rapid uptitration)
Normal GFR start with 100 mg daily and uptitrate by 100 mg i uric
acid goal (< 6.0 mg/dL) not met
CKD consider starting with lower dose (e.g., 50 mg daily) and
escalate dose more slowly (e.g., by 50 mg) i uric acid goal not met
Febuxostat: newer and more expensive; use i allopurinol intolerance
Increase uric acid urinary excretion: uricosuric agents (e.g., probenecid)
Should be started in patients that underexcrete uric acid: < 600 mg o
uric acid in a 24-hour urine sample
Must have good renal unction and no history o nephrolithiasis
In the patient above, when should you start a uric-acid lowering therapy?1

Do not start during acute are o gouty arthritis. Rather, treat acute are
rst with an anti-in ammatory agent (e.g., colchicine) until in ammation
subsides
T en, start uric-acid lowering therapy (e.g., allopurinol) with overlap o
colchicine. Continue overlap until uric acid level normalizes, no gouty
attacks or 6 months, and tophi (i present) resolve
Do any other common medications have a mild uricosuric e ect?3
Losartan (use ul option or patients with gout and hypertension)

REFERENCES
1. Schumacher H, Chen LX. Chapter 333. Gout and other crystal-associated arthropathies. In: Longo DL, Fauci AS, Kasper DL,
Hauser SL, Jameson J, Loscalzo J, eds. Harrisons Principles of Internal Medicine. 18th ed. New York, NY: McGraw-Hill; 2012.
2. Burns C, Wortmann RL. Chapter 44. Gout. In: Imboden JB, Hellmann DB, Stone JH, eds. CURREN Rheumatology Diagnosis &
reatment. 3rd ed. New York, NY: McGraw-Hill; 2013.
3. Keenan R , Krasnokutksy S, Pillinger MH. Chapter 255. Gout, pseudogout, and osteoarthritis. In: McKean SC, Ross JJ, Dressler
DD, Brotman DJ, Ginsberg JS, eds. Principles and Practice of Hospital Medicine. New York, NY: McGraw Hill; 2102:21312139

94

DIAGNOSTIC APPROACH

RHEUMATOLOGY

CRPversus ESR

ESR

CRP

Use ulness or diagnosing


acute in ammation

Comparatively lower

Comparatively higher

Use ulness in monitoring


disease progression

High

High a

Accuracy in low-grade
in ammation

More speci c, less


sensitive

More sensitive, less


speci c

Range o values

Limited

Wide

Af ected by size/shape/
number o RBCs

Yes

No

With the exception o systemic lupus erythematous.

CRP

ESR

7 days
(Adapted rom: From Kushner I, Rzewnicki DL. The acute phase response. In: Mackowiak PA, ed. Fever, Basic
Mechanisms and Management. 2nd ed. Philadelphia, PA: Lippincott-Raven; 1997:165176.)

Figure 10-5. Test characteristics o CRP and ESR (above); CRP and ESR
changes over disease course (below).
95

RHEUMATOLOGY

DIAGNOSTIC APPROACH

CRPversus ESR
Case: A 73-year-old emale presents with 4 weeks o pain and stif ness
in shoulders and hips and a new-onset headache. Her erythrocyte
sedimentation rate is 105 mm/hr.
How does the erythrocyte sedimentation rate (ESR) serve as a proxy or

in ammation?
ESR is de ned as the rate (mm/hr) at which red blood cells suspended in
plasma settle when placed in a Westergren tube. T is rate is in uenced by:
1. Presence o in ammatory byproducts, especially brinogen, which cause
clumping o red blood cells and a aster rate o RBC settling
2. Number o RBCs; settling is impeded by density o RBCs. Anemia ESR
3. Age and gender; upper limit o normal values can be adjusted or such:
Women = (age in years + 10)/2; Men = (age)/2
What are the most common causes o an elevated ESR (> 100 mm/hr)?
Rheumatic: giant cell arteritis (GCA) or polymyalgia rheumatica (PMR)
Nonrheumatic: in ection, malignancy, renal disease1
How does C-reactive protein (CRP) di er rom ESR?

ESR rate changes relatively slowly as patients condition worsens/improves,


whereas CRP changes rapidly, and may better re ect acute disease2
CRP elevations: more strongly associated with in ection, especially
bacterial (> 10 mg/dL)3
ESR value limited by length o Westergren tube (120 mm), CRP has a
wider range
CRP is more sensitive to low-grade in ammation and in general less speci c
(may re ect smoking, diabetes, depression, chronic atigue, hormone
replacement therapy)
What is the role o measuring CRP and ESR in autoimmune disease?

Both are commonly used to monitor disease activity and clinical response
to therapy (e.g., monitoring course in rheumatoid arthritis, diagnosing
GCA/PMR)
Discrepancies between the two can be in ormative
For example, lupus: ESR may be markedly elevated; CRP is of en not4
What other settings can ESR and CRP be use ul in?
CRP and ESR can help prognosticate malignant disease,5 type II diabetes,
peripheral vascular disease, and ischemic stroke. CRP may be use ul in
cardiovascular disease.2

REFERENCES
1. Fincher RM, Page MI. Clinical signi cance o extreme elevation o the erythrocyte sedimentation rate. Arch Intern Med.
1986;146(8):15811583.
2. Gabay C, Kushner I. Acute-phase proteins and other systemic responses to in ammation. N Eng J Med. 1999;340(6):448454.
3. Vanderschueren S, Deeren D, Knockaert DC, Bobbaers H, Bossuyt X, Peetermans W. Extremely elevated C-reactive protein. Eur J
Intern Med. 2006;17(6):430433.
4. Gaitonde S, Samols D, Kushner I. C-reactive protein and systemic lupus erythematous. Arthritis Rheum. 2008;59(12):18141820.
5. Legou E, Rodriguez C, Picot MC, et al. C-reactive protein serum level is a valuable and simple prognostic marker in non Hodgkins
lymphoma. Leuk Lymphoma. 1998;31(3-4):351357.

95

DIAGNOSTIC APPROACH

RHEUMATOLOGY

Sarcoidosis
Clin ic a l fe a ture s s u g g e s tive o f s a rc o id o s is :
- Bilateral hilar adenop athy on CXR
- Erythema nodos um, macular papular s kin les ions
- Uveitis
- Hyp ercalcemia, CNS or cardiac involvement

Rule o ut o the r p o s s ib ilitie s :


- TB
- Environmental or occupational
expos ures (e.g., s ilicos is )
- Malignancy (e.g., lymphoma)

Biops y*

Negative biops y but


no evidence of
alternative d iagnos is

Noncas eating
granuloma

Sarcoid os is likely

Abnormal

Further imaging and biops y


of s econd s ite if pos s ible

Normal

Seek other diagnos is


*Biopsy not necessary or Lo grens syndrome.
(Modi ed with permission rom Longo DL, Fauci AS, Kasper DL, et al. Harrisons Principles o Internal Medicine.
18th ed. New York, NY: McGraw-Hill Education; 2012.)

Figure 10-6. Diagnostic algorithm or suspected sarcoidosis.


96

RHEUMATOLOGY

DIAGNOSTIC APPROACH

Sarcoidosis
Case: A 45-year-old emale presents with chronic dry cough and is ound
to have nodular lesions on her anterior legs. You suspect sarcoid.
What clinical eatures are most suggestive o sarcoidosis?

T oracic involvement is most common, occurring in > 90% o patients,1


usually symmetrical hilar adenopathy (unilateral only in 35%)
Skin lesions: varies widely, with erythema nodosum and macular-papular
lesions among the most common
Eyes: 25% o patients have ocular involvement (uveitis)
Miscellaneous: cardiac (arrhythmias), hypercalcemia, cranial nerve or other
CNS involvement
When clinical eatures suggest sarcoid, what other diseases should be ruled
out?
uberculosis, malignancy, and occupational/environmental exposures
How do you conf rm the diagnosis o sarcoidosis?

issue biopsy demonstrating noncaseating granulomas


With the exception o Lo gren syndrome (acute presentation o erythema
nodosum, hilar adenopathy, and arthritis, seen in 2050% o cases2), and
asymptomatic patients with only hilar adenopathy, all other patients need
biopsy to con rm a diagnosis o sarcoidosis1
Biopsy o erythema nodosum is not help ul as it reveals nonspeci c
septal panniculitis. Biopsy any other lesion, e.g., skin, pulmonary, lymph
nodes
What other tests may be per ormed in suspected sarcoid?

Assess or organ involvement: LF s, calcium, eye exam, EKG, and ECHO


PF s: decreased di usion capacity and restrictive and/or obstructive de ect1
Serum ACE level > 2 times upper limit o normal3
Bronchoalveolar lavage: lymphocytosis > 2 times upper limit o normal3
Interstitial disease on C is nonspeci c, but adenopathy > 2 cm supports
sarcoidosis
What is the initial treatment o sarcoidosis?
Minimal symptoms = observe; a ecting eyes/skin only = topical steroids;
multiple organs = steroids, consider methotrexate, NF inhibitors. Consider
ICD or cardiac involvement

REFERENCES
1. Iannuzzi, MC, Fontana JR. Sarcoidosis: clinical presentation, immunopathogenesis, and therapeutics. JAMA. 2011;305(4):
391399.
2. Newman LS, Rose CS, Maier, LA. Sarcoidosis. N Engl J Med. 1997;336(17):12241234.
3. Baughman RP, Lower EE. Chapter 329. Sarcoidosis. In: Longo DL, Fauci AS, Kasper DL, Hauser SL, Jameson J, Loscalzo J. eds.
Harrisons Principles of Internal Medicine. 18th ed. New York, NY: McGraw-Hill; 2012.

96

PATHOPHYSIOLOGY

RHEUMATOLOGY

Scleroderma Renal Crisis

Endo the lia l Wa ll Injury*

Intimal
proliferation

Platelet
aggregation

Fibrin
depos ition

Luminal narrowing of arteries

TREATMENT

Decreas ed GFR

Renin-angiotensin
s ystem inhibition
(ACE-inhibitors )

Increas ed renin
Severe hyertens ion

Progres s ive renal failure

*The exa ct cau se of end othelial wa ll injury re mains unknown but


may b e due to infection, is chemia or immune-mediated proces s es 1

Figure 10-7. Pathogenesis and treatment o scleroderma renal crisis.

97

RHEUMATOLOGY

PATHOPHYSIOLOGY

Scleroderma Renal Crisis


Case: A 62-year-old man with systemic scleroderma presents with a
headache and is noted to have severe hypertension, hemolytic anemia,
and an acute rise o serum creatinine to 3.0 mg/dL.
What is the di erential diagnosis o this patients presentation?

Scleroderma renal crisis (SRC), malignant hypertension or other thrombotic


microangiopathies (e.g., P, HUS)
How does SRC typically present?2
Symptoms: headache, blurred vision, chest pain, decreased urine output
Signs: hypertension and acute, progressive renal ailure
Laboratory: urinalysis typically normal or with mild proteinuria
hematuria
Microangiopathic hemolytic anemia also commonly seen (43% o patients
with SRC) with schistocytes on smear thrombocytopenia
In which orm o scleroderma systemic or limited is renal crisis more
common?2
Systemic scleroderma (rare in patients with limited scleroderma
approximately 1%)
What is the pathophysiology o scleroderma renal crisis and based on

this, what is the f rst-line treatment?3,4


Pathophysiology:
rigger event (exact mechanism unknown) endothelial damage
vascular luminal narrowing decreased renal per usion compensatory rise in renin production
Renin continues to increase in maladaptive pattern severe hypertension and ultimately decreased GFR
reatment: renin-angiotensin system (RAS) inhibition with ACE-inhibitors
Captopril is the agent o choice, and should be rapidly uptitrated to
achieve a normotensive state (maximum dose 150 mg 3x/day)
Does renal biopsy di erentiate SRC rom other microangiopathies?3

No: need to make diagnosis on clinical grounds (key eature is evidence o


systemic scleroderma)
Can SRC be prevented?4
No: there is no role or ACE-inhibition in the prophylaxis o SRC
However, requent BP monitoring in high risk-patients (early di use
scleroderma, prior treatment with steroids) allows or early detection

REFERENCES
1. Wigley FM. Vascular disease in scleroderma. Clin Rev Allergy Immunol. 2009;36(23):150175.
2. Nast CC, Adler SG. Chapter 34. T rombotic microangiopathies. In: Lerma EV, Berns JS, Nissenson AR, eds. Current Diagnosis &
reatment: Nephrology & Hypertension. New York, NY: McGraw-Hill; 2009.
3. Dellaripa PF, odd DJ. Chapter 254. Rheumatologic emergencies. In: McKean SC, Ross JJ, Dressler DD, Brotman DJ, Ginsberg JS,
eds. Principles and Practice of Hospital Medicine. New York, NY: McGraw-Hill; 2012.
4. Rhew EY, Barr WG. Scleroderma renal crisis: new insights and developments. Curr Rheumatol Rep. 2004;6(2):129136.

97

RHEUMATOLOGY

TREATMENT APPROACH

SystemicLupus Erythematosus (SLE) Flare

SYSTEMIC LUPUS
ERYTHEMATOSUS (SLE) FLARE

Initia l la b o ra to ry te s ting :
Anti-ds DNA, ESR/CRP, and C3, C4
Rule out lupus nephritis : creatinine, urinalys is and urine s ediment
NO need to s end ANA or other autoantibodies if the
diagnos is of SLE has previous ly been es tablis hed

Signs of lupus nephritis pres ent (i.e., active urine s ediment)

Yes

Urgent nephrology cons ult


for cons ideration of renal
biops y and initiation of
s teroids

No

1. Rheumatology cons ult


2. As s es s clinical s everity and treat:
Seros itis : EKG, CXR, +/- ECHO
NSAIDs , + /- colchicine, low dos e
s teroids
Autoimmune hemolytic anemia: cons ult
hematology, CBC, reticulocyte count,
Coombs tes t, +/ APLS labs
High dos e s teroids
Lupus cerebritis : MRI, lumbar puncture
High dos e s teroids
Arthritis :
NSAIDs , low dos e s teroids
3. Cons ider hydroxychloroquine if not already
initiated

Figure 10-8. Treatment algorithm or SLE are.

98

RHEUMATOLOGY

TREATMENT APPROACH

SystemicLupus Erythematosus (SLE) Flare


Case: A 34-year-old woman with a history o systemic lupus erythematosus
(SLE) presents with 2 days o pleuritic chest pain and joint pain. You suspect
an SLE are.
What actors might precipitate an acute SLE are?1

Exposure to sun/UV light, in ection, stress, and surgery. Medications (e.g.,


NF- inhibitors) may lead to drug-induced lupus erythematosus (DILE)
What are the common presentations o SLE ares and which o these
necessitate hospitalization?
Common: rash, joint pain, ever, serositis (pleuritis, pericarditis)
May require hospitalization: cardiac mani estations (pericarditis, myocarditis, Libman Sachs endocarditis), hematologic mani estations (autoimmune
hemolytic anemia, anti-phospholipid syndrome), nephritis, neurologic
mani estations (lupus cerebritis)
What initial workup should be ordered on all patients presenting with an

SLE are?
Markers correlating with disease severity: anti-dsDNA, ESR/CRP, and
C3, C4
All patients should be screened or lupus nephritis with a creatinine,
urinalysis and urine sediment, ollowed up with a quanti cation o urine
protein excretion as necessary
Kidney disease is common and carries a high morbidity/mortality
Organ-speci c imaging/testing (see image) to assess clinical severity
What are principles o management o an SLE are?

Lupus nephritis: nephrology consultation steroids, renal biopsy


Cyclophosphamide or mycophenolate mo etil af er diagnosis established
General SLE are: rheumatology consultation
Low-dose prednisone or arthritis, serositis, rash
High-dose prednisone or autoimmune hemolytic anemia, cerebritis,
di use alveolar hemorrhage
Hydroxychloroquine to reduce urther ares (not immunosuppressive; can
be used postoperative and during in ection). akes time to take e ect
A steroid-sparing immunosuppressive: e.g., azathioprine, methotrexate,
or mycophenolate mo etil. Cyclophosphamide or severe disease or lupus
nephritis

REFERENCE
1. Criscione-Schreiber L, Bolster MB. Chapter 256. Systemic lupus erythematosus. In: McKean SC, Ross JJ, Dressler DD, Brotman DJ,
Ginsberg JS, eds. Principles and Practice of Hospital Medicine. New York, NY: McGraw-Hill; 2012.

98

DIAGNOSTIC APPROACH

RHEUMATOLOGY

Temporal Arteritis

Test

Positive Likelihood
Ratio (95% CI)

Negative Likelihood
Ratio (95% CI)

Historical eatures
Jaw claudication

4.2 (2.86.2)

0.72 (0.650.81)

Diplopia

3.4 (1.38.6)

0.95 (0.910.99)

Headache

1.2 (1.11.4)

0.70 (0.570.85)

Beaded temporal artery

4.6 (1.118.4)

0.93 (0.880.99)

Prominent temporal artery

4.3 (2.18.9)

0.67 (0.50.89)

Tender temporal artery

2.6 (1.93.7)

0.82 (0.740.92)

Exam Findings

(Data rom Smetana GW, Shmerling RH. Does this patient have temporal arteritis? JAMA. 2002;287(1):
92101.)

Figure 10-9. Utility o historical eatures and exam ndings in


diagnosing temporal arteritis.

99

RHEUMATOLOGY

DIAGNOSTIC APPROACH

Temporal Arteritis
Case: A 72-year-old woman presents with headache and pain while
chewing meat. You are considering a diagnosis o temporal arteritis (TA).
What is the complication o TA that warrants prompt diagnosis?1

Blindness (usually irreversible due to posterior ciliary artery occlusion)


Which historical eatures and exam f ndings are most use ul or diagnosing

TA?2
Historical eatures
Rule-in (high positive likelihood ratio): jaw claudication and diplopia
Not use ul: headache (positive LR = 1.2), history o polymyalgia
rheumatica (positive LR = 0.97), or constitutional symptoms such as
ever (positive LR = 1.2)
Exam ndings
Rule-in (high positive likelihood ratio): beading, prominence, or
tenderness o temporal artery
Not use ul: any undoscopic abnormality (positive LR = 1.1) or scalp
tenderness (positive LR = 1.6)
I the clinical presentation and physical exam are concerning or TA, what

laboratory test should be ordered next?2


Erythrocyte sedimentation rate (ESR): most help ul to rule-out A
I ESR is normal, then decreased likelihood o A (negative LR = 0.2)
I ESR > 100 mm/hr, then increased likelihood o A, but positive
likelihood ratio only 1.9
LR lower than other historical eatures or exam ndings
Not use ul: anemia (positive LR = 1.5) or moderately elevated ESR o
> 50 mm/hr (positive LR = 1.2)
When should treatment be initiated on a patient with suspected TA?1
Immediately (do not await results o biopsy) with high-dose steroids (i.e.,
prednisone 4060 mg daily or consider IV steroids i acute vision changes
present)
How long does it take or steroids to lower the yield o a temporal artery
biopsy?3
Biopsies may show evidence o arteritis even af er more than 2 weeks o
corticosteroid therapy when active disease is present

REFERENCES
1. Hellmann DB.Chapter 30. Giant cell arteritis & polymyalgia rheumatica. In: Imboden JB, Hellmann DB, Stone JH, eds. CURREN
Rheumatology Diagnosis & reatment. 3rd ed. New York, NY: McGraw-Hill; 2013.
2. Smetana GW, Shmerling RH. Does this patient have temporal arteritis? JAMA. 2002;287(1):92101.
3. Achkar AA, Lie J , Hunder GG, et al. How does previous corticosteroid treatment a ect the biopsy ndings in giant cell (temporal)
arteritis? Ann Intern Med. 1994;120(12):987992.

99

100

ri

ch

RHEUMATOLOGY
DIAGNOSTIC APPROACH

Vasculitis

RHEUMATOLOGY

DIAGNOSTIC APPROACH

Vasculitis
Case: A 34-year-old emale presents with hemoptysis. She also has
microscopic hematuria. You suspect that she has vasculitis.
In which patients should you suspect vasculitis?

Patients presenting with: constitutional symptoms, unexplained multiorgan


ailure, new glomerulonephritis, di use alveolar hemorrhage (DAH),
mononeuritis multiplex, palpable purpura
What are the clinical mani estations o vasculitis?1

T e mani estations depend on the size and speci c blood vessels involved
(see chart)
How do you approach the initial investigation o patients with suspected

vasculitis?1
1. Exclude vasculitic mimics:
In ections can be an underlying cause o vasculitis: hepatitis B and C,
in ective endocarditis
T ese need to be treated be ore starting immunosuppressive therapy
Drugs (e.g., hydralazine, propylthiouracil, allopurinol) can induce an
anti-neutrophil cytoplasmic antibody (ANCA)-associated vasculitis2
2. Vasculitic labs: ANCA, ESR, CRP, complement levels, antinuclear antibody
(ANA), hepatitis serologies, cryoglobulins, rheumatoid actor, eosinophil
count
3. Additional evaluation based on size o vessel involved:
a. Large vessels: check or equal blood pressures and pulses bilaterally;
carotid, subclavian, and emoral bruits; temporal artery tenderness
b. Medium vessels: abdominal imaging i abdominal pain is present
c. Small vessels: urine sediment
4. Biopsy and/or angiography i diagnosis remains unclear
When should empiric treatment with steroids be considered?
Concern or irreversible organ involvement or high morbidity/mortality,
(e.g., glomerulonephritis, DAH, and mononeuritis multiplex)
Giant cell arteritis: biopsy results do not change until 2 weeks af er steroids
are started; treat early to avoid visual loss1

REFERENCES
1. Coblyn JS, Bermas B, Weinblatt M, Hel gott S. Brigham and Womens Experts Approach to Rheumatology. Burlington, MA: Jones
and Bartlett Learning; 2011.
2. Choi HK, Slot MC, Pan G, et al. Evaluation o antineutrophil cytoplasmic antibody seroconversion induced by minocycline, sul asalazine, or penicillamine. Arthritis Rheum. 2000;43(11):24882492.

BKP/N 1259643409

ISBN 9781259643408
100

Potrebbero piacerti anche